Official Report (for web) - 42nd International Chemistry Olympiad

Transcription

Official Report (for web) - 42nd International Chemistry Olympiad
42nd International Chemistry Olympiad
Official Report
July 19-28, 2010, Japan
42nd International Chemistry Olympiad
Closing Ceremony Remarks
It is a pleasure to see you again. I hope you have enjoyed the 42nd International Chemistry
Olympiad.
Louis Pasteur once said, “Science has no borders, but scientists have their own
fatherlands.” Science is objective. But making scientific discoveries and accumulating
scientific knowledge are human activities.
Every region and ethnic group has its own culture, and the world’s scientists have grown up
in these diverse cultures. But for the scientists of the 21st century, I believe we have a
mission to work in close solidarity, while respecting our diverse cultures, for the sustaining
of humankind. We must build a civilization that accepts and respects diverse cultures.
The 20th century was an era of international competition, symbolized by war and economic
rivalry. In the 21st century, however, we must cooperate for the survival of humanity
within the limitations of our planet earth. Every nation has a different history and sometimes
our interests clash. But no country can exist on its own. Everything begins with
understanding between individuals. Friendships across the seas and firm relationships of
trusts are the cornerstones of international security.
You young people here today will be the next generation of leaders. It is my hope that
learning will be a process of joy for you and that you will develop a firm grounding in
whatever specialty you may undertake in the future. But this alone is not enough. You must
also acquire a thorough understanding of society’s conventions and ethics, and the skills to
work with other people. Society needs all kinds of people. We all have different talents and
values. What society needs are people who are highly motivated and have diverse skills. I
hope to see you mature into the kind of person who can make significant contribution to
society.
Now that the Olympiad is over you will be departing Japan for your various home countries
and regions. Please build on the networks of friendship and science that you have formed
here. I hope that you will never forget this time in Tokyo and that the experience will be a
stepping stone to your future and to the world.
Finally, I would like to close by expressing my appreciation to everyone who worked so hard
to make this event possible.
Thank you.
Ryoji Noyori, IChO Chair
July 27, 2010
1
Official report “Report of the 42nd IChO” Contents
Closing Ceremony remarks
1
Contents
2
Overview of the 42
nd
IChO
3
Hosts and Sponsors
3
Venues, Participants, and Results
Participating Countries
5
6
Programs
Students
7
Mentors
8
Guests
9
Tasks
10
Translation Languages
Practical Problems
Theoretical Problems
10
11
38
Final Results and Ranking
70
Statistical Analysis of the Problems
77
Minutes of the International Jury and Steering Committee Meetings
85
Regulations of the International Chemistry Olympiad (IChO)
89
List of Participants
107
Head mentors, Mentors, Observers, and Guests
Students
107
114
Country Participation Fees
121
Budget of the 42nd IChO
122
List of Organizers
123
2
Overview of the 42nd IChO
42nd International Chemistry Olympiad
19-28 July 2010, Japan
Hosts
IChO Japan Committee
Waseda University
The University of Tokyo
The Association for the Progress of New Chemistry
Catalysis Society of Japan
The Ceramic Society of Japan
The Chemical Society of Japan
The Electrochemical Society of Japan
Japan Chemical Industry Association
The Japan Institute of Energy
Japan Oil Chemists' Society
The Japan Petroleum Institute
Japan Science Foundation
The Japan Society for Analytical Chemistry
Japan Society for Bioscience, Biotechnology, and Agrochemistry
The Pharmaceutical Society of Japan
The Society of Chemical Engineers, Japan
The Society of Polymer Science, Japan
The Society of Synthetic Organic Chemistry, Japan
Ministry of Education, Culture, Sports, Science and Technology
Ministry of Economy, Trade and Industry
Science Council of Japan
Japan Science and Technology Agency
Japan Chemical Innovation Institute
Japan Society of Physics and Chemistry Education
Zenkoku Tyugakkou Rikakyouiku Kenkyukai
Japan Broadcasting Corporation
The Asahi Shimbun
The Chemical Daily
The Chunichi Shimbun
The Mainichi Newspapers
Nikkei Inc.
Sankei Shimbun Co.
The Science News Ltd
The Yomiuri Shimbun
Sponsors
Adeka Corporation
Asahi Glass Co., Ltd.
Astomos Energy Corporation
Brighestone Corporation
Central Glass Co., Ltd.
Daicel Chemical Industries, Ltd.
Daikin Industries, Ltd.
Dainippon Tosho Publishing Co., Ltd.
Denki Kagaku Kogyo Kabushiki Kaisha
Dow Corning Toray Co., Ltd.
Du Pont-Mitsui Polychemicals Co., Ltd
Ebara Corporation
Fujifilm Corporation
Fukuvi Chemical Industry Co., Ltd.
Gun Ei Chemical Industry Co., Ltd.
Hitachi Chemical Co., Ltd.
Hodogaya Chemical Co., Ltd.
Idemitsu Kosan Co., Ltd.
Japan Energy Corporation
Japan Steel Drum Association
Japan Vilene Company, Ltd.
JSR Corporation
Kaneka Corporation
Kinden Corporation
Kodansha Ltd.
Konishi Co., Ltd.
Kureha Corporation
Air Water Inc.
Asahikasei Corporation
Bando Chemical Industries, Ltd.
Casio Computer Co., Ltd.
Cosmo Oil Co., Ltd.
Daicel-Evonik Ltd.
Dainichiseika Color & Chemicals Mfg. Co., Ltd
Daiso Co., Ltd.
DIC Corporation
Du Pont-Mitsui Fluorochemicals Company, Ltd.
Du Pont-Toray Co., Ltd.
Exxon Mobil Corporation
Fujitsu Limited
Fuso Chemical Co., Ltd.
Harima Chemicals, Inc.
Hitachi, Ltd.
Honshu Chemical Industry Co., Ltd.
Inabata & Co., Ltd.
Japan Oil Transportation
Japan U-Pica Company Ltd.
JSP Corporation
Kagaku-Dojin Publishing Co., Inc.
Kao Corporation
Kobe Steel, Ltd.
Koei Chemical Co., Ltd.
Kuraray Co., Ltd.
Kygnus Sekiyu K.K.
3
Kyokutou Kai
Kyowa Chemical Industry Co., Ltd.
Kyushu Oil Co., Ltd.
Lanxess K.K.
Lion Corporation
Marubeni Corporation
Maruishi Chemical Trading Co., Ltd.
Maruzen Co., Ltd.
Maruzen Petrochemical Co., Ltd.
Matsumoto Yushi-Seiyaku Co., Ltd.
Meiwa Industry Co., Ltd.
Meiwa Plastic Industries, Ltd.
Microsoft Corporation
Mitsubishi Chemical Corporation
Mitsubishi Corporation
Mitsubishi Engineering-Plastics Corporation
Mitsubishi Gas Chemical Company, Inc.
Mitsubishi Heavy Industries, Ltd.
Mitsubishi Materials Corporation
Mitsubishi Materials Electronic Chemicals Co., Ltd
Mitsubishi Plastics, Inc.
Mitsubishi Rayon Co., Ltd.
Mitsui & Co., Ltd.
Mitsui Chemicals, Inc
Mitsui Engineering & Shipbuilding Co., Ltd.
Mitsui Mining & Smelting Co., Ltd.
Mitsui-Soko Co., Ltd.
Mitsui Sumitomo Insurance Co., Ltd.
Nagoya Asahi Kai
Nanbu Plastics Co., Ltd.
NEC Corporation
Nihon Medi-Physics Co., Ltd
Nihon Millipore K.K.
Nihon Oxirane Co., Ltd.
Nippon Kasei Chemical Company Limited
Nippon Kayaku Co., Ltd.
Nippon Oil Corporation
Nippon Paint Co., Ltd.
Nippon Paper Industries Co., Ltd.
Nippon Polyurethane Industry Co., Ltd.
Nippon Sheet Glass Co., Ltd.
Nippon Shokubai Co., Ltd.
Nippon Soda Co., Ltd.
Nissan Chemical Industries, Ltd.
Nitto Denko Corporation
NOF Corporation
Oji paper Co., Ltd.
Organo Corporation
Osaka Kyokusou Kai
Osaka Organic Chemical Industry Ltd.
Panasonic Corporation
Sanki
Sankyu Inc.
Sanyo Chemical Industries, Ltd.
Sekisui Chemical Co., Ltd.
Sekisui Jushi Corporation
Sekisui Plastics Co., Ltd.
Shin-Etsu Chemical Co., Ltd.
Shinto Paint Co., Ltd.
Shiseido Co., Ltd.
Shoko Co., Ltd
Showa Denko K.K
Showa Engineering Co., Ltd.
Showa Highpolymer Co., Ltd.
Showa Paxxs Corporation
Showa Tansan Co., Ltd.
Soda Aromatic Co., Ltd.
Sumika Chemical Analysis Service, Ltd.
Sumika Color Co., Ltd.
Sumitomo Bakelite Co., Ltd.
Sumitomo Chemical Co., Ltd.
Sumitomo Chemical Engineering Co., Ltd.
Sumitomo Corporation
Sumitomo Seika Chemicals Company Limited
Sunallomer Ltd.
Taiheiyo Cement Corporation
Taiyo Nippon Sanso Corporation
Taiyo Oil Company, Limited
Taiyo Vinyl Corporation
Takasago International Corporation
Taoka Chemical Co., Ltd.
Teijin Limited
The Japan Steel Works, Ltd.
The Japan Wool Textile Co., Ltd.
The Nippon Synthetic Chemical Industry Co., Ltd. Toagosei Co., Ltd.
Toho Chemical Industry Co., Ltd.
Tokuyama Corporation
Tokyo Chemical Industry Co., Ltd.
Tokyo Electric Power Company, Inc.
Tokyo Gas Co., Ltd.
Tokyo Ohka Kogyo Co., Ltd.
Tokyo Printing Ink Mfg. Co., Ltd.
TOLI Corporation
Toray Fine Chemicals Co., Ltd.
Toray Industries, Inc.
Toray Research Center, Inc.
Toshiba Corporation
Toshiba Mitsubishi-Electric Industrial Systems Corporation
Tosoh Corporation
Toyo Engineering Corporation
Toyo Ink Mfg. Co., Ltd.
Toyoda Gosei Co., Ltd.
Toyota Motor Corporation
Ube Industries, Ltd.
Ube Material Industries, Ltd.
Ube-Mitsubishi Cement Corporation
Ube-Nitto Kasei Co., Ltd.
UMG ABS, Ltd.
Unitika Ltd.
Yamatake Corporation
Yokogawa Electric Corporation
ZKAI Co., Ltd.
4
Venues
Practical Exam, Closing Ceremony: Waseda University
Theoretical Exam: The University of Tokyo
Students Accommodations, Opening Ceremony:
National Olympics Memorial Youth Centre (NYC)
Mentors Accommodations:
Overseas Vocational Training Association (OVTA)
Participants
Countries:
68 Participating Countries
3 Observing Countries:
Liechtenstein – 2nd year
Nigeria – 1st year
Serbia – 1st year
Unable to send students but sent observers:
Saudi Arabia
Invited but did not participate:
Egypt
Observation approved but no registration nor delegation arrived:
Uzbekistan
Number of participants:
Students:
Head Mentors, Mentors:
Observers:
Guests:
267
133
65
37
Results
Gold Medalists:
Silver Medalists:
Bronze Medalists:
Honorable Mentions:
IUPAC Awardees:
32
58
86
9
2
5
Participating Countries
Country
Code
Country
Code
1
Argentina
ARG
35
Kuwait
KWT
2
Armenia
ARM
36
Kyrgyzstan
KGZ
3
Australia
AUS
37
Latvia
LVA
4
Austria
AUT
38
Lithuania
LTU
5
Azerbaijan
AZE
39
Malaysia
MYS
6
Belarus
BLR
40
Mexico
MEX
7
Belgium
BEL
41
Moldova
MDA
8
Brazil
BRA
42
Mongolia
MNG
9
Bulgaria
BGR
43
Netherlands
NLD
10
Canada
CAN
44
New Zealand
NZL
11
China
CHN
45
Norway
NOR
12
Chinese Taipei
TPE
46
Pakistan
PAK
13
Costa Rica
CRI
47
Peru
PER
14
Croatia
HRV
48
Poland
POL
15
Cuba
CUB
49
Portugal
PRT
16
Cyprus
CYP
50
Romania
ROU
17
Czech Republic
CZE
51
Russian Federation
RUS
18
Denmark
DNK
52
Singapore
SGP
19
Estonia
EST
53
Slovakia
SVK
20
Finland
FIN
54
Slovenia
SVN
21
France
FRA
55
Spain
ESP
22
Germany
DEU
56
Sweden
SWE
23
Greece
GRC
57
Switzerland
CHE
24
Hungary
HUN
58
Syria
SYR
25
Iceland
ISL
59
Tajikistan
TJK
26
India
IND
60
Thailand
THA
27
Indonesia
IDN
61
Turkey
TUR
28
Iran, I. R. of
IRN
62
Turkmenistan
TKM
29
Ireland
IRL
63
Ukraine
UKR
30
Israel
ISR
64
United Kingdom
GBR
31
Italy
ITA
65
United States
USA
32
Japan
JPN
66
Uruguay
URY
33
Kazakhstan
KAZ
67
Venezuela
VEN
34
Korea
KOR
68
Viet Nam
VNM
Egypt
EGY
Saudi Arabia
SAU
Liechtenstein
LIE
Nigeria
NGA
Serbia
SRB
Uzbekistan
UZB
6
7
14:00
14:30
15:00
15:30
16:00
16:30
17:00
17:30
18:00
18:30
19:00
19:30
20:00
20:30
21:00
21:30
22:00
22:30
23:00
7:00
7:30
8:00
8:30
9:00
9:30
10:00
10:30
11:00
11:30
12:00
12:30
13:00
13:30
Time
Dinner
at NYC
Move to
NYC
Registration
Lunch at
OVTA
Registration
Transfers
Arrivals
Monday
19-Jul
Dinner
at NYC
(Asakusa)
(Tokyo
Tower)
Tokyo
Welcome
Lunch(NYC)
Transfer
Opening
Ceremony
(NYC)
Move
Breakfast
Tuesday
20-Jul
Program: Students
(Dinner)
(Great
Budha)
(Lunch)
(Tsurugaoka
Shrine)
Excursion
to Kamakura
Breakfast
Wednesday
21-Jul
Transfer
Dinner
(Waseda
Univ.)
Practical
Exam
(Waseda
Univ.)
Lunch
Lab safety
instruction
Transfer
Breakfast
Thursday
22-Jul
Dinner
at NYC
(National
Science
Museum)
Sightseeing
Tokyo2
Lunch
Japanese
Culture
Experience
(Gajo-en)
Breakfast
Friday
23-Jul
Transfer
Reunion
Party
(Yokohama)
Strolling
Yokohama
Bay Area
Transfer
Lunch
and
Social Event
(Univ. of
Tokyo)
Theoretical
Exam
(Univ. of
Tokyo)
Transfer
Breakfast
Saturday
24-Jul
Drum
Performance
(Kijima Taiko)
Dinner
at NYC
Sports and
Traditional
Arts
(NYC)
Transfer
(Lunch)
Judo watching
and Experience
(Kodokan)
Transfer
(Lunch)
Sports and
Traditional
Arts
(NYC)
Breakfast
Sunday
25-Jul
(Dinner)
(Edomura:
Experience of
the Life in the
Age of Bushi)
(Lunch)
(Tosho-gu
Shrine)
Excursion
to Nikko
Breakfast
Monday
26-Jul
Transfer
Farewell
party
(Rihga
Royal Hotel
Tokyo)
Closing
Ceremony
(Okuma
Auditorium,
Waseda
Univ.)
(Lunch)
Free Time
in Tokyo
Breakfast
Tuesday
27-Jul
Departures
Breakfast
Wednesday
28-Jul
8
14:00
14:30
15:00
15:30
16:00
16:30
17:00
17:30
18:00
18:30
19:00
19:30
20:00
20:30
21:00
21:30
22:00
22:30
23:00
7:00
7:30
8:00
8:30
9:00
9:30
10:00
10:30
11:00
11:30
12:00
12:30
13:00
13:30
Time
Dinner
Registration
Lunch at
OVTA
Registration
Transfers
Arrivals
Monday
19-Jul
1st Jury
Meeting
re: Practical
Exam
Dinner
Meet with
Authors
Transfer
Lab Inspection
(Waseda univ.)
Transfer to
OVTA
Welcome
Lunch (NYC)
Transfer
Opening
Ceremony
(NYC)
Transfer to
NYC
Breakfast
Tuesday
20-Jul
Program: Mentors
Dinner
(20:00limit)
Lunch
Translation
Breakfast
Wednesday
21-Jul
2nd Jury
Meeting
re:
Theoretical
Exam
Dinner
Meet with
Authors
(Asakusa)
(Lunch)
(Imperial
Palace
Square)
Sightseeing
Tokyo
Breakfast
Thursday
22-Jul
Dinner
(20:00limit)
Lunch
Translation
Breakfast
Friday
23-Jul
Transfer to
OVTA
Reunion
Party
(Yokohama)
Strolling
Yokohama
Bay Area
(Great
Budha)
(Lunch)
(Tsurugaoka
Shrine)
Excursion
to Kamakura
Breakfast
Saturday
24-Jul
3rd Jury
Meeting
Business
(Dinner)
(Boso-no-mura
Museum:
Experience of
Traditional
Villege Life)
(Shinsho-ji
Temple, Narita)
(Lunch)
Excursion to
the Rural Area
of Chiba
Marking Task
Breakfast
Sunday
25-Jul
4th Jury
Meeting
Dinner
Lunch
Arbitration
Breakfast
Monday
26-Jul
(Night Time)
Transfer to
OVTA
Farewell party
(Rihga Royal
Hotel Tokyo)
Closing
Ceremony
(Okuma
Auditorium
Waseda Univ.)
(Lunch)
Free Time
(Transfer
to Waseda)
Breakfast
Tuesday
27-Jul
Departures
Breakfast
Wednesday
28-Jul
9
7:00
7:30
8:00
8:30
9:00
9:30
10:00
10:30
11:00
11:30
12:00
12:30
13:00
13:30
14:00
14:30
15:00
15:30
16:00
16:30
17:00
17:30
18:00
18:30
19:00
19:30
20:00
20:30
21:00
21:30
22:00
22:30
23:00
Time
Dinner
Registration
Lunch
at OVTA
Registration
Transfers
Arrivals
Monday
19-Jul
Dinner
(OVTA)
(Korakuen
Garden)
(Tokyo
Tower)
Tokyo 1
Transfer
Welcome
Lunch
(NYC)
Opening
Ceremony
(NYC)
Transfer
to
NYC
Breakfast
Tuesday
20-Jul
Program: Guests
Dinner
(OVTA)
(Oshino
Hakkai
Forest)
Dinner
(OVTA)
Dinner
(OVTA)
Transfer to
OVTA
Reunion
Party
(Yokohama)
Strolling
Yokohama
Bay Area
(Great
Budha)
(Visiting
the old
City Area)
(Asakusa)
(Kawaguchi
Lake)
(Sumida
River
Cruising)
(Lunch)
(Tsurugaoka
Shrine)
Excursion
to Kamakura
Breakfast
Saturday
24-Jul
(Lunch)
Excursion
to
Kawagoe
Breakfast
Friday
23-Jul
(Lunch)
(Imperial
Palace
Square)
Sightseeing
Tokyo 2
Breakfast
Thursday
22-Jul
(Lunch)
Excursion
to the base
of Mt. Fuji
Breakfast
Wednesday
21-Jul
(Dinner)
(Boso-no-mura
Museum:
Experience of
Traditional
Villege Life)
(Shinsho-ji
Temple, Narita)
(Lunch)
Excursion to
the Rural Area
of Chiba
Breakfast
Sunday
25-Jul
Dinner
(OVTA)
Souvenir
Hunting
(Schuttle to
Lalaport
Shopping
Complex)
Breakfast
Monday
26-Jul
(Night Time)
Transfer to
OVTA
Farewell
party
(Rihga Royal
Hotel Tokyo)
Closing
Ceremony
(Okuma
Auditorium,
Waseda
Univ.)
(Lunch)
Free Time
(Transfer
to Waseda)
Breakfast
Tuesday
27-Jul
Departures
Breakfast
Wednesday
28-Jul
Translation Languages
Country
Languages
Country
Languages
1
Argentina
Spanish
35
Kuwait
Arabic
2
Armenia
Armenian
36
Kyrgyzstan
Russian
3
Australia
English
37
Latvia
Latvian, Russian
4
Austria
German
38
Lithuania
Lithuanian
5
Azerbaijan
English, Russian
39
Malaysia
English
6
Belarus
Russian
40
Mexico
Spanish
7
Belgium
Dutch, French
41
Moldova
Russian
8
Brazil
Portuguese
42
Mongolia
Mongolian
9
Bulgaria
Bulgarian
43
Netherlands
Dutch
10
Canada
English
44
New Zealand
English
11
China
Chinese
45
Norway
Norwegian
12
Chinese Taipei
Traditional Chinese
46
Pakistan
English
13
Costa Rica
Spanish
47
Peru
Spanish
14
Croatia
Croatian
48
Poland
Polish
15
Cuba
Spanish
49
Portugal
Portuguese
16
Cyprus
Greek
50
Romania
Romanian
17
Czech Republic
Czech
51
Russian Federation
Russian
18
Denmark
Danish
52
Singapore
English
19
Estonia
Estonian, Russian
53
Slovakia
Slovak
20
Finland
Finnish
54
Slovenia
Slovenian
21
France
French
55
Spain
Spanish
22
Germany
German
56
Sweden
Swedish
23
Greece
Greek
57
Switzerland
French, German
24
Hungary
Hungarian
58
Syria
Arabic
25
Iceland
Icelandic
59
Tajikistan
English, Russian
26
India
English
60
Thailand
Thai
27
Indonesia
Indonesian
61
Turkey
Turkish
28
Iran, I. R. of
Farsi
62
Turkmenistan
English, Russian
29
Ireland
English
63
Ukraine
Russian
30
Israel
Hebrew
64
United Kingdom
English
31
Italy
Italian
65
United States
English
32
Japan
Japanese
66
Uruguay
Spanish
33
Kazakhstan
Russian
67
Venezuela
Spanish
34
Korea
Korean
68
Viet Nam
Vietnamese
10
Instructions
Examination Procedures
• You have 5 hours to complete Tasks 1, 2, and 3.
You may perform the tasks in any
order you choose.
• There will be an additional 15 minutes reading time before the start.
• DO NOT begin working on the tasks until the START command is given.
• When the STOP command is given at the end of the 5 hours, you must stop your
work on the tasks immediately. A delay in doing so may lead to your
disqualification from the examination.
• After the STOP command has been given, wait in your lab space.
will check your lab space.
A supervisor
The following items should be left behind:
9
The problem booklet (this booklet)
9
The answer booklet
9
Your chosen TLC plates in zipper storage bags A and B with your student
code (from Task 1)
9
Your product and glass microfiber filter sheet in a crystallization dish with a
lid in zipper storage bag C with your student code (from Task 1)
• Do not leave the examination hall until you are instructed to do so by the supervisors.
Safety
• Safety is the most important issue in the laboratory. You are expected to follow the
safety rules given in the IChO regulations. Safety glasses and lab coats must be
worn at ALL TIMES.
• If you behave in an unsafe manner, you will receive one warning before you are
asked to leave the laboratory. If required to leave due to a second warning, you will
receive a score of zero for the entire experimental examination.
• NO eating or drinking is allowed in the laboratory.
• In case of emergency, follow the instructions given by the supervisors.
Notes on the booklets and answer methods
• The problem booklet comprises 23 pages including cover page.
• The answer booklet comprises 6 pages.
11
Do not attempt to separate the sheets.
• You should confirm your student code inscribed on the booklets and write your
name and student code on every answer sheet.
• Use only the pen provided for filling in the answer sheets.
You may also use the
calculator and the ruler provided. Use the mechanical pencil provided only for
experiments in Task 1. Do not use the mechanical pencil for filling in the answer
sheets.
• All results must be written in the appropriate areas on the answer sheets. Results
written elsewhere will not be graded.
If you need to do rough calculations, etc.,
use the back of the sheets.
• You should take care to report answers to an appropriate number of significant
figures.
• Keep your answer booklet in the envelope provided.
Take out the booklet only when
you write the answers. Do not seal the envelope.
Notes on the Examination
• You may need to reuse some glassware during the examination.
If this is the case,
clean it carefully in the sink closest to you.
• Contact a supervisor near you if you have any questions regarding the tasks or if you
need a refreshment/toilet break.
• Use the labeled waste containers under the hood or near the windows for disposal of
liquids and solids. A waste container (plastic beaker) is also available on each
bench for aqueous waste. Discard used glass capillaries into a labeled plastic tube .
• Replacement of chemicals and laboratory ware will be provided if necessary.
Other than the first, for which you will be pardoned, each such incident will result in
the loss of 1 point from your 40 practical points.
Refilling of washbottle water is
permitted with no loss of points.
• An official English version of this examination is available upon request if you require
clarification.
12
Periodic table with relative atomic masses
1
2
3
4
5
6
7
8
9
10
11
12
13
14
15
16
17
1
18
2
H
He
1.01
4.00
3
4
5
6
7
8
9
10
Li
Be
B
C
N
O
F
Ne
6.94
9.01
10.81
12.01
14.01
16.00
19.00
20.18
11
12
22.99
24.30
13
Na Mg
19
K
Ca
Sc
Ti
22
23
V
Cr
Mn
Fe
Co
Ni
Cu
Zn
39.10
40.08
44.96
47.87
50.94
52.00
54.94
55.85
58.93
58.69
63.55
65.38
41
42
92.91
95.96
37
20
21
Rb
Sr
38
39
Y
Zr
85.47
87.62
88.91
91.22
57-71
Hf
55
Cs
56
Ba
72
132.91 137.33
87
Ra
-
-
Nb Mo
73
Ta
74
W
25
43
Tc
75
Re
26
44
Ru
27
28
45
46
Rh
Pd
29
47
Ag
30
48
Cd
Si
14
15
16
S
Cl
Ar
26.98
28.09
30.97
32.06
35.45
39.95
31
32
69.72
72.64
Ga Ge
49
In
50
Sn
P
33
104
89-103
105
106
107
76
Os
77
78
Ir
Pt
79
Au
108
109
110
Sg
Bh
Hs
Mt
Ds
Rg
-
-
-
-
-
-
-
-
60
61
62
63
58
Ce
59
Pr
Nd Pm Sm Eu
138.91 140.12 140.91 144.24
89
Ac
-
90
Th
91
Pa
92
U
232.04 231.04 238.03
36
Se
Br
Kr
74.92
78.96
79.90
83.80
52
53
51
Sb
Te
I
54
Xe
80
Hg
81
Tl
82
Pb
83
Bi
93
Np
-
84
85
86
Po
At
Rn
-
-
-
69
70
111
Db
57
35
18
101.07 102.91 106.42 107.87 112.41 114.82 118.71 121.76 127.60 126.90 131.29
Rf
La
34
17
As
178.49 180.95 183.84 186.21 190.23 192.22 195.08 196.97 200.59 204.38 207.2 208.98
88
Fr
40
24
Al
64
Gd
65
Tb
66
Dy
67
Ho
68
Er
Tm Yb
71
Lu
150.36 151.96 157.25 158.93 162.50 164.93 167.26 168.93 173.05 174.97
94
95
96
97
-
-
-
-
Pu Am Cm Bk
13
98
Cf
-
99
100
101
102
103
-
-
-
-
-
Es Fm Md No
Lr
Apparatuses
Apparatuses
Number
For multiple tasks (on the desk or in Box 1):
20-mL beaker for taking a small portion of liquid to rinse inside of
glassware
1
Paper
3
2-mL pipette bulb
1
5-mL pipette bulb
1
Pipette rack
1
200-mL plastic beaker for waste
1
Safety bulb
1
Spatula
1
Stand
1
100-mL washbottle
1
500-mL washbottle
1
For Task 1 (in Box 1, on the desk or on pipette rack):
Büchner funnel with rubber adapter
1
Clamp with muff (clamp holder)
1
200-mL conical beaker
1
300-mL conical beaker
1
Diaphragm vacuum pump with tubing and connecter
1
Glass capillary tube (in a plastic tube)
8
Glass microfiber filter sheet in a crystallization dish with lid
1
2-mL graduated pipette
3
5-mL graduated pipette
1
Magnetic stirrer
1
10-mm magnetic stirring bar
1
22-mm magnetic stirring bar
1
10-mL measuring glass
1
pH test paper (in a zipper storage bag)
3
10-mL plastic graduated cylinder
1
Plastic tube for used glass capillary
1
Suction flask
1
10-mL test tube
1
14
100-mL test tube
1
TLC developing chamber with lid
1
TLC plate (in a zipper storage bag)
4
Tweezers
1
Zipper storage bags A and B for submission of TLC plates
Zipper storage bag C for submission of glass microfiber filter sheet in a
crystallization dish
1 for each
1
For Task 2 (in Box 2, on the desk or on pipette rack):
2-mL graduated pipette
1
5-mL graduated pipette
1
Label (in a zipper storage bag)
4
LED light box (in a zipper storage bag: do not remove from the bag at any
time.)
1
Nessler tube
5
Nessler tube rack
1
50-mL volumetric flask
2
5-mL volumetric pipette
1
10-mL volumetric pipette
1
For Task 3.1 (in Box 2 or on pipette rack):
Burette
1
Burette clamp
1
100-mL conical beaker
6
Glass funnel (for transferring chemicals to a burette)
1
1-mL graduated pipette
2
5-mL volumetric pipette
1
20-mL volumetric pipette
1
For Task 3.2 (in Box 2):
10-mL vial (in a zipper storage bag)
10
Plastic Pasteur pipette
1
Shared equipment:
Gloves of various sizes
UV lamp
Cleaning tissue
15
Chemicals on Each Desk
Chemical
Quantity
Container
R phrases
S phrases
Plastic bottle None listed
None listed
Vial
36/37/38
26
Vial
36/37/38
26
For multiple tasks (in Box 1):
0.5 mol L-1 hydrochloric acid
(0.5 mol L-1 HCl)
50 mL
For Task 1 (in Box 1):
1,4-dihydro-2,6-dimethylpyridine-3,5
1g
-dicarboxylic acid diethyl ester
(C13H19NO4; 1,4-DHP_powder)
1,4-DHP for TLC
3 mg
(1,4-DHP_TLC)
Ethanol (C2H5OH)
10 mL
Glass bottle
11
7-16
Ethyl acetate (CH3COOC2H5)
25 mL
Glass bottle
11-36-66-67
16-26-33
Heptane (C7H16)
20 mL
Glass bottle
11-38-50/53-6567
9-16-29-3360-61-62
Potassium iodide (KI)
150 mg
Glass bottle
None listed
None listed
Sodium metabisulfite (Na2S2O5)
1g
Glass bottle
22-31-41
26-39-46
Saturated sodium
hydrogencarbonate solution
(Sat. NaHCO3 solution)
25 mL
Glass bottle
None listed
None listed
1g
Vial
8-34
17-2636/37/39-45
30 mL
Plastic bottle None listed
None listed
Plastic bottle None listed
None listed
Plastic bottle None listed
None listed
Plastic bottle None listed
None listed
Urea hydrogen peroxide
(CH4N2O•H2O2; UHP)
For Task 2 (in Box 2):
Sample solution (labeled as
“Sample solution”)
Standard Fe(bpy)32+ solution 1
(containing 2.0 mg of iron in 1 L
50 mL
solution) (labeled as “Standard
Fe(bpy)32+ solution 1”)
Standard Fe(bpy)32+ solution 2
(containing 3.0 mg of iron in 1 L
50 mL
solution) (labeled as “Standard
Fe(bpy)32+ solution 2”)
Acetate buffer solution
(pH 4.6, 1:1 mixture of acetic acid
50 mL
and sodium acetate;
CH3COOH-CH3COONa solution)
16
0.1 mol L-1 disodium
25 mL
hydrogenphosphate solution
(0.1 mol L-1 Na2HPO4)
0.2 %(w/v) 2,2’-bipyridine aqueous
25 mL
solution
(0.2 %(w/v) C10N2H8)
Sodium thioglycolate
(C2H3NaO2S)
20 mg
Plastic bottle None listed
None listed
Plastic bottle None listed
None listed
Vial
36
22-38
For Task 3.1 (in Box 2 or on the desk):
Polysaccharide solution (labeled as
50 mL
“Polysaccharide solution”)
Plastic bottle None listed
None listed
Poly(diallyldimethylammonium
chloride) aqueous solution
(PDAC)
240 mL
Glass bottle
None listed
None listed
240 mL
Glass bottle
36/37/38
26-36
0.5 mol L-1 sodium hydroxide
aqueous solution
(0.5 mol L-1 NaOH)
50 mL
Plastic bottle 34
26-37/39-45
1 g L-1 toluidine blue (TB) aqueous
solution
(1 g L-1 C15H16N3SCl)
6 mL
Dropper
bottle
None listed
CH2
H3C
N+
Cl-
CH2
CH3
n
Potassium poly(vinyl sulfate)
aqueous solution (0.0025 mol L-1;
monomer unit concentration)
(0.0025 mol L-1 PVSK)
CH2 CH
O
O S O
O-K+ n
17
None listed
For Task 3.2 (in Box 2):
Solution X-1 (X: A-H)
10 mL
Dropper
bottle
Solution X-2 (X: A-H)
10 mL
Dropper
bottle
Solution X-3 (X: A-H)
10 mL
Dropper
bottle
Solution X-4 (X: A-H)
10 mL
Dropper
bottle
Solution X-5 (X: A-H)
10 mL
Dropper
bottle
18
36/37/38
26-36
Risk Phrases
Number
Special Risks
8
Contact with combustible material may cause fire.
11
Highly flammable
22
Harmful if swallowed
31
Contact with acids liberates toxic gas.
34
Causes burns
36
Irritating to eyes
38
Irritating to skin
41
Risk of serious damage to eyes
65
Harmful: may cause lung damage if swallowed.
66
Repeated exposure may cause skin dryness or cracking.
67
Vapors may cause drowsiness and dizziness.
36/37/38
Irritating to eyes, respiratory system and skin
50/53
Very toxic to aquatic organisms, may cause long term adverse effects in the
aquatic environment.
19
Safety Phrases
Number
Safety Recommendations
7
Keep container tightly closed.
9
Keep container in a well ventilated place.
16
Keep away from sources of ignition - No Smoking.
17
Keep away from combustible material.
26
In case of contact with eyes, rinse immediately with plenty of water and seek
medical advice.
29
Do not empty into drains.
33
Take precautionary measures against static discharges.
36
Wear suitable protective clothing.
37
Wear suitable gloves.
39
Wear eye/face protection.
45
In case of accident or if you feel unwell, seek medical advice immediately. (Show
the label where possible.)
46
If swallowed, seek medical advice immediately and show the container or label.
60
This material and its container must be disposed of as hazardous waste.
Avoid release to the environment. Refer to special instructions/ material safety data
sheet.
If swallowed, do not induce vomiting: seek medical advice immediately and show
the container or label
61
62
24/25
Avoid contact with skin and eyes.
36/37/39
Wear suitable protective clothing, gloves and eye/face protection.
37/39
Wear suitable gloves and eye/face protection
20
Task 1
1a
1b
1c
1d
4
4
2
2
1e
i
ii
iii
2
2
24
Total
40
Reaction of Hantzsch Ester with Urea Hydrogen Peroxide
In this experiment, you are required to synthesize a pyridinedicarboxylate derivative
from 1,4-dihydro-2,6-dimethylpyridine-3,5-dicarboxylic acid diethyl ester (1,4-DHP or
Hantzsch
ester)
by
oxidation
with
urea
hydrogen
peroxide
(UHP),
an
environmentally-friendly oxidant.
O
H
CH3CH2
O
O
O
H3C
O
N
H
CH2CH3
N
H
UHP
CH3
N
H
H
O
O O
H
H CH3CH2
O
O
O
H3C
KI
N
CH2CH3
CH3
1,4-DHP
Procedures
(1) Place a 22-mm magnetic stirring bar in a 100-mL test tube. Fix the test tube on a
magnetic stirrer using a clamp.
Add 1,4-DHP (1 g) (labeled as 1,4-DHP_powder), and
potassium iodide (150 mg) to the test tube, followed by ethanol (5 mL), with a 5-mL
graduated pipette.
(2) Add 1 g UHP (wear gloves) and stir the mixture.
(Caution: this reaction is
exothermic.)
(3) For thin layer chromatography (TLC) analysis, prepare a mixture of ethyl
acetate:heptane (1:2 in volume) with a measuring glass and place an appropriate
amount of the mixture in a TLC developing chamber. Add 1 mL of ethyl acetate to the
vial (labeled as 1,4-DHP_TLC) to dissolve 1,4-DHP (3 mg).
(4) Check your TLC plates before using.
without penalty.
If they are damaged, they can be replaced
Draw a start line on the lower portion of a TLC plate with a pencil (see
Fig. 1.1).
(5) During the reaction, the reaction mixture becomes clear (usually within 20 min).
When
the reaction mixture becomes clear (the precipitates may form when it cools, but
precipitates will not affect the TLC analysis), take a small portion of the mixture using a
21
glass capillary and load it to make two spots in the center and right positions on the TLC
plate. Load an appropriate amount of the 1,4-DHP solution prepared in procedure (3)
in the center and left positions, so that there are three spots on the plate, with the center
spot containing both the reaction mixture and 1,4-DHP (see Fig. 1.1).
Develop the
TLC plate in the TLC chamber (see Figs. 1.1 and 1.2). Mark the solvent front with the
pencil. Visualize the spots using a UV lamp (254 nm) and draw a line around the
UV-active spots on the TLC clearly with the pencil.
Assess the completion of the
reaction based on the TLC results. Repeat the TLC analysis after ten minutes, if you
find significant amounts of 1,4-DHP in the reaction mixture. [Note that you will perform
TLC analysis again in procedure (8).]
Place the last TLC plate in a zipper storage bag
marked “A.”
X X+Y Y
Fig. 1.1 Spots on the TLC plate before
development;
X: 1,4-DHP, Y: Reaction mixture.
Fig. 1.2
TLC plate placed in the
TLC developing
chamber.
(6) Set up the suction filtration equipment (see Fig. 1.3).
Connect the suction flask to the diaphragm
vacuum pump. Place a Büchner funnel fitted
with a rubber adapter onto the suction flask.
Place a glass microfiber filter sheet on the
funnel.
(7) Add water (5 mL) to the reaction mixture
using a 10-mL plastic graduated cylinder.
Add sodium metabisulfite (1 g), transfer the
contents of the tube (including the stirring bar)
into a 200-mL conical beaker and wash the
test tube with water (30 mL).
Place the
200-mL conical beaker on the magnetic stirrer
22
Fig. 1.3 Suction filtration equipment:
i, Büchner funnel; ii, rubber adopter; iii,
suction flask; iv, diaphragm vacuum
pump.
and stir the solution.
Add saturated sodium hydrogencarbonate solution in small
portions using a 2-mL graduated pipette until the pH of the aqueous phase becomes
just over 7 (check the pH with pH test paper).
Filter the precipitate formed through the
Büchner funnel with suction using the diaphragm vacuum pump, and wash the
precipitate with a small portion of water.
Suck air through the precipitates for a minute
to dry the product.
(8) Transfer the filtrate from the suction flask to a 300-mL conical beaker. Transfer the
filtrate (2 mL) to a 10-mL test tube using a 2-mL graduated pipette. Place a 10-mm
magnetic stirring bar in the test tube and fix it securely with the clamp.
Add 1 mL of
ethyl acetate to the test tube using a 2-mL graduated pipette and stir the solution
vigorously for 30 seconds on the magnetic stirrer.
to separate into two layers.
Stop stirring and wait for the solution
Analyze the upper layer by TLC to see if there is any
product remaining in the filtrates.
Spot the filtrates on the plate in the same way as
procedure (5). Mark the solvent front and the spot(s), if any.
Place the TLC plate in a
zipper storage bag marked “B.” If you detect the product on the TLC plate, add more
saturated sodium hydrogencarbonate solution.
(9) At this stage, if you find a precipitate formed, filter and wash it.
If you find no
precipitate, skip this filtration process.
(10) Suck air through the precipitate for 10 minutes to dry the product. Place your product
and the glass microfiber filter sheet in the crystallization dish. Cover the dish with the
lid marked with your code.
Avoid placing the stirring bar in the dish.
crystallization dish with the lid in a zipper storage bag marked “C.”
a) Copy (sketch) the TLC plate in bag “A” on your answer sheet.
Indicate the solvent front line and the base line.
1) If there are less than three spots loaded on the
base line, 3 points will be subtracted.
2) If the spots are not separated on the TLC after
development, 2 points will be subtracted.
3) If the solvent front line and/or the base line is
missing, 1 point will be subtracted for each.
23
Place the
b) Determine and record the Rf values (to the 2nd decimal place) of the spots on the TLC
plate in bag “A.”
Spot
Rf value
1,4-DHP
0.32-0.42
Product
0.61-0.71
Two points each will be awarded for Rf values (to the 2nd decimal place) in the ranges
shown above. No points will be awarded for values outside the ranges. A score of 1
will be given if the value is reported down to the 1st decimal place.
c) Draw
the
structural
formula
of
the
organic
cation
before
adding
sodium
hydrogencarbonate.
If the correct structural formula is drawn as is shown below, 2 points will be
awarded.
O
CH3CH2
O
O
H3C
O
N
CH2CH3
CH3
H
d) What is (are) the final product(s) derived from UHP?
Give the chemical formula(e) of
the product(s).
If correct chemical formulae are written as shown below, 1 point each will be
awarded.
H2O and CH4N2O
e) Submit the following:
i) TLC plate in bag “A”
ii) TLC plate in bag “B”
iii) Your product and filter paper in the crystallization dish placed in bag “C”
iv) TLC plate in bag “A”
If the outline to be drawn with a pencil around the UV-active spots is unclear or
missing, 1 point will be subtracted.
24
v) TLC plate in bag “B”
1) If the outline to be drawn with a pencil around the UV-active spots is unclear or
missing, 1 point will be subtracted.
2) If the solvent front line and/or the base line is missing, minus 1 point for each will
be subtracted.
vi) Your product and filter paper in the crystallization dish stored in bag “C”
1) The scientific committee will measure the percent yield after drying at 60 °C.
2) In most cases, the sample is pure and dissolved in CDCl3 completely.
The
following calculation based on the percent yields obtained will be applied only if
no 1,4-DHP or byproducts is observed in the 1H NMR spectrum and the product is
completely soluble in CDCl3.
If 80.0 ≤ %yield <92.0, 0-24 points
If 92.0 ≤ %yield < 99.0, 24 points.
If 99.0 ≤ %yield < 102.0, 24-0 points.
3) If there are peaks of 1,4-DHP (ca δ 2.19 ppm) and the corresponding pyridine
product (ca δ 2.85 ppm) in the 1H NMR spectrum and the percent yield is 100% or
less, the actual percent yield is calculated by the following equation:
Sample mass (g)
Theoretical yield (g)
x
(Integral at δ 2.85 ppm) x 251.3
(integral at δ 2.19) x 253.3 + (integral at δ 2.85 ppm) x 251.3
x 100
4) If insoluble material remains after the addition of CDCl3 for 1H NMR measurement,
6 points will be subtracted.
5) If byproducts are detected evidently in the 1H NMR spectrum, 6 points will be
subtracted.
25
Task 2
2a
2
2b
2
2c
15
2d
15
2e
3
2f
Total
i
ii
3
5
45
Determination of Fe(II) and Fe(III) by visual colorimetry
In this experiment, you are required to determine Fe(II) and Fe(III) in a given sample
solution which simulates a dissolved magnetite ore by visual colorimetric analysis involving
a color reaction between Fe(II) and 2,2’-bipyridine (bpy) to form an intensely red complex,
Fe(bpy)32+.
The amount of Fe(bpy)32+ complex can be quantified by visual colorimetric
measurement using Nessler tubes. This is a quite simple technique that was employed
before photoelectric instruments were generally available, but an accuracy of less than
±5% can be achieved.
In this technique, a pair of Nessler tubes is used; one is filled with a
reference solution, and the other is filled with a solution to be tested. The depths of colors
of the two solutions are balanced by adjusting the heights of liquid columns of the solutions.
When the colors look the same, the concentration can be calculated from that of the
reference solution with a known concentration and the height of the column of each solution
based on the Lambert-Beer law:
A = εcl
where A is the absorbance, c is the concentration, l is the pass length and ε is the molar
absorption coefficient.
First, you will learn to employ this technique by conducting
measurements A and B, and then you will determine the concentrations of Fe(II) and
Fe(III) with measurements C and D.
Procedures
(1) Add 5 mL of acetate buffer solution, 5 mL of disodium hydrogenphosphate solution (to
mask Fe(III)), 5 mL of 2,2’-bipyridine solution and 10.00 mL of sample solution into a
50-mL volumetric flask using appropriate pipettes for each and dilute the resulting
solution with water to the 50-mL mark.
Then stopper the flask and mix the solution well.
Allow it to stand for at least 20 min to fully develop color.
This solution is named
“sample 1.”
(2) Add 5 mL of acetate buffer solution, 5 mL of 2,2’-bipyridine solution and 5.00 mL of
sample solution into a 50-mL volumetric flask. Then add 20 mg of sodium thioglycolate
26
powder (in excess) to reduce Fe(III) to Fe(II). Dilute the solution with water to the
50-mL mark, stopper the flask and mix the solution well. Allow it to stand for at least 20
min. This solution is named “sample 2.”
(3) Perform visual colorimetric measurements A – D based on the “Instructions for visual
colorimetric measurement” shown below.
Instructions for visual colorimetric measurement
Set a pair of Nessler tubes on a Nessler tube rack placed on an LED light box (do not
remove it from the bag at any time) and turn on the light (see Fig. 2.1). Pour the
provided “standard Fe(bpy)32+ solution 1” into one tube to an appropriate height (70 –
90 mm is recommended) from the bottom (etched marks on the tube indicate fixed
heights from the bottom in mm) and use this as a reference for measurements A - D.
Pour the solution to be measured into the other tube, and then compare its depth of
color with that of the reference solution by looking downward through the solutions
toward the LED light box.
Adjust the height of the liquid column of the
test solution by adding or removing the solution
with a graduated pipette until the depth of color
in the two tubes is identical.
Estimate your
reading to at least 1 mm.
Note that the depths of color in a certain
range may be recognized as identical to human
eyes.
The appropriate value for the height of
the test solution, h, should be determined by
taking the range into the consideration.
For
example, if you adjust the height of the liquid
column of the test solution only by increasing (or
decreasing) the volume, you could reach a lower
(or higher) value than the true one.
A possible
way to estimate the true value is to take an
Fig. 2.1 Visual colorimetric
measurement: i, Nessler tube; ii,
Nessler tube rack; iii, LED light
box in a zipper storage bag; iv,
power switch.
average between the values of lower and higher
limits.
Measurement A:
Perform a measurement using “standard Fe(bpy)32+ solution 1” as
both the reference and the test solutions.
27
In this measurement, pour the reference
solution into a Nessler tube to achieve an appropriate height, and then pour the test
solution into the other Nessler tube until the colors of the two solutions match each other.
(When the colors match, the heights should IDEALLY be the same.) Then add more test
solution until you recognize that the colors have become different from each other. Report
both the lower and higher limits of the height of the liquid column of test solution with the
same depth of color as the reference solution.
a) Report your results for measurement A using the table provided on the answer sheet.
h' (height of Lower limit of Higher limit of h (estimated
standard
h /mm
h /mm
height of test
solution 1) /
solution)
/
mm
mm
Measurement A
Any value
Any value
Any value
Any value
Two points will be awarded, except when there is no answer.
Measurement B:
Perform a measurement of “standard Fe(bpy)32+ solution 2” as a test
solution using “standard Fe(bpy)32+ solution 1” as a reference.
b) Report your results for measurement B using the table provided on the answer sheet.
h' (height of standard h (estimated height of test
solution 1) / mm
solution) / mm
Measurement B
Any value
Any value
Two points will be awarded, except when there is no answer.
Measurement C:
Perform measurement of sample 1.
c) Report your results for measurement C using the table provided on the answer sheet.
h' (height of standard h (estimated height of test
solution 1) / mm
solution) / mm
sample 1: 1.23 h'
Measurement C
Experimental value of h'
sample 2: 1.16 h'
sample 3: 1.10 h'
28
Measurement D:
Perform measurement of sample 2.
d) Report your results for measurement D using the table provided on the answer sheet.
h' (height of
solution 1) / mm
standard h (estimated height of test
solution) / mm
sample 1: 0.763 h'
Experimental value of h'
Measurement D
sample 2: 0.725 h'
sample 3: 0.749 h'
For 2c and 2d, a full score of 15 points will be awarded for values within a ±5% error range.
A score of zero will be given if the absolute error is 15% or more. A linear point scale will be
applied for scores from zero to 15; points will be
calculated by the following equation:
15
⎡
MV − h − MV ⋅ 0.05 ⎤
⎢
⎥
P = 15 1−
⎢⎣ (MV ⋅ 0.15) − (MV ⋅ 0.05) ⎥⎦
Points
10
h'⋅2.0 (mg L−1 )
MV =
c
5
P : Points (no negative value; zero if P < 0)
0
MV : Master value of h (mm)
-20 -15 -10
-5
0
5
10
15
20
% error
h : Experimental height of liquid column of the
test solution (mm)
h’ : Experimental height of liquid column of reference solution (mm)
c : Concentration of Fe in correctly prepared test solutions (mg L-1)
for 2c, c = 1.63,1.72 and 1.82 for Sample 1, 2 and 3, respectively.
for 2d, c = 2.62, 2.76, and 2.67 for Sample 1, 2 and 3, respectively.
e) Express the concentration of the test solution, c, using the concentration of the
reference solution, c’, and the height of each liquid column, h and h’.
c=
c ' h'
h
3 points. Any equivalent formula is acceptable.
29
f) Calculate the concentrations of Fe(II) and Fe(III) in the original sample solution in mg
L-1.
For Fe2+,
[Fe2+ ] =
2.0(mg L−1 ) × hC' × 50(mL )
hC × 10(mL)
[Fe2+]: concentration of Fe2+ in the sample solution (mg L-1)
hC: experimental height (mm) of the liquid column of the test solution in the
measurement C
h’C : experimental height (mm) of the liquid column of the standard solution in
the measurement C
If the concentrations are calculated correctly from the experimental data, full
marks will be awarded 3 points.
For Fe3+
2.0(mg L−1 ) × hD' × 50(mL )
[Fe ] =
− [Fe2+ ]
hD × 5(mL )
3+
[Fe3+]: concentration of Fe3+ in the sample solution (mg L-1)
hD : experimental height (mm) of the liquid column of the test solution in the
measurement D
h’D : experimental height (mm) of the liquid column of the standard solution in
the measurement D
If the concentrations are calculated correctly from the experimental data, full
marks will be awarded 5 points.
Concentrations of Fe2+ and Fe3+ in each original sample solution
[Fe2+] / mg L-1
[Fe3+] / mg L-1
Sample 1
8.16
18.0
Sample 2
8.60
19.0
Sample 3
9.08
17.7
30
Task 3
3.1a
3.1b
3.1c
3.1d
3.1e
3.1f
3.2
Total
4
10
1
10
1
4
20
50
Polymers in Analysis
Polymers can be used in various analyses. In this task, you are first required to
analyze a polysaccharide using a polymer-polymer interaction, which will then be utilized to
identify polymers in the second part.
3.1 Analysis of Polysaccharide by Colloid Titration
You are provided with a solution of a polysaccharide containing sulfonate (-SO3-) and
carboxylate (-COO-) groups. You are asked to determine the concentrations of these two
groups by colloid titration under the basic and acidic conditions based on the differences in
the protonation behavior of these acid groups. A back-titration technique is utilized.
When these acid groups are ionized, the polysaccharide becomes a polyanion.
Upon addition of polycation, poly(diallyldimethylammonium) (provided as its chloride salt,
PDAC), it forms a polyion complex. PDAC solution is standardized using the standard
solution of potassium poly(vinyl sulfate) (PVSK). At the endpoint of colloid titration, the
number of anionic groups becomes equal to that of cationic groups.
Procedures
(1) Take precisely 20 mL of the PDAC solution using a volumetric pipette into a 100-mL
conical beaker. Add 2 drops of toluidine blue (TB) into the conical beaker. Titrate the
resulting blue solution with the 0.0025 mol L-1 PVSK (monomer unit concentration)
standard solution.
At the endpoint, the color turns purple.
becomes gradually turbid as the endpoint approaches.
Note that the solution
The endpoint is determined
when the color remains purple for 15-20 seconds. Repeat if necessary.
1a)
Report the PVSK solution volume (in mL) consumed in the standardization of PDAC.
Record your reading to 0.05 mL.
PVSK solution volume consumed in
the standardization of PDAC:
z mL
31
MV(z) = 20.06 mL
A full score of 4 points will be awarded if the answer is MV(z) ± 0.15 mL. (MV: Master Value)
A score of zero will be given if the answer is less than (MV(z) – 0.5) mL or greater than
(MV(z) + 0.5) mL. A linear point scale will be applied for answers in between.
Two points will be subtracted if the value is not reported down to the 2nd decimal place (in
mark
mL).
‐0.5 ‐0.4 ‐0.3 ‐0.2 ‐0.1
0 0.1 0.2 0.3 0.4 0.5
ΔV
(2) Take precisely 5 mL of the polysaccharide solution and 20 mL of the PDAC solution
using volumetric pipettes into another conical beaker.
Add 0.4 mL of 0.5 mol L-1 NaOH
and 2 drops of TB to the solution. Titrate the resulting blue solution with the PVSK
standard solution in a similar manner.
Repeat if necessary.
(The appearance of
coagulation may be different, depending on the pH of the solution.)
1b)
Report the PVSK solution volume (in mL) consumed in the titration under basic
conditions. Record your reading to 0.05 mL.
PVSK solution volume
under basic conditions:
consumed
Sample A: MV(x) = 13.14 mL
Sample B: MV(x) = 12.07 mL
x mL
Sample C: MV(x) = 10.91 mL
A full score of 10 points will be awarded if the answer is MV(x) ± 0.25 mL.
A score of zero will be given if the answer is less than (MV(x) – 0.6) mL or greater than
(MV(x) + 0.6) mL. A linear point scale will be applied for answers in between.
Two points will be subtracted if the value is not reported down to the 2nd decimal place (in
mL). A score of zero will be applied if the value becomes negative after the subtraction.
1c)
Mark the acid group(s) ionized under the basic conditions on the answer sheet.
conditions
basic
acid group
□
X -SO H □
X –COOH
3
Total 1 point.
(3) Repeat procedure 2 above with the addition of 0.5 mL of 0.5 mol L-1 HCl instead of 0.5
mol L-1 NaOH.
32
1d)
Report the PVSK solution volume (in mL) consumed in the titration under acidic
conditions. Record your reading to 0.05 mL.
PVSK solution volume consumed
under the acidic conditions:
y mL
Sample A: MV(y) = 15.26 mL
Sample B: MV(y) = 14.61 mL
Sample C: MV(y) = 13.59 mL
A full score of 10 points will be awarded if the answer is MV(y) ± 0.25 mL.
A score of zero will be given if the answer is less than (MV(y) – 0.6) mL or greater than
(MV(y) + 0.6) mL. A linear point scale will be applied for answers in between.
Two points will be subtracted if the value is not reported down to the 2nd place of decimals
(in mL). A score of zero will be applied if the value becomes negative after the subtraction.
1e)
Mark the acid group(s) fully ionized under acidic conditions on the answer sheet.
conditions
acidic
acid group
□
X -SO H □ –COOH
3
Total 1 point.
1f)
Calculate the concentrations of the -SO3- (or -SO3H) groups and the -COO- (or
-COOH) groups (in mol L-1) in the given polysaccharide solution.
-SO3- (or -SO3H) group:
0.0005(z - y)
-COO- (or -COOH) group:
0.0005(y - x)
mol L-1
mol L-1
Total 4 points, 2 points for each.
A score of 2 is given to the values within (calculated value) ± 0.2. A score of 1 is given to
the values which were outside the above allowance (± 0.2 mol L-1) and within (calculated
value) ± 0.5 mol L-1.
33
3.2
Identification of compounds
You are provided with five solutions (X-1~5, “X” designates your sample code,
which is a letter in the Roman alphabet from A to H), and each solution contains one of
the compounds below (all of which are used). The concentration is 0.05 mol L-1 (for
polymers, monomer unit concentration). Your task is to identify all the compounds by
carrying out the following procedures.
HOCH2CH2OCH2CH2OCH2CH2OH
(TEG)
CH3
CH2CH2O
CH2 C
n
COONa n
(PMANa)
(PEO)
CH2 CH
CH2
H3C
SO3Na n
(PSSNa)
CH2
N+
Cl-
CH3
n
(PDAC)
[Abbreviations: TEG, triethylene glycol; PEO, poly(ethylene oxide);
PMANa, poly(sodium methacrylate); PSSNa, poly(sodium 4-styrenesulfonate);
PDAC, poly(diallyldimethylammonium chloride) MW. stands for molecular weight.]
Helpful comments
1) Aggregates observed in Task 3.1 could be observed when mixing two polymer solutions
in an appropriate combination, in which an interaction takes place between the two
polymers. They can be utilized to identify polymer samples.
2) The volume of a solution measuring 5 mm in height from the bottom of the vial is
approximately 1 mL. Remind that you have only 10 mL of each solution.
34
Procedures
(1) Mix similar volumes of two solutions together in a vial.
(2) If necessary, you can acidify the resulting mixture. Ten drops of hydrochloric acid (0.5
mol L-1 HCl) from a plastic Pasteur pipette are sufficient for this purpose.
Identify the compound in each solution based on the experimental results. For each
solution, mark one of the five boxes to indicate your identification.
You are also asked to
fill in the blanks with one of the letters in the Roman alphabet, from A to H, to indicate your
sample code.
Sample code
-1
□ TEG
□ PEO
□ PMANa
□ PSSNa
□ PDAC
-2
□ TEG
□ PEO
□ PMANa
□ PSSNa
□ PDAC
-3
□ TEG
□ PEO
□ PMANa
□ PSSNa
□ PDAC
-4
□ TEG
□ PEO
□ PMANa
□ PSSNa
□ PDAC
-5
□ TEG
□ PEO
□ PMANa
□ PSSNa
□ PDAC
Before (upper rows) and after (lower rows) the addition of HCl
TEG
PEO
PMANa
PSSNa
TEG
PEO
PMANa
PSSNa
PDAC
-
+
-
+
-
+
+
+: Precipitation, -: No precipitation (or the precipitate disappears)
35
PDAC
PMANa and PSSNa are polyanions, and they interact with a polycation (PDAC) to
form a precipitate. Under acidic conditions, the carboxylate (-COO-) groups in PMANa
undergo protonation, and PMANa changes to protonated poly(methacrylic acid) (PMA).
The resulting carboxy (-COOH) groups interact with the ether oxygen atoms in PEO
through hydrogen bonding to form a precipitate. Since protonated PMA is no longer a
polyanion, the precipitate (the complex between PMANa and PDAC) disappears after the
addition of HCl.
On the other hand, PSSNa does not exist as the protonated form, even under
acidic conditions, and no precipitate is observed with PEO at a lower pH. Since TEG is a
small molecule, its interaction with PMA is not strong enough to form a precipitate.
1) For each correct answer, 4 points will be awarded.
2) If two or more boxes are marked for one sample, 0 points will be given for that sample
even if the correct answer is included in the marked compounds.
3) If the same box is marked for more than two samples, 0 points will be given for these
samples even if the correct answer is included in the marked samples.
Table List of samples in Task 3.2
TEG
PEO
PMANa
PSSNa
PDAC
A-3
A-2
A-1
A-4
A-5
B-2
B-1
B-5
B-3
B-4
C-1
C-5
C-4
C-2
C-3
D-5
D-4
D-3
D-1
D-2
E-3
E-2
E-1
E-4
E-5
F-2
F-1
F-5
F-3
F-4
G-1
G-5
G-4
G-2
G-3
H-5
H-4
H-3
H-1
H-2
36
HOCH2CH2OCH2CH2OCH2CH2OH
(TEG)
CH3
CH2CH2O
CH2 C
n
COONa n
(PMANa)
(PEO)
CH2 CH
CH2
H3C
SO3Na n
(PSSNa)
CH2
N+
Cl-
CH3
n
(PDAC)
[Abbreviations: TEG, triethylene glycol; PEO, poly(ethylene oxide);
PMANa, poly(sodium methacrylate); PSSNa, poly(sodium 4-styrenesulfonate);
PDAC, poly(diallyldimethylammonium chloride)
37
Instructions
• Ensure that your name and student code are written in the spaces provided at the top
of each answer sheet.
• You have 5 hours to work on the problems.
• Use only the pen and the calculator provided.
• All results must be written in the appropriate boxes. Anything written elsewhere will
not be graded. Use the reverse of the sheets if you need scratch paper.
• Write any relevant calculations in the appropriate boxes when necessary. If you
provide no working and only the correct result for a complicated calculation, you will
receive no marks.
• Numerical answers are meaningless without the appropriate units. You will be heavily
penalized if units are not given where required.
• You must stop work immediately when the STOP command is given. A delay in doing
this may lead to your disqualification from the exam.
• When you have finished the examination, you must put your papers into the envelope
provided, and seal the envelope by yourself.
• Do not leave your seat until permitted by the supervisors.
• This examination has 22 pages. The answer booklet comprises 17 pages.
• The official English version of this examination is available on request only for
clarification.
38
Constants and Formulae
Avogadro
constant:
NA = 6.022 x 1023 mol–1
Ideal gas equation:
pV = nRT
Gas constant:
R = 8.314 J K–1 mol–1
Gibbs energy:
G = H – TS
Faraday constant: F = 96485 C mol–1
o
Δ r G o = −RT log e K = −nFE cell
Planck constant:
h = 6.626 x 10–34 J s
Nernst equation:
E = Eo +
Speed of light:
c = 2.998 x 108 m s–1
Energy of a
photon:
E=
Zero of the
Celsius scale:
273.15 K
Lambert-Beer law:
A = log 10
hc
λ
c
RT
log e ox
zF
c red
= hν
I0
= εcl
I
In equilibrium constant calculations all concentrations are referenced to a standard
concentration of 1 mol L-1. Consider all gases ideal throughout the exam.
39
Periodic table with relative atomic masses
1
18
1
2
H
1.01
2
13
14
15
16
17
3
4
5
6
7
8
9
He
4.00
10
Li
Be
B
C
N
O
F
Ne
6.94
9.01
10.81
12.01
14.01
16.00
19.00
20.18
11
12
Na Mg
13
14
15
16
17
18
Al
Si
P
S
Cl
Ar
39.95
22.99
24.30
3
4
5
6
7
8
9
10
11
12
26.98
28.09
30.97
32.06
35.45
19
20
21
22
23
24
25
26
27
28
29
30
31
32
33
34
35
36
Ti
V
Cr
Co
Ni
Cu
Zn
As
Se
Br
Kr
50.94
52.00
54.94
55.85
58.93
58.69
63.55
65.38
69.72
72.64
74.92
78.96
79.90
83.80
41
42
43
44
45
46
47
48
49
50
51
52
53
54
Tc
Ru
Rh
Pd
Ag
Cd
In
Sn
Sb
Te
I
Xe
K
39.10
Ca Sc
40.08 44.96 47.87
37
38
39
40
Rb
Sr
Y
Zr
85.47
56
Cs
Ba
132.91 137.33
87
88
Fr
Ra
-
-
5771
89103
Ga Ge
92.91
95.96
-
72
73
74
75
76
77
78
79
80
81
82
83
84
85
86
Hf
Ta
W
Re
Os
Ir
Pt
Au
Hg
Tl
Pb
Bi
Po
At
Rn
-
-
-
69
70
87.62 88.91 91.22
55
Nb Mo
Mn Fe
101.07 102.91 106.42 107.87 112.41 114.82 118.71 121.76 127.60 126.90 131.29
178.49 180.95 183.84 186.21 190.23 192.22 195.08 196.97 200.59 204.38 207.2 208.98
104
105
106
107
108
109
110
111
Rf
Db
Sg
Bh
Hs
Mt
Ds
Rg
-
-
-
-
-
-
-
-
60
61
62
63
57
58
59
La
Ce
Pr
Nd Pm Sm Eu
138.91 140.12 140.91 144.24
-
89
90
91
92
93
Ac
Th
Pa
U
Np
-
232.04 231.04 238.03
-
64
65
66
67
68
Gd
Tb
Dy
Ho
Er
Tm Yb
71
Lu
150.36 151.96 157.25 158.93 162.50 164.93 167.26 168.93 173.05 174.97
94
95
96
97
Pu Am Cm Bk
-
-
40
-
-
98
Cf
-
102
103
Es Fm Md No
99
Lr
-
100
-
101
-
-
-
Problem 1
8% of the total
1a 1b 1c 1d 1e 1f 1g 1h 1i Task 1
2 4 2 1 1 1 3 2 1
17
In 1894, Lord Rayleigh reported that the mass of chemically prepared nitrogen was
different from that of nitrogen extracted from the atmosphere, as shown in Tables 1 and 2.
Later, this difference was attributed to the presence of argon in atmospheric nitrogen. The
masses of gases were measured by using a glass vessel with a known volume under
atmospheric pressure (1.013 × 105 Pa).
Table 1. Mass of Chemical Nitrogen in the Vessel
From nitric oxide
From nitrous oxide
From ammonium nitrite purified at a red heat
From urea
From ammonium nitrite purified in the cold
2.3001 g
2.2990 g
2.2987 g
2.2985 g
2.2987 g
Mean
2.2990 g
Table 2. Mass of Atmospheric Nitrogen in the Vessel
O2 was removed by hot copper (1892)
O2 was removed by hot iron (1893)
O2 was removed by ferrous hydrate (1894)
2.3103 g
2.3100 g
2.3102 g
Mean
a)
2.3102 g
Calculate the volume V [m3] of the vessel used by Rayleigh from the mean mass of
chemical nitrogen, which must have been pure nitrogen.
Assume that the
measurements were carried out at a temperature of 15.0 °C.
The amount n of the pure nitrogen (chemical nitrogen), M = 28.02 g mol–1, is
m 2.2990
=
= 8.205 × 10–2 mol.
[or equivalent] (1 pt)
n=
M
28.02
nRT
Then, from the ideal gas law, V =
p
=
8.205 ⋅ 10−2 × 8.314 × 288.15
1.013 ⋅ 10
5
= 1.940 × 10–3 m3.
V=
41
(1 pt)
m3
b)
Estimate the mole fraction x of argon in Rayleigh's atmospheric nitrogen, by
assuming that argon and nitrogen were the only constituents. Use the mean masses
of the atmospheric and chemical nitrogen for the calculation.
The equation for the ratio of the mass of atmospheric nitrogen to the mass of
chemical nitrogen is
28.02(1 − x ) + 39.95 x 2.3102
.
[or equivalent] (1 pt)
=
28.02
2.2990
Transformation gives
(2.3102 − 2.2990) / 2.2990
x=
[or equivalent] (2 pt)
× 28.02
39.95 − 28.02
= 1.14 × 10–2 (or 1.14%)
(1 pt)
x=
Ramsay and Clève discovered helium in cleveite (a mineral consisting of uranium oxide
and oxides of lead, thorium, and rare earths; an impure variety of uraninite) independently
and virtually simultaneously in 1895. The gas extracted from the rock showed a unique
spectroscopic line at around 588 nm (indicated by D3 in Figure 1), which was first
observed in the spectrum of solar prominence during a total eclipse in 1868, near the wellknown D1 and D2 lines of sodium.
587
588
589
D3
590 nm
D2
He
D1
Na
Figure 1. Spectral lines around 588 nm
c)
Calculate the energy E [J] of a photon with the wavelength of the D3 line of helium
shown in Figure 1.
According to Figure 1, the wavelength of the D3 line is approximately 587.7 nm (no
punishment if 587.8 or 588 is used).
hc
The corresponding photon energy is E =
λ
=
6.626 ⋅ 10
−34
× 2.998 ⋅ 10
8
(1 pt)
587.7 ⋅ 10−9
= 3.380 × 10–19 J.
(1 pt)
E=
42
J
Figure 2 shows an energy diagram of the atomic orbitals of helium. The arrows indicate
the "allowed" transitions according to the spectroscopic principle.
3.6
E / 10–18 J
3.4
3.2
3d
3.6
[D]
[ E]
[C]
2p
3.4
3.2
2s
[ B]
[ A]
3.0
0.0
3p
3s
3.0
1s
Figure 2. Energy diagram of atomic orbitals of helium
when an electron resides in the 1s orbital.
d)
Identify the transition relevant to the D3 line of helium among the transitions [A] to [E]
indicated in Figure 2. Mark one on the answer sheet:
[E] The energy, 3.382 × 10–19 J, matches with the energy of the transition [E]
between the 2p and 3d orbitals.
(1 pt)
–19
cf.) Energy difference [10 J] = [A]:33.6, [B]:36.9, [C]:5.1, [D]:2.8, [E]:3.4
e)
Which equation explains the occurance of helium in cleveite among [A] to [D] below?
Mark one on the answer sheet:
[A]
238
U → 234Th + α
[B] UHe2 → U + 2He
[C]
240
U → 240Np + β–
[D]
235
U + n → 95Y + 139I + 2n
[A] Considering that the α particle is the nucleus of helium, α-decay [A] is the
relevant source of helium in such rocks. No compound of He such as UHe2
in [B] is known to be stable at ambient temperature. [C] is a radio active
decay of 240U in the thorium series. [D] is a nuclear fission reaction of 235U
occuring in nuclear reactors. The correct answer is [A].
(1 pt)
43
Argon is also found in minerals such as malacon.
f)
Which equation explains the occurance of argon in rocks among [A] to [D] below?
Mark one on the answer sheet.
[A] ArF2 → Ar + F2
[B] ArXe → Ar + Xe
[C]
40
K → 40Ar + ε/β+ (electron capture / positron emission)
[D]
126
I → 126Ar + β–
[C] [C] is a well-known radioactive decay reaction occurring with a half-life of the
order of the age of the earth. No stable compound of Ar, such as ArF2 or
ArXe, can be expected. Products of [D] should be 126Xe + β–. The correct
answer is [C].
(1 pt)
One of the strongest evidences for the monoatomicity of argon and helium is the ratio of
the heat capacity under constant pressure to that at constant volume, γ = Cp / CV, which is
exactly 5/3 (1.67 ± 0.01) for a monoatomic gas. The ratio was derived from the
measurement of speed of sound vs by using the following equation, where f and λ are the
frequency and wavelength of the sound, and R, T, and M denote the molar gas constant,
absolute temperature, and molar mass, respectively.
vs = f λ =
γ RT
M
For an unknown gas sample, the wavelength of the sound was measured to be λ = 0.116
m at a frequency of f = 3520 Hz (Hz = s–1) and temperature of 15.0 °C and under
atmospheric pressure (1.013 × 105 Pa). The density ρ of the gas for these conditions was
measured to be 0.850 ± 0.005 kg m–3.
g)
Calculate the molar mass M [kg mol–1] of this gas.
nM
.
V
By combining with the ideal gas law gives:
ρ RT 0.850 × 8.314 × 288.15
M=
=
p
1.013 ⋅ 105
= 2.01 × 10–2 kg mol–1. (20.1 g mol–1)
The density ρ is given by ρ =
(1 pt)
[or equivalent]
(1 pt)
(1 pt)
M=
44
[or equivalent]
kg mol–1
h)
Calculate the heat capacity ratio γ for this gas sample.
From the equation for the sonic velocity, f λ =
γ=
M
2.01⋅ 10−2
(3520 × 0.116)2
( f λ )2 =
8.314 × 288.15
RT
γ RT
M
,
[or equivalent]
= 1.40
(or, using
(1 pt)
(1 pt)
M
ρ
ρ
0.850
(3520 × 0.116)2 = 1.40)
= , γ = ( f λ )2 =
5
RT p
p
1.013 ⋅ 10
γ =
i)
Which is this gas among [A] to [D]? Mark one on the answer sheet:
[A] HCl
[B] HF
[C] Ne
[D] Ar
[B] From M = 20.1 g mol–1, this gas must be HF or Ne.
From γ = 1.4 (≠ 5/3≈1.67), this is NOT a monoatomic gas (i.e., HCl or HF).
Thus, this gas must be [B] HF.
(1 pt)
Note: It is not possible to distinguish between HF (M = 20.01) and Ne (M = 20.18)
from the molar mass only, which is 20.10±0.12 by taking into account the
uncertainty of ρ (±0.005 / 0.850 = ±0.6%). However, the precision of γ =
1.40 is enough to exclude the possibility of monoatomic gas (γ = 5/3≈1.67).
45
Problem 2
2a
4
2b
4
2c
4
2d
3
6% of the total
2e
5
Task 2
20
Crystal structure of alkali metal halide
In crystals of ionic compounds, cations are generally arranged in the interstices of the
closest packed lattice of anions. The structure of an ionic crystal such as sodium chloride
becomes stable when the cations are in contact with the nearest anions.
a)
In the crystal of sodium chloride, both Na+ and Cl- ions form a face-centered cubic
lattice. Give the numbers of Na+ and Cl- ions in a unit cell and the coordination
numbers of Na+ and Cl- ions in sodium chloride crystal.
Number of ions
Na+:
4
Cl-:
4
Coordination number
Na+:
6
Cl-:
6
[Total 4 pts]
[2 pt] Both number of Na+ and Cl- ions are correct.
[1 pt] Each coordination number of Na+ and Cl- ions is correct.
b)
The ionic radii of Na+ and Cl- ions in the crystal of sodium chloride are 0.102 nm and
0.181 nm, respectively. Calculate the density [kg m-3] of the sodium chloride crystal.
[Total 4 pts]
Length of lattice l: l = 0.102 × 2 + 0.181 × 2 = 0.566 nm [2 pt]
Density ρ:
( 22.99 + 35.45) × 4
ρ=
= 2.1408 × 10 6 g m −3 = 2.14 × 10 3 kg m −3
23
−9 3
( 0.566 × 10 ) × 6.022 × 10
[1 pt for the equation of density, 1 pt for final answer.]
Density of NaCl crystal (kg m-3): 2.14 × 103 kg m-3
46
Born-Haber cycle and lattice enthalpy
In ionic inorganic compounds such as sodium chloride, the heat of lattice formation from
gaseous ions is very high, and the contribution of the change in entropy is small. Therefore,
the lattice formation enthalpy is estimated from enthalpy data by using a Born-Haber cycle.
c)
The figure below shows the Born-Haber cycle of NaCl. The labels “g” and “s”
represent “gas” and “solid” states respectively. Show chemical equations in the A and
F steps.
Na+ (g) + Cl (g) + eD: Dissociation of Cl2 (g)
E: Electron gain by Cl (g)
C: Ionization of Na (g)
F: Dissociation of NaCl (s)
B: Sublimation of Na (s)
A: Formation of NaCl (s)
from elemental substances.
NaCl (s)
A: Na (s) + 1/2Cl2 (g) → NaCl (s) [2 pt]
F: NaCl (s) → Na+ (g) + Cl- (g)
d)
[2 pt]
Calculate the enthalpy of the lattice formation of NaCl [kJ mol-1] by using the following
enthalpy data of the respective steps in the above Born-Haber cycle.
Formation of
NaCl (s)
Sublimation
of Na (s)
Ionization of
Na (g)
–411 kJ mol-1
109 kJ mol-1
496 kJ mol-1
Dissociation
of Cl2 (g)
242 kJ mol-1
Electron gain
by Cl (g)
–349 kJ mol-1
[Total 3 pts]
Enthalpy conservation condition: –A + B + C + D/2 = F – E [1 pt]
From the above equation, –(–411) + 109 + 496 + (242/2) = F + 349,
thus, F=788 [1 pt]
The lattice formation enthalpy of NaCl is –F, thus, –788 kJ mol-1 [1 pt]
Lattice formation enthalpy of NaCl (kJ mol-1): –788 kJ mol-1
47
Synthesis of sodium carbonate by the ammonia-soda process (Solvay process)
Sodium carbonate (anhydrous soda ash) is a raw material in the manufacture of glass,
medicaments, alkaline detergents, etc.
e)
The total chemical reaction in the ammonia-soda process is represented as follows:
2NaCl + CaCO3 → Na2CO3 + CaCl2
This reaction between sodium chloride and calcium carbonate does not proceed
directly. The process comprises the following five reactions involving ammonia:
CaCO3 Δ
→[A]+[B]
NaCl + NH3 + [ B ] + H2O →[ C ] + [ D ]
Δ
2[C]→
Na2CO3 + H2O + [ B ]
[ A ] + H2O → [ E ]
[ E ] + 2 [ D ] → CaCl2 + 2H2O + 2NH3
where Δrepresents applying heat treatment. Insert the chemical formulas of the
appropriate compounds in the blank spaces [ A ]–[ E ] in the above reactions.
A: CaO
B: CO2
C: NaHCO3
D: NH4Cl
E: Ca(OH)2
[Total 5 pts]
1 pt for one correct chemical formula.
48
Problem 3
3a 3b 3c 3d
2 3
1 3
7% of the total
Task 3
9
The chemical oxygen demand (COD) refers to the amount of oxidizable substance, such
as organic compounds, in a sample solution, and it is used as an indication of water quality
in seas, lakes, and marshes. For example, the COD of service water is kept below
1 mg L-1. The COD (mg L-1) is represented by mass of O2 (mg) which accepts the same
amount of electrons which would be accepted by the strong oxidizing agent when 1 L of a
sample solution is treated with it. An example of the operation is presented below.
******************************************************
Analytical Operation
A 1.00-L sample solution was acidified with a sufficient amount of sulfuric acid, and
chloride ions were removed by the addition of silver nitrate solution. To the sample solution,
1.00 × 10-1 L of 5.00 × 10-3 mol L-1 potassium permanganate solution was added, and the
mixture was heated for 30 min. Further, 1.00 × 10-1 L of 1.25 × 10-2 mol L-1 disodium
oxalate (Na2C2O4 or NaOOC-COONa) standard solution was added, and the mixture was
stirred well. Oxalate ions that remained unreacted were titrated with 5.00 × 10-3 mol L-1
potassium permanganate solution; 3.00 × 10-2 L of the solution was used for the titration.
******************************************************
a)
Give the equation of the redox reaction of potassium permanganate and disodium
oxalate.
2KMnO4 + 5Na2C2O4 + 8H2SO4 → 2MnSO4 + 5Na2SO4 + K2SO4 + 10CO2+ 8H2O
or
2KMnO4 + 5H2C2O4 + 3H2SO4 → 2MnSO4 + 10CO2 + 8H2O + K2SO4
or
2 MnO4- + 5C2O42- + 16H+ → 2Mn2+ + 10CO2 + 8H2O
[Total 2 pts]
b)
Calculate the amount of O2 (mg) that will oxidize the same number of moles of
oxidizable substance as 1.00 × 10-3 L of 5.00 × 10-3 mol L-1 potassium
permanganate does.
The reactions of potassium permanganate and O2 are as follows:
MnO4- + 8H+ + 5e- → Mn2+ + 4H2O
O2 + 4H+ + 4e- → 2H2O
49
Hence, 1 mol of KMnO4 amounts to 1.25 mol of O2.
5 × 5.00 × 10-3 (mol L-1) × 10-3 (L) = 4 × X / 32 (mol)
where X is the amount of O2 (g).
Thus, X = 2.00 × 10-4 g.
c)
→
2.00 × 10-1 mg
[Underlined (or equivalent)
equation: 2 pt]
[1 pt]
[Total 3 pts]
From the following choices, select the most appropriate reason for the removal of
chloride ions:
[A] Some of the chloride ions react with potassium permanganate, resulting in an
error in COD.
[B] Some of the chloride ions react with disodium oxalate, resulting in an error in
COD.
[C] Some of the chloride ions react with organic compounds in the sample solution,
resulting in an error in COD.
[D] A color is developed during titration, resulting in an error in COD.
[A]
[Total 1 pt]
d)
Calculate the COD (mg L-1) of the sample solution described in the analytical
operation above.
The amounts of electron used for reduction and oxidation are equal, then
5 × 5.00 × 10-3 (mol L-1) × (1.00 × 10-1 + A) (L) =
2 × 1.25 × 10-2 (mol L-1) × 1.00 × 10-1 (L) + X
(1)
[Underlined (or equivalent)
where A (mL) is the amount of potasium permanganate
equation: 2 pt]
used for the final titration, and X (mol) is the amount of
electron for the oxidizable substance.
Eq.(1) gives X = 2.50 × 10-2 × A.
At A = 3.00 ×10-2 (L), X = 7.50 × 10-4 (mol).
Hence, COD = (32/4) (g mol-1) × 7.50 × 10-4 (mol) × 103(mg/g) × 1/1(L-1)
= 6.00 mg L-1.
[1 pt]
[Total 3 pts]
or
The amount of potasium permanganate consumed for the oxidizable substance, B (mL), is
5 × 5.00 × 10-3 × (1.00× 10-1 + A − B) = 2 × (1.25 ×10-2) × (1.00 × 10-1).
[2 pt]
-2
-2
At A = 3.00 ×10 L, B equals to 3.00 ×10 L.
From the solution to question b) above,
COD = (2.00 ×10-1) / (1.00 ×10-3) (mg/L) × 3.00 × 10-2 (L) × 1/1(L-1) = 6.00 mg L-1. [1 pt]
[Total 3 pts]
50
Problem 4
4a 4b 4c 4d
2 3 2 1
6% of the total
Task 4
8
The rechargeable lithium ion battery has been developed in Japan.
The standard electromotive force of the battery is 3.70 V. Assume that the half-reaction at
the cathode is
CoO2 + Li+ + e- → LiCoO2,
and the half-reaction at the anode is
LiC6 → 6C + Li+ + e-.
a)
Write the total reaction equation of the battery and calculate the value of the standard
Gibbs energy of the reaction [kJ mol-1].
Total reaction equation:
CoO2 + LiC6 → LiCoO2 + 6C
(1 pt)
The standard Gibbs energy of the reaction:
ΔG0 = –nFE0 = –1 × 96485 C mol-1 × 3.70 V = –357 kJ mol-1
b)
(1 pt)
The battery cell is constructed using LiCoO2 and graphite (C) as the electrode
materials. Calculate the mass of the anode in the completely charged state and that
in completely discharged state if 10.00 g of LiCoO2 and 10.00 g of graphite (C) are
present initially.
In the completely charged state: 10.71 g
(2 pt)
The amount of LiCoO2 is 10.00/97.87 = 0.1022 mol.
The amount of C is 10.00/12.01 = 0.8326 mol, which is larger than 0.1022 mol × 6 =
0.6132 mol.
Thus, the mass in the completely charged state of the anode is 10.00 + 0.1022 × 6.94 =
10.709 g = 10.71 g.
In the completely discharged state: 10.00 g
51
(1 pt)
c)
Calculate the maximum energy generated per mass of the lithium ion battery cell [kJ
kg-1]. Assume that the correct ratio for complete reaction between the cathode and
anode materials is used and the sum of the mass of electrodes is 50.0% of the total
mass of the battery cell. In comparison, the energy density of lead-acid batteries used
for vehicles is about 200 kJ kg-1.
The mass of 1 mol LiCoO2 is 97.87 g
The mass of 6 mol C is 12.01 × 6 g = 72.06 g
The total mass of the electrode is (97.87 + 72.06) g = 169.93 g
The mass of the cell is 169.93 / 0.500 g = 340 g
The maximum energy generated is 357 kJ.
Thus, the maximum energy per unit mass of the cell is 1050 kJ kg-1
d)
(2 pts)
Because an aqueous solution cannot be used as an electrolyte, an organic solution is
used in the lithium ion battery cell. Give the chemical formula of the gas generated if
water is present in the electrolyte.
H2 or H2 and O2 (1 pt)
52
Problem 5
7% of the total
5a-1 5a-2 5b 5c 5d 5e 5f Task 5
1
1
2 2 3 4 5
18
When an atom X absorbs radiation with a photon energy greater than the ionization
energy of the atom, the atom is ionized to generate an ion X+ and the electron (called a
photoelectron) is ejected at the same time. In this event, the energy is conserved as
shown in Figure 1, that is,
Photon energy (hν) = ionization energy (IE) of X + kinetic energy of photoelectron.
When a molecule, for example, H2, absorbs short-wavelength light, the photoelectron is
ejected and an H2+ ion with a variety of vibrational states is produced. A photoelectron
spectrum is a plot of the number of photoelectrons as a function of the kinetic energy of
the photoelectrons. Figure 2 shows a typical photoelectron spectrum when H2 in the
lowest vibrational level is irradiated by monochromatic light of 21.2 eV. No
photoelectrons are detected above 6.0 eV. eV is a unit of energy and 1.0 eV is equal to
1.6 × 10-19 J.
Kinetic energy of
photoelectron
IE
hν
Photoelectron spectrum of H 2
h ν = 21.2 eV
Intensity (arb.)
X+
X
Figure 1. Schematic diagram of
photoelectron spectroscopy.
6.0
5.0
4.0
Kinetic energy of photoelectron (eV)
3.0
Figure 2. Photoelectron spectrum of H2. The energy of the
incident light is 21.2 eV.
53
a-1)
Determine the energy difference ΔEA1 (eV) between H2 (v = 0) and H2+ (v ion = 0) to
the first decimal place. v and v ion denote the vibrational quantum numbers of H2 and
H2+, respectively.
a-2)
Determine the energy difference ΔEA2 (eV) between H2+ (v ion = 0) and H2+ (v ion = 3)
to the first decimal place.
a-1) & a-2)
The spectral peak at 5.8 eV in Fig. 2 corresponds to the electron with the highest kinetic
energy, which is generated by the reaction H2(v = 0) → H2+(vion = 0) + e. Accordingly,
ΔEA1 = 21.2 eV – 5.8 eV = 15.4 eV
One can estimate from Fig. 2 that the energy difference ΔEA2 between H2+ (v ion = 0) and
H2+ (v ion = 3) is approximately 0.8 eV.
ΔEA1 (eV) = 15.4 eV
1 pt
ΔEA2 (eV) = 0.8 eV
1 pt
b)
The electronic energy levels E nH of a hydrogen atom are given by the equation
E nH = −
Ry
n2
(n = 1, 2, 3Λ )
.
Here n is a principal quantum number, and Ry is a constant with dimensions of energy.
The energy from n = 1 to n = 2 of the hydrogen atom is 10.2 eV. Calculate the
ionization energy EB (eV) of the hydrogen atom to the first decimal place.
The ionization energy corresponds to n = ∞. Accordingly,
3
∆E n =2←n =1 = Ry
4
∆E n =∞ ←n =1 = Ry
Thus, the energy required for the ionization is 4/3 times larger than the transition energy of
the Lyman α line.
4
EB = 10.2 eV × = 13.6 eV
3
EB (eV) = 13.6 eV
2 pts
54
c)
The energy threshold for the generation of two electronically excited hydrogen atoms
H* (n = 2) from H2 (v = 0) has been derived to be 24.9 eV by an experiment.
Determine the bond energy EC (eV) of H2 to the first decimal place.
24.9 eV = the binding energy of a hydrogen molecule + 10.2 eV + 10.2 eV.
Thus, the binding energy of a hydrogen molecule = EC = 4.5 eV.
EC (eV) = 4.5 eV
d)
2 pts
Considering an energy cycle, determine the bond energy ED (eV) of H2+ to the first
decimal place. If you don’t have the values for EB and EC, then use 15.0 eV and 5.0
eV for EB and EC, respectively.
From Figure 3 below,
ED = EB + EC – ΔEA1 =13.6 + 4.5 – 15.4 = 2.7 eV.
ED (eV) = 2.7 eV
e)
3 pts
Calculate the threshold energy EE (eV) of the following dissociative ionization
reaction to the first decimal place:
H ⎯⎯
→ H* (n = 2) + H+ + e- .
2
If you don’t have the values for EB and EC, then use 15.0 eV and 5.0 eV for EB and
EC, respectively.
H + H+ + eED = 2.7 eV
H2+ + e-
EB=13.6 eV
ΔEA1=15.4 eV
H+H
EC=4.5 eV
H2
From Figure 3 above, the threshold energy for the dissociative ionization reaction
H2 → H* (n = 2) + H+ + e- is EB + EC + 10.2 eV = 13.6 + 4.5 + 10.2 = 28.3 eV.
EE (eV) = 28.3 eV
4 pts
f)
When H2 absorbs monochromatic light of 21.2 eV, the following dissociation
process occurs at the same time.
55
H
2
21.2
eV
⎯⎯
⎯⎯
⎯→ H (n = 1) + H (n = 1)
Two hydrogen atoms move in opposite directions with the same speed. Calculate
the speed u (m s-1) of the hydrogen atoms generated in the above reaction. H2 is
assumed to be at rest. If you don’t have the value for EC, then use 5.0 eV for EC.
The excess energy is 16.7 eV (= 21.2 eV – 4.5 eV). Because two hydrogen atoms are
generated upon photodissociation, half of this excess energy is released as translational
energy of the hydrogen atoms.
1
mu 2 = 8.35 eV = 1.34 × 10 -18 J
2
(2 pts)
1.008 × 10 -3 kg mol -1
− 27
m=
= 1.67 × 10 kg
6.022 × 10 23 mol -1
Then,
u 2 = 1.6 × 10 9 m 2 s -2 u ≈ 4.0 × 10 4 m s -1
u (m/s) = 4.0 × 104 m/s
5 pts
56
Problem 6
6 % of the total
6a 6b 6c 6d Task 6
5 4 6 11
26
Read the description of four kinds of isomeric organic compounds of A, B, C, and D. All
have C8H10O and contain a benzene ring. Answer the questions that follow. If there are
stereoisomers, give all structural formulas. Note that any wrong isomers will be penalized.
z
(1)At room temperature, a piece of sodium metal was added to A, B, and C in test
tubes and the evolution of hydrogen gas was observed only in the case of C.
z
When an iron(III) chloride aqueous solution was added to C and D, no coloration was
observed in C, whereas D was colored.
z
A was oxidized when (2)aqueous potassium permanganate was added to it and the
mixture was heated; the acidification of the heated mixture and its isolation afforded
benzoic acid.
z
Let’s imagine that (3)a hydrogen atom in the benzene ring is replaced by a chlorine
atom, it is possible to obtain four kinds of monochlorinated structural isomers from B,
while only two kinds of such isomers can be obtained from D.
z
Hydrogenation of the benzene ring in C and D using a catalyst gave saturated
alcohol(s). It was found that the saturated alcohol(s) obtained from C has no
stereogenic carbons, but the one(s) from D has stereogenic carbon(s).
a)
Among all the isomeric organic compounds of C8H10O having a benzene ring, give
the structural formulas of all the isomers that do NOT yield hydrogen gas in the
underlined procedure (1), in which a piece of sodium is added to the neat samples in
the case of the liquid samples and to the concentrated solution of the samples in an
aprotic solvent in the case of the solid ones.
O
O
O
O
O
1pt each
wrong isomer: -1pt each
total pts ≥ 0 (not negative)
b)
Among all the isomeric organic compounds of C8H10O having a benzene ring, give
the structural formulas of all the isomers that yield benzoic acid in the underlined
procedure (2).
57
OH
OH
OH
O
1pt each
wrong isomer: -1pt each
total pts ≥ 0 (not negative)
c)
Among all the isomeric organic compounds of C8H10O having a benzene ring, give
the structural formulas of all the isomers that could yield four different monochlorinated structural isomers when the underlined transformation in (3) is performed.
OH
OH
OH
OH
O
O
1pt each
wrong isomer: -1pt each
total pts ≥ 0 (not negative)
d)
Give the structural formulas of A, B, C, and D. When several isomers can be
considered, give the structural formulas of all of them.
A
B
O
O
O
1pt
wrong isomer: -1pt each
total pts ≥ 0 (not negative)
1pt each
wrong isomer: -1pt each
total pts ≥ 0 (not negative)
OH
C
OH
D
2pts each
wrong alcohol/phenol: -1pt each
ether: -2pts each
total pts ≥ 0 (not negative)
58
OH
OH
2pts each
wrong alcohol/phenol: -1pt each
ether: -2pts each
total pts ≥ 0 (not negative)
Problem 7
7% of the total
7a 7b 7c 7d Task 7
4 9 6 5
24
Certain varieties of puffer fish, Fugu in Japanese, are highly prized as foods in Japan.
Since the viscera (especially ovaries and livers) of the fish contain a potent toxin
(tetrodotoxin), food poisoning often results from its ingestion. Studies on tetrodotoxin (1)
have been performed from the beginning in the 20th century; its chemical structure was
elucidated in 1964.
O–
HO
H
H
N
N
H HO
H2N
O
OH
O
H
H
H
H
OH
OH
tetrodotoxin (1)
a)
The guanidine group in tetrodotoxin exhibits strong basicity. The guanidinium ion
resulting from protonation on the guanidine group is stabilized by the existence of the
following resonance. Draw two resonance structures B and C.
NHR1
B
C
NHR2
H2N
A
B
C
NHR1
NHR1
H 2N
NHR2
H2N
NHR2
2 pts each.
b)
Many derivatization reactions were performed in structure studies of tetrodotoxin.
Treatment of tetrodotoxin (1) with ethanolic potassium hydroxide upon heating
afforded quinazoline derivative 2, which provided an insight into the nature of the
fundamental skeleton of tetrodotoxin. The reaction mechanism can be described as
follows. First, tetrodotoxin is hydrolyzed into carboxylate 3. Then the hydroxyl group
highlighted with a frame is eliminated by the base to give intermediate D. A retroaldol reaction of D cleaves a carbon-carbon bond to provide intermediates E and F.
Finally, dehydration and aromatization from E produce quinazoline derivative 2. Draw
structures of the postulated intermediates D, E, and F.
59
O–
HO
H
H
N
N
H HO
H2N
H2O
base
O
OH
O
HO
H
H
N
N
H HO
H2N
H
H
H
H
1
OH
OH
OH OH
H
COO–
OH
HO
OH
H
H
H
H
OH
HN
H2N
OH
N
H
HO
H
H2N
N
intermediate E
dehydration &
aromatization
OH
base
base
base
OH
OH
OH
COO–
3
3
dehydration
(-H2O)
N
OH
OH
intermediate D
retro-aldol reaction
F
2
D
E
OH OH
H
OH OH
H
OH
HN
H2N
HN
OH
N
H
F
H2N
HO
H
O
COO–
OH
N
H
H
OH
O
COO–
O
3 pts each. Other stereoisomers are acceptable. Each free form is acceptable.
Tautomors concerning guanidine moiety are all acceptable. Enol form is acceptable.
E: dehydrated products and zwitterionic structure are acceptable.
c)
Although biosynthesis of tetrodotoxin still remains to be clarified, it is proposed that
tetrodotoxin may be biologically synthesized from L-arginine and isopentenyl
diphosphate. Among the carbons included in tetrodotoxin, circle all the carbons that
are expected to be of L-arginine origin.
NH2
H2N
COO–
N
H
–O
NH2
L-arginine
H 2N
O
O
P O P O
O–
O–
isopentenyl diphosphate
O–
HO
H
H
N
N
H HO
O
OH
O
H
H
H
H
OH
OH
60
tetrodotoxin (1)
6 pts. 1 pt each for correct carbon. Deduct 1 pt for each carbon over 6. 0 pts for all
carbons circled.
In the 1990s, an alternative biosynthetic pathway of tetrodotoxin was proposed.
Condensation between 2-deoxy-3-oxo-D-pentose and guanidine provides
intermediate G with cyclic guanidine moiety (molecular formula C6H11N3O3).
Tetrodotoxin may be biologically synthesized from intermediate G and isopentenyl
diphosphate. Draw a structure of the postulated intermediate G showing the
stereochemistry.
d)
HO
O
NH2
OH
HN
O
G ( C6H11N3O3)
NH2
HO
OH
tetrodotoxin (1)
O
O
–O P O P O
O–
O–
2-deoxy-3-oxo-D-pentose
isopentenyl diphosphate
CHO
O
G
OH
H
N
HO
NH
NH
OH
5 pts (enantiomer at C4, 3 pts, C4 stereochemistry is unclear, 3 pts).
OH
OH
HO
H
N
HO
HO
NH
NH
NH
N
NH
OH
HO
HO
OH
H
N
NH
N
acceptable
Each zwitterionic structure (and protonated structure) like below is acceptable.
OH
HO
H
N
NH2
NH
O–
Tautomers concerning guanidine moiety are all acceptable.
61
Problem 8
8a-1
2
8a-2
4
8a-3
3
8b-1
4
6% of the total
8b-2
7
Task 8
20
The esterification reaction between bi-functional molecules gives one of the typical linear
chain polymers as shown in eq.(1) by polycondensation (often called “condensation
polymerization”). The control of polymerization conditions and procedures determines the
length of polymer strands, i.e., the average degree of polymerization, X (note that X = 2n
in the present instance). Because X (and also n ) is an averaged number, it is not always
an integer but a value with decimal figures.
n HOOC-R1-COOH + n HO-R2-OH → HO-[COR1CO-OR2O]n-H + (2n-1)H2O (1)
X can be estimated from the consumption of functional groups (here, -COOH and -OH).
Let us define the degree of reaction, p, as p = (N0 - N) / N0 (≦ 1), where N0 and N denote
the total numbers of functional groups before and after the polymerization, respectively.
For each functional group of the dicarboxylic acid molecules (A) and diol molecules (B),
we add the suffixes of “A” or “B” such as NA0, NB0, NA or NB, respectively, i.e., N0 = NA0 +
NB0 and N = NA + NB. When the initial feed is unbalanced such as NA0 ≦ NB0, X is
expressed by pA and r as shown in eq.(2), where r = NA0 / NB0 (≦ 1) and pA = (NA0 – NA) /
NA0. If r = 1, pA is identical to p and eq.(2) becomes the same to the Carothers equation.
X = (1 + r) / (1 + r - 2pAr)
(2)
a) Some nylon-6,6 sample was prepared by polycondensation between an equimolar
mixture of adipic acid (hexanedioic acid) and hexamethylenediamine (hexane-1,6diamine).
a-1) Show the chemical structure of this nylon-6,6 sample. [Caution: what are the end
groups when polycondensation was started from the equimolar mixture?]
HO-[CO(CH2)4CO-NH(CH2)6NH]n-H or equivalent structures are all OK.
Total 2 pts. -0.5 pt for lacking “n,” another -0.5 pt for lacking each of the end group(s).
62
a-2) When this nylon-6,6 sample carries an average molecular weight, M, of 5507.25
(g mol-1), give its X value to the second decimal place.
Calculation procedures must be shown by suitable equation(s) (otherwise, no
score will be provided):
The unit molecular weight, Mu, is calculated to be,
Mu = (12.01 × 12 + 1.01 × 22 + 14.01 × 2 + 16.00 × 2) / 2 = 226.36 / 2 = 113.18
X = (5507.25 - 18.02) / Mu = (5507.25 - 18.02) / 113.18 = 48.50, or
X = 2n = 2 × [(5507.25 - 18.02) / 226.36] = 48.50
X = 48.50
Black parts are prewritten in the answer boxes (same to all questions in Problem 8).
Underlined (or equivalent) calculation procedures are required. Total 4 pts. -1 pt for
calculation mistakes.
a-3) Give the p value necessary to prepare this nylon-6,6 sample of M = 5507.25
(g mol-1) to the fifth decimal place. If you get no numerical answer in a-2), use
52.50 instead.
From eq.(3) at r = 1 (Carothers eq.), X = 48.50 = 1 / (1 - p), then p = 0.979381
≒ 0.97938
p = 0.97938 (0.98095 when X = 52.50.)
Total 3 pts. -1 pt for calculation mistakes.
b) The low-molecular-weight polyester (oligoester) is prepared from the mixture of 36.54
(g) of adipic acid (hexanedioic acid) and an unknown amount [W (g)] of butane-1,4-diol
(Bdiol). Under the condition of pA→1, the oligoester with X = 11.00 carrying Bdiol units
in both chain ends is obtained.
b-1) Show the precise chemical structure of this oligoester of X = 11.00.
[HO(CH2)4O]1.000-[CO(CH2)4CO-O(CH2)4O]5.000-H or
HO(CH2)4O-[CO(CH2)4CO-O(CH2)4O]5.000-H is accurate, however,
is acceptable.
HO(CH2)4O-[CO(CH2)4CO-O(CH2)4O]5-H
Total 4 pts, -1 pt for lacking the number of unit repeating or writing “n” instead of “5.00
(or 5)”. Another -1 pt for lacking HO- and/or -H end group(s). No point if lacking the leftmost HO(CH2)4O- group.
63
b-2) Calculate the unknown amount, W (g), to the first decimal place.
Calculation procedures must be shown by suitable equation(s) (otherwise, no
score will be provided):
Mw(adipic acid) = 146.16, Mw(Bdiol) = 90.14
Ans.1 Since X = 11.00, the oligoester contains 5.00 units of adipate and 6.00 units of
Bdiol. [cf) 5.00 + 6.00 = 11.00 = X] When pA→1, the initial molar feed ratio of the
monomers is equal to the molar composition of the resulting oligoester.
[adipic acid]0 / [Bdiol]0 = 5.00 / 6.00, W = 90.14 × (6.00 / 5.00) × (36.54 / 146.16) =
27.042 ≒ 27.0 (g)
Ans.2 From eq.(2), when pA→1, X = (1 + r) / (1 - r). Therefore,
11.00 = [1 + {(36.54 / 146.16) / (W / 90.14)}] / [1 - {(36.54 / 146.16) / (W / 90.14)}]
= [(W / 90.14) + 0.2500] / [(W / 90.14) - 0.2500]
11.00 × [(W / 90.14) - 0.2500] = [(W / 90.14) + 0.2500], 10.00 × (W / 90.14) = 3.000
W = 3.000 × 90.14 / 10.00 = 27.042 ≒ 27.0 (g)
W = 27.0 (g)
Either calculation procedures are acceptable. Underlined (or equivalent) calculation
procedures are required. Total 7 pts, -1 pt for calculation mistakes.
64
Problem 9
7% of the total
9a 9b 9c 9d 9e 9f Task 9
4 2 8 4 8 8
34
α-Cyclodextrin (αCyD), which is a cyclic oligosaccharide of six α(1 → 4) linked α-Dglucopyranoside units, can be topologically represented as toroids (Figure 1). α-Dglucopyranoside units in αCyD are usually in the most stable chair conformation.
α
αCyD
Figure 1. Space filling model of αCyD. Left:
view through the hole. Right: side
view.
αCyD
a)
Give the absolute configuration (R or S) at stereogenic carbons C-2 and C-5 of Dglucose. Also, draw a stereostructure of the open chain form of D-glucose.
Absolute configuration at C-2:
R
Chain form:
1 pt
O
H
H
OH
HO
OH
Absolute configuration at C-5:
R
1 pt
OH
O
HO
H
OH
OH
or
H
H
OH
H
OH
OH
2 pts (carbon skeleton: 1 pt; others: 1 pt)
65
b)
Choose the most stable conformation from the four incomplete α-D-glucopyranose
formulas given in the answer box and enclose it in a box. Also, add four OH groups
and four H atoms to complete the α-D-glucopyranose formula.
H
H
OH
OH
O
O
H
O
O
H
OH
OH
Answer:
OH
H
H
HO
O
H
HO
H
2 pts (4C1 : 1 pt; -OH: 1 pt)
OH
OH
H
HO
H
H
OH
O
H
H
H
OH
OH
1 pts (4C1 : 0 pt; -OH: 1 pt)
OH
Others 0 pt
66
αCyD in water is able to host hydrophobic molecules. When the host/guest (H/G)
stoichiometry is 1/1, the inclusion complexation can be given by the following equilibrium.
G
+
H
k1
k-1
HG
(1)
where k1 and k-1 are the rate constant for the forward and backward reaction, respectively.
The complexation of a guest to αCyD causes a chemical shift change in 1H NMR spectra.
Figure 2 shows a part of 1H NMR spectra (signals from H-1 of αCyD) showing the
chemical
shift
change
in
the
presence
of
varying
amounts
of
1,10-bis(trimethylammonium)decane diiodide (BTAD). The doublet peak at 5.06 ppm is
from H-1 of free αCyD, while the doublet at 5.14 ppm is from H-1 of αCyD complexed with
BTAD. (Note that the spectra given in Figure 2 were measured in the complexation
equilibrium state.)
BTAD
Figure 2. Expanded 1H NMR
spectra (signals from H-1 of
αCyD) of solutions containing
5.0×10-3 mol L-1 αCyD and
0-3.0 ×10-2 mol L-1 BTAD.
c)
In the spectrum of 5.0 x10-3 mol L-1/5.0 x10-3 mol L-1 αCyD/BTAD, the relative peak
areas of the doublets at 5.06 and 5.14 ppm are 0.41 and 0.59, respectively. Calculate,
to 2 significant figures, the concentration equilibrium constant, K for the inclusion
complexation of αCyD/BTAD.
[HG]
[αCyD]0×a5.14
5.0 x10-3 M × 0.59
K = --------- = --------------------------------------------------------- = -------------------------- = 0.70 x103
[H][G] [αCyD]0 × a5.06 × {[BTAD]0 – [αCyD]0 × a5.14} (5.0x10-3 M × 0.41)2
1 pt
3 pts
3 pts
a5.06: relative area of the peak at 5.06 ppm = mole fracrion of free αCyD
a5.14: relative area of the peak at 5.14 ppm = mole fracrion of αCyD complexed with BTAD
K: 7.0 × 102
8 pts in total
67
1 pt
Complexation of αCyD with hexyltrimethylammonium bromide (HTAB) appears in NMR
spectra in a way different from the αCyD/BTAD complexation. Figure 3 shows a part of 1H
NMR spectra (H-6 signal of HTAB) in αCyD/HTAB solutions. The signal appears as one
triplet (not two triplets), which shifts depending on the concentration of αCyD from the
position of free HTAB to the position of αCyD/HTAB in proportion to the fraction of the
complex in the solution. The H-6 signals from free HTAB and HTAB complexed with αCyD
are triplets at 0.740 ppm and 0.860 ppm, respectively.
HTAB
Figure 3. Expanded 1H NMR
spectra (H-6 signal of HTAB)
of
solutions
containing
1.0×10-2 mol L-1 HTAB and
0-3.0×10-2 mol L-1 αCyD.
d)
The signal of HTAB in αCyD/HTAB solutions appears as one triplet, which shifts
depending on the concentration of αCyD. Choose the rational interpretation(s) just
from these spectra.
hint: When a guest molecule move in and out of αCyD rapidly and repeatedly, only
one signal of the guest is observed at the weighted average of the chemical
shifts of the free guest and the shift of the guest included in αCyD.
a. k1 of αCyD/HTAB > k1 of αCyD/BTAD
b. k1 of αCyD/HTAB < k1 of αCyD/BTAD
c. K of αCyD/HTAB > K of αCyD/BTAD
d. K of αCyD/HTAB < K of αCyD/BTAD
a
4 pts (additional choice : –2 pts for each)
68
e)
The signals of HTAB in 1.0 x10-2 mol L-1/1.0 x10-2 mol L-1 αCyD/HTAB are positioned
at 0.815 ppm. Calculate, to 2 significant figures, K for the complexation of
αCyD/HTAB.
In 1.0 x10-2 mol L-1/1.0 x10-2 mol L-1 αCyD/HTAB,
s10/10 – sfree
0.815 – 0.740
f10/10 = ------------------- = ---------------------- = 0.625
scomplex – sfree
0.860 – 0.740
3 pts
sfree, scomplex: chemical shift of HTAB in free, and complexed state
s10/10: chemical shift of HTAB in 10.0 mM/10.0 mM αCyD/HTAB
f10/10: mole fraction of complexed HTAB in 10.0 mM/10.0 mM αCyD/HTAB
[HG]
K = --------[H][G]
[HTAB]0 × f10/10
1.0 x10-2 mol L-1 × 0. 625
= ---------------------------------------------------------- = ------------------------------------------{[αCyD]0 – f10/10 [HTAB]0}[HTAB]0(1 – f10/10) [1.0 x10-2 mol L-1 × (1 – 0. 625)]2
2 pts
2 pts
2
= 4.4 × 10
K: 4.4 × 102
f)
1 pt
8 pts in total
At 40.0 ºC and 60.0 ºC, K for the complexation of αCyD/HTAB are 3.12 × 102 and
2.09 × 102 respectively. Calculate, to 2 significant figures, the enthalpy change, ∆Hº
[kJ mol-1], and the entropy change, ∆Sº [J K-1 mol-1]. (Ignore the temperature
dependence of ∆Hº and ∆Sº.)
From ΔGº= –RT ln K,
ΔGº (40.0 oC) = –8.314 × 313.2 ln (3.12 × 102) = –14.94 × 103 J mol–1
ΔGº (60.0 oC) = –8.314 × 333.2 ln (2.09 × 102) = –14.79 × 103 J mol–1
2 pts each
From ΔGº=ΔHº-TΔSº
–14.94 × 103 = ΔHº – 313.2 × ΔSº
–14.79 × 103 = ΔHº – 333.2 × ΔSº
2 pts
ΔSº = –7.5 J K–1 mol–1, ΔHº = –17 kJ mol–1
ΔSº: –7.5 J K–1 mol–1
ΔHº: –17 kJ mol–1
1pt
1pt
8 pts in total
69
Final Results and Ranking
Rank
Name
1
Xianghang Shangguan
2
Daniil Khokhlov
3
4
6
7
Pilkeun Jang
Robert Pollice
Seyed Amirhossein
Nasseri
Qilei Zhu
Alif Noikham
8
Ruth Franklin
9
Khetpakorn Chakarawet
10
Yu-Chi Kuo
11
12
13
14
17
18
19
20
21
22
23
24
Zhiyao Zhou
Assaf Mauda
Manoel Manuputty
Ruyi Wang
Rafael Angel Rodriguez
Arguedas
Pinnaree
Tea-Mangkornpan
Hayate Saitoh
Eszter Najbauer
Ken-Ichi Endo
Gleb Široki
Colin Lu
Alexander Siegenfeld
Máté Somlyay
Hyeonjae Lee
25
Ondrej Hak
26
27
Fong Jie Ming Nigel
Lum Jian Yang
28
Frantisek Petrous
5
15
16
30
31
Nicolas Villagran
Dos Santos
Jaehyun Lim
Vladimiras Oleinikovas
32
Ming-Ko Cho
33
34
Diptarka Hait
Deniz Caglin
35
Pavel Svec
36
Constantin Giurgiu
29
Country
China
Russian
Federation
Korea
Austria
Practical
Theoretical
(max 40)
(max 60)
36.960
59.611
Total
(max 100)
96.571
Medal
Gold
39.350
56.567
95.917
Gold
37.077
36.389
57.625
58.200
94.702
94.589
Gold
Gold
I. R. of Iran
37.404
56.370
93.774
Gold
China
Thailand
United
Kingdom
Thailand
Chinese
Taipei
China
Israel
Indonesia
China
35.725
35.503
57.403
57.482
93.128
92.985
Gold
Gold
36.089
56.890
92.979
Gold
35.234
57.635
92.869
Gold
35.885
56.924
92.809
Gold
32.974
35.623
36.285
32.668
59.708
56.936
56.222
59.769
92.682
92.559
92.507
92.437
Gold
Gold
Gold
Gold
Costa Rica
36.392
55.894
92.286
Gold
Thailand
32.200
60.000
92.200
Gold
Japan
Hungary
Japan
Estonia
United States
United States
Hungary
Korea
Czech
Republic
Singapore
Singapore
Czech
Republic
37.359
34.225
32.672
34.675
36.155
33.705
35.637
34.297
54.787
57.499
58.825
56.694
54.756
56.859
54.881
56.167
92.146
91.724
91.497
91.369
90.911
90.564
90.518
90.464
Gold
Gold
Gold
Gold
Gold
Gold
Gold
Gold
37.075
52.870
89.945
Gold
34.364
33.936
55.495
55.744
89.859
89.680
Gold
Gold
32.145
57.498
89.643
Gold
Argentina
33.040
56.208
89.248
Gold
Korea
Lithuania
Chinese
Taipei
India
Turkey
Czech
Republic
Romania
38.236
35.372
50.908
53.531
89.144
88.903
Gold
Gold
30.608
58.148
88.756
Gold
31.867
34.469
56.568
53.941
88.435
88.410
Silver
Silver
38.050
50.110
88.160
Silver
33.496
54.506
88.002
Silver
70
Rank
37
38
39
40
41
42
Name
Florian Berger
Hiroki Uratani
Binh Nguyen Duc
Mehmet Cem Sahiner
Levindo Jose Garcia
Quarto
Jiraborrirak
Charoenpattarapreeda
43
Wei-Che Tsai
44
45
46
Attila Sveiczer
Tng Jia Hao Barry
Witold Hoffmann
47
Bo-Yun Gu
48
49
50
51
52
53
54
55
56
57
58
Manuel Eberl
Connie Zhao
Kornel Ocytko
Luca Zucchini
Richard Li
Won Jae Kim
Vranješević Filip
Marek Buchman
Ladislav Hovan
Nikunj Saunshi
Mads Bøttger Hansen
Mohammadreza
Amirmoshiri
59
60
Ondrej Henych
61
62
63
64
65
66
67
68
69
Fatih Alcicek
Anton Topchiy
Surendra Kotra
Kengo Kataoka
Brian Bi
Dominik Štefanko
Leonard Hasenclever
Khu Boon Hou Derek
Áron Szigetvári
70
Alexander Kochnev
71
Kirill Sukhoverkov
72
73
74
Rémi Olivier Patin
Cyril Tang
Richard Liu
Country
Germany
Japan
Viet Nam
Turkey
Practical
Theoretical
(max 40)
(max 60)
34.692
52.896
37.134
50.376
35.851
51.162
31.826
55.101
Total
(max 100)
87.588
87.510
87.013
86.927
Medal
Silver
Silver
Silver
Silver
Brazil
35.442
51.406
86.848
Silver
Thailand
33.781
52.948
86.729
Silver
28.048
58.533
86.581
Silver
35.159
29.352
37.719
51.263
56.506
47.940
86.422
85.858
85.659
Silver
Silver
Silver
28.458
57.024
85.482
Silver
31.505
34.218
29.314
31.910
32.825
26.245
31.562
36.790
26.526
31.751
32.564
53.510
50.427
55.273
52.658
51.612
58.029
52.554
46.699
56.881
51.551
50.456
85.015
84.645
84.587
84.568
84.437
84.274
84.116
83.489
83.407
83.302
83.020
Silver
Silver
Silver
Silver
Silver
Silver
Silver
Silver
Silver
Silver
Silver
28.281
54.594
82.875
Silver
30.600
52.228
82.828
Silver
31.316
28.337
31.216
30.076
25.268
29.098
33.312
28.460
27.137
51.438
54.377
51.485
52.587
57.325
53.463
49.048
53.804
54.896
82.754
82.714
82.701
82.663
82.593
82.561
82.360
82.264
82.033
Silver
Silver
Silver
Silver
Silver
Silver
Silver
Silver
Silver
31.034
50.230
81.264
Silver
28.307
52.838
81.145
Silver
28.322
26.653
27.240
52.806
54.041
53.180
81.128
80.694
80.420
Silver
Silver
Silver
Chinese
Taipei
Hungary
Singapore
Poland
Chinese
Taipei
Germany
Canada
Poland
Italy
United States
Korea
Croatia
Slovakia
Slovakia
India
Denmark
I. R. of Iran
Czech
Republic
Turkey
Ukraine
India
Japan
Canada
Slovakia
Germany
Singapore
Hungary
Russian
Federation
Russian
Federation
France
Australia
Canada
71
Rank
75
76
77
78
79
80
81
82
83
84
85
86
87
88
89
90
91
92
93
94
95
96
97
98
99
100
101
102
103
104
105
106
107
108
109
110
111
112
113
114
115
116
117
118
Name
Country
Practical
(max 40)
United
Kingdom
Quang Luu Nguyen Hong Viet Nam
Dzianis Kuliomin
Belarus
Dominykas Sedleckas
Lithuania
Jarkko Timo Olavi Järvelä Finland
Roberts Bluķis
Latvia
Hossein Dadashazar
I. R. of Iran
Vidmantas Bieliunas
Lithuania
Alimatun Nashira
Indonesia
Sergiy Shyshkanov
Ukraine
Yeoh Keat Hor
Malaysia
United
David Edey
Kingdom
Baptiste Couet
France
Hanieh Safari
I. R. of Iran
Kucanda Kristina
Croatia
Sebastian Gogg
Austria
Stewart Alexander
New Zealand
Stuart Ferrie
Australia
Kelvin Cheung
Australia
Utsarga Sikder
United States
Maciej Gryszel
Poland
Pablo Giomi
Spain
Tudor Balan
Romania
Maksim Mišin
Estonia
Lujia Xu
New Zealand
Emilis Bruzas
Lithuania
David Bellamy
New Zealand
Alexandru Sava
Romania
Abylay Shakhizadayev
Kazakhstan
Alain Vaucher
Switzerland
Ilya Skripin
Kazakhstan
Amarsanaa Davaasuren
Mongolia
Wepa Roziyev
Turkmenistan
Žiga Perko
Slovenia
Marcin Malinowski
Poland
Árni Johnsen
Iceland
Viktors Pozņaks
Latvia
Ioana Moga
Romania
Stephen Yuwono
Indonesia
Lizaveta Durovich
Belarus
Nejc Petek
Slovenia
David Ahlstrand
Sweden
Russian
Maxim Kozlov
Federation
Agung Hartoko
Indonesia
Joshua Stedman
72
Theoretical
(max 60)
Total
(max 100)
Medal
34.396
45.962
80.358
Silver
29.736
27.194
33.044
30.029
33.638
26.330
33.240
25.887
25.238
35.490
50.462
52.985
46.857
49.798
46.183
53.219
46.297
52.783
52.941
42.607
80.198
80.179
79.901
79.827
79.821
79.549
79.537
78.670
78.179
78.097
Silver
Silver
Silver
Silver
Silver
Silver
Silver
Silver
Silver
Silver
31.020
46.987
78.007
Silver
30.606
24.419
32.102
36.369
31.984
32.109
28.933
22.115
36.638
35.240
23.087
22.472
29.246
26.047
21.264
24.635
20.559
26.840
28.386
29.657
25.933
31.742
31.208
31.564
28.510
28.899
27.265
25.252
32.996
30.742
47.003
53.080
45.150
40.810
44.924
44.686
47.858
54.446
39.819
40.960
52.809
53.407
46.204
49.354
53.717
50.336
54.203
47.669
46.080
44.784
48.499
42.539
42.831
41.321
44.203
43.098
44.355
46.267
38.143
40.349
77.609
77.499
77.252
77.179
76.908
76.795
76.791
76.561
76.457
76.200
75.896
75.879
75.450
75.401
74.981
74.971
74.762
74.509
74.466
74.441
74.432
74.281
74.039
72.885
72.713
71.997
71.620
71.519
71.139
71.091
Silver
Silver
Silver
Silver
Bronze
Bronze
Bronze
Bronze
Bronze
Bronze
Bronze
Bronze
Bronze
Bronze
Bronze
Bronze
Bronze
Bronze
Bronze
Bronze
Bronze
Bronze
Bronze
Bronze
Bronze
Bronze
Bronze
Bronze
Bronze
Bronze
22.878
48.069
70.947
Bronze
29.761
41.069
70.830
Bronze
Rank
119
120
121
122
123
124
125
126
127
128
129
130
131
132
133
134
135
136
137
138
139
140
141
142
143
144
145
146
147
148
149
150
151
152
153
154
155
156
157
158
159
160
161
Name
Dmytro Frolov
Valter Bergant
Johannes Hellwagner
Zhalgas Serimbetov
Anandagopal Srinivasan
Ezequiel Maidanik
Ivan Jakovlev
Mikhail Kavalchuk
Tuan Le Anh
Cuc Mai Thu
Lukas Wagner
Alan Carrasco-Carballo
Michael Michelachvili
Yannick Suter
Konstantin Krautgasser
Markovic Igor
Christos Anastassiades
Makbule Esen
Alexander Blokhuis
Andre Silva Franco
Jessica Kazumi Okuma
Mario Rugiero
Agil Azimzada
Vasil Vasilev
Kadi Liis Saar
Country
Ukraine
Slovenia
Austria
Kazakhstan
Ireland
Argentina
Estonia
Belarus
Viet Nam
Viet Nam
Germany
Mexico
Israel
Switzerland
Austria
Croatia
Cyprus
Turkey
Netherlands
Brazil
Brazil
Argentina
Azerbaijan
Bulgaria
Estonia
United
David Wade
Kingdom
Eviatar Degani
Israel
Daniel Quill
Ireland
Ingrid Eidsvaag Andersen Norway
Anatolij Babič
Netherlands
Antton Curutchet
France
Cédric Martin
France
Istvan Kleijn
Netherlands
Rahym Ashirov
Turkmenistan
Andreu Tortajada Navarro Spain
Buiucli Serafim
Moldova
Allan Chau
Australia
Ivan Bojidarov Dimov
Bulgaria
Miras Bekbergenov
Kazakhstan
Jesús Alvaro Gómez
Spain
Iregui
Niels Christian Holm
Denmark
Sanden
Natallia Yelavik
Belarus
Amit Panghal
India
Practical
Theoretical
(max 40)
(max 60)
28.592
42.109
29.861
40.699
26.373
44.048
20.069
50.349
31.027
39.278
22.608
47.518
27.136
42.921
15.822
53.291
26.388
42.625
24.133
44.508
27.902
40.454
33.339
34.160
23.295
44.130
27.566
39.806
30.577
36.608
34.209
32.634
19.253
47.490
26.701
39.938
23.991
42.522
26.683
39.791
23.243
43.098
29.476
36.583
23.945
41.649
21.030
44.425
22.166
43.169
73
Total
(max 100)
70.701
70.560
70.421
70.418
70.305
70.126
70.057
69.113
69.013
68.641
68.356
67.499
67.425
67.372
67.185
66.843
66.743
66.639
66.513
66.474
66.341
66.059
65.594
65.455
65.335
Bronze
Bronze
Bronze
Bronze
Bronze
Bronze
Bronze
Bronze
Bronze
Bronze
Bronze
Bronze
Bronze
Bronze
Bronze
Bronze
Bronze
Bronze
Bronze
Bronze
Bronze
Bronze
Bronze
Bronze
Bronze
Medal
21.705
43.146
64.851
Bronze
18.723
25.620
21.411
27.621
24.863
27.489
28.170
23.978
28.172
23.548
24.806
17.944
13.523
46.103
38.960
43.151
36.604
38.851
36.174
35.416
39.043
34.775
38.468
37.204
43.952
48.162
64.826
64.580
64.562
64.225
63.714
63.663
63.586
63.021
62.947
62.016
62.010
61.896
61.685
Bronze
Bronze
Bronze
Bronze
Bronze
Bronze
Bronze
Bronze
Bronze
Bronze
Bronze
Bronze
Bronze
26.625
35.017
61.642
Bronze
32.001
29.387
61.388
Bronze
14.564
18.673
46.262
41.878
60.826
60.551
Bronze
Bronze
Rank
Name
162
163
Edvard Sargsyan
Rashad Yusifov
Matias Lanus Mendez
Elizalde
Vladyslav Panarin
Jari Tapio Huisman
Suvi Kaarina Klapuri
Kristian Holten Møller
Raymundo
Esquer-Rodriguez
Manuel Van Rijn
Jaimin Choi
Fani Georgieva
Madzharova
Alberto Branchi
Luciano Barluzzi
Oscar Salomon Kivinen
Raul Bruno Machado
Da Silva
Saidali Kholzoda
Ulugbek Barotov
Philip Sohn
Oscar Garcia Montero
Jorge Pedro Martins
Nogueiro
Marek Vician
Panayiota Katsamba
Božidar Aničić
Tania Lizeth Lopez-Silva
Gonçalo
Vitorino
Bonifácio
Oscar Hans Emil
Mickelin
Jakob Bank Kodal
Negrescu Dan
Nicholas Thong Li Jie
Espen Auseth Nielsen
Muhammad Anus
Enkhbat Myagmar
Tsvetan Hristov Tarnev
Lars Moen Strømsnes
Gantulga Batbayar
Dmitrijs Jevdokimovs
Michelle Frei
Jeroen Van Cleemput
Viktor Mattias Johansson
Myrat Annamuhammedov
164
165
166
167
168
169
170
171
172
173
174
175
176
177
178
179
180
181
182
183
184
185
186
187
188
189
190
191
192
193
194
195
196
197
198
199
200
201
Country
Armenia
Azerbaijan
Practical
Theoretical
(max 40)
(max 60)
19.759
40.778
19.282
40.972
Total
(max 100)
60.537
60.254
Bronze
Bronze
Medal
Argentina
28.846
31.143
59.989
Bronze
Ukraine
Finland
Finland
Denmark
20.531
27.185
21.119
28.252
39.427
32.628
38.478
31.131
59.958
59.813
59.597
59.383
Bronze
Bronze
Bronze
Bronze
Mexico
27.822
30.510
58.332
Bronze
Netherlands
New Zealand
30.004
16.652
28.276
41.287
58.280
57.939
Bronze
Bronze
Bulgaria
24.246
33.141
57.387
Bronze
Italy
Italy
Finland
19.601
19.940
23.536
37.763
37.328
33.401
57.364
57.268
56.937
Bronze
Bronze
Bronze
Brazil
15.058
41.575
56.633
Bronze
Tajikistan
Tajikistan
Canada
Costa Rica
27.467
19.231
21.525
26.869
28.231
36.129
33.774
28.382
55.698
55.360
55.299
55.251
Hon. Men.
Hon. Men.
Hon. Men.
Hon. Men.
Portugal
25.641
29.283
54.924
Hon. Men.
Slovakia
Cyprus
Slovenia
Mexico
26.645
19.094
17.734
28.779
27.989
35.488
35.736
24.433
54.634
54.582
53.470
53.212
Hon. Men.
Hon. Men.
Hon. Men.
Hon. Men.
Portugal
23.280
28.616
51.896
Sweden
24.427
27.460
51.887
Denmark
Moldova
Malaysia
Norway
Pakistan
Mongolia
Bulgaria
Norway
Mongolia
Latvia
Switzerland
Belgium
Sweden
Turkmenistan
21.848
23.042
27.963
14.857
15.738
14.034
27.009
22.364
14.559
22.926
25.562
19.678
22.186
16.227
29.859
28.656
23.543
36.647
35.550
36.873
23.634
28.159
35.491
26.460
23.113
28.280
25.238
30.875
51.707
51.698
51.506
51.504
51.288
50.907
50.643
50.523
50.050
49.386
48.675
47.958
47.424
47.102
74
Rank
202
203
204
205
206
207
208
209
210
211
212
213
214
215
216
217
218
219
220
221
222
223
224
225
226
227
228
229
230
231
232
233
234
235
236
237
238
Name
Maartje Iris Romijn
Dermot Gillen
María Victoria Moreno
Hernández
Selenge Enkhtuya
Anael Ben Asher
Alisher Rakhimov
Giuseppe Recchia
Oscar
Palomino-Hernandez
Izhar Ali
Artur Aslanyan
Marconi Nicolás Peñas
De Frutos
Rabi 'Atul Adibah
'Allauddin
Jonathan Wilson
Wainer Camacho Araya
Ramón Lorenzo Panades
Barrueto
Sigtryggur Kjartansson
Vahagn Tamazyan
Stelios Chatzimichail
Azizbek Usvaliev
Siti Fatma Hawaria
Mokhtar
Helgi Björnsson
Nikolaos Kaplaneris
Shakhboz Zulfaliev
Petricevic Fran
Tachmajal Corrales
Sanchez
Jānis Briška
Michele Oliosi
Konráð Þór Þorsteinsson
Stefanos Tyros
Vugar Mirzakhanov
Dosca Anastasia
Luis Fernando Merma
Paucar
Emil Marklund
Kevin Renier
Hafiz Hassan Ali
Marta Cristina Neves
Aguiar
Sebastian Andres
Martinez
Country
Practical
(max 40)
Theoretical
(max 60)
Total
(max 100)
Norway
Ireland
26.915
13.958
19.929
32.849
46.844
46.807
Venezuela
28.372
18.349
46.721
Mongolia
Israel
Tajikistan
Italy
15.819
15.432
12.790
22.544
30.674
30.969
33.581
23.801
46.493
46.401
46.371
46.345
Mexico
15.265
30.225
45.490
Pakistan
Armenia
23.060
12.814
22.422
32.651
45.482
45.465
Spain
18.577
26.624
45.201
Malaysia
20.074
25.122
45.196
Ireland
Costa Rica
21.229
25.459
23.144
18.244
44.373
43.703
5.726
37.909
43.635
Iceland
Armenia
Cyprus
Kyrgyzstan
21.445
9.655
10.225
11.956
21.955
32.873
32.241
30.472
43.400
42.528
42.466
42.428
Malaysia
22.344
19.877
42.221
Iceland
Greece
Tajikistan
Croatia
22.934
19.421
11.031
16.988
19.211
22.550
30.846
24.709
42.145
41.971
41.877
41.697
Costa Rica
21.569
18.601
40.170
Latvia
Switzerland
Iceland
Greece
Azerbaijan
Moldova
16.481
10.369
16.584
15.363
12.722
11.098
23.162
29.077
20.002
21.007
23.637
24.782
39.643
39.446
36.586
36.370
36.359
35.880
Peru
17.364
18.033
35.397
Sweden
Belgium
Pakistan
14.726
20.666
15.819
20.619
14.030
18.755
35.345
34.696
34.574
Portugal
10.597
23.867
34.464
Uruguay
17.120
17.080
34.200
Cuba
75
Medal
Rank
Name
239
Minahil Sana Qasim
Arnaldo Enmanuel Marin
Suárez
Michael Matalliotakis
Alexandre Faia Carvalho
Andreas Sofokli
Norberto Andres Canepa
Carlos Javier Berrio
Barrera
Erwin Wilfredo Mora
Flores
Georgios Papadimitriou
Mohammad Shubat
Jasmine De Becker
Agil Safaralizade
Melissa Bariani
Florence Thiry
Pîrău Tudor
Saltanat Mambetova
Sagynbek Dadybaev
Hessah Alquraishi
Alejandro Rodriguez
Anthony John Salcedo
Meza
Mohammad
Alabdulrazzaq
Begmyrat Cholukov
Davit Arzumanyan
Kalysbek Abykeshov
Shahad Albaloul
Mariam Aldarweesh
Rouaa Al Nan
Ali Issa
Ali Mourtada
240
241
242
243
244
245
246
247
248
249
250
251
252
253
254
255
256
257
258
259
260
261
262
263
264
265
266
267
Country
Practical
(max 40)
Theoretical
(max 60)
Total
(max 100)
Pakistan
13.884
20.228
34.112
Venezuela
15.059
18.476
33.535
Greece
Portugal
Cyprus
Uruguay
8.598
19.630
10.084
13.869
24.391
12.816
21.105
16.185
32.989
32.446
31.189
30.054
Venezuela
14.633
14.518
29.151
Venezuela
13.198
15.801
28.999
Greece
Syria
Belgium
Azerbaijan
Uruguay
Belgium
Moldova
Kyrgyzstan
Kyrgyzstan
Kuwait
Uruguay
11.878
19.962
13.838
5.720
12.638
14.404
8.024
11.268
6.764
15.570
8.658
17.111
8.382
14.221
22.150
14.959
13.146
19.433
14.719
18.642
9.612
15.200
28.989
28.344
28.059
27.870
27.597
27.550
27.457
25.987
25.406
25.182
23.858
7.419
14.847
22.266
12.529
9.033
21.562
4.591
4.193
1.950
8.368
5.114
4.947
0.000
3.520
14.162
12.372
12.798
6.330
6.304
1.712
5.850
1.221
18.753
16.565
14.748
14.698
11.418
6.659
5.850
4.741
Peru
Kuwait
Turkmenistan
Armenia
Kyrgyzstan
Kuwait
Kuwait
Syria
Syria
Syria
76
Medal
Statistical Analysis of the Problems
77
78
79
80
81
82
83
84
Minutes of the International Jury
Monday 19th – Tuesday 28th July 2010
(Recorded by Duckhwan Lee)
1. 1st International Jury Meeting(20:00-01:30 20th July at the Auditorium of OVTA)
- The Business section was chaired by Duckhwan Lee.
1) A brief summary of the Cambridge IChO was presented.
2) Liechtenstein, Serbia and Nigeria were introduced.
3) The members of the SC were introduced:
- Elected members: Vadim Eremin(Russia), Wolfang Hampe(Germany), Gabor
Magyarfalvi(Hungary, not present for personal reason), John Kiappes(USA), Mark
Ellison(Australia), Duckhwan Lee(Korea)
- Organizers: Peter Wothers (UK, 2009), Tadashi Watanabe [Ito Masato] (Japan, 2010), Jale
Hacaloglu(Turkey, 2011), G. Bryan Balazs(USA, 2012)
- Coopted members: Carlos Castro-Acuna(Mexico), Manfred Kerschbaumer(Austria), Anton
Sirota (Slovakia)
* The underlined members are up for re-election at the closing of the 42st IChO
4) The schedule for the SC meetings was announced.
5) The discussions on the practical tasks were chaired by Professor Sugahara.
2. The SC Meeting (13:00-14:30 Tuesday 21st July at Rm 3019 of OVTA)
- Future hosts
9 Finalized: Turkey(2011), USA(2012), Vietnam(2014)
9 To be finalized at the December meeting: Russia(2013)
- Observing countries:
9 Liechtenstein: will participate with students in 2011
9 Nigeria: will have to observe IChO-2011 [§ 2(3)… Incoming countries must send observers to
two consecutive Olympiads before its pupils can participate in IChO (see also § 3, section 5).]
- Request of Uzbekistan for appointing Vadim as their observer
9 Voted not to accept the request since it would be a direct violation of the Regulation.
9 A letter will be sent to the Government of Uzbekistan.
- Election Procedure
9 The position of Gabor will be replaced by a new member with two-year term.
9 The election procedure for Europe(3 positions) and Americas(1 position) will be announced at
the beginning of the 2nd International Jury Meeting.
9 A form for the listing of candidates will be posted in front of the dining room until 20:00 on
Friday.
9 Each candidate will have a minute to announce his/her candidacy at the 3rd International Jury.
85
9 Each head mentor will write on the prepared ballot the names of that country’s selected
candidates, but no more than the number of open positions.
- Miscellaneous
9 A printed version of the proposals for revision of the Regulation will be distributed before the
3rd International Jury Meeting.
9 The co-opted member should not be selected solely on the basis of his/her native languages.
9 We may need some sort of guidelines in order to resolve issues with registration rates arising
from the constantly changing exchange rates.
3. 2nd International Jury Meeting (20:00-00:30 Sunday 22nd July at the Auditorium of OVTA)
- The election procedures were announced.
- The revision procedures were announced and the printed copy of the proposals for revision were
distributed to the head mentors.
4. 3rd International Jury Meeting (20:00-21:30 Sunday 25th July at the Auditorium of OVTA)
The business session was chaired by Duckhwan Lee.
1) Announcement of the arbitration schedule.
2) Future hosts:
¾ The recommendation for Vietnam as the host of IChO-2014 was approved unanimously.
¾ O. Yavuz Ataman and Bryan Balazs introduced the plans for IChO-2011 and 2012.
¾ Vadim explained the Russian plan for presenting the official document for the December SC.
¾ Spain (Juan Antonio) is working for the IChO-2015.
3) Election of SC members
¾ Sasha Gladilin(Russia), Peter Wothers(UK), Wolfgang Hampe(Germany), and John
Kiappes(USA) were elected for Europe and Americas through secret vote.
4) Revision of the IChO regulations
¾ The proposal prepared by the December SC meeting was distributed through the Prep. Problems
as well as the hand-outs at the beginning of the Tokyo Olympiad.
¾ The Proposal A and C were approved without discussion by qualified majority of 59 votes out
of 68.
¾ The Proposal B was approved as presented by qualified majority of 59 votes out of 68.
5) The elected members and the hosts (Japan, Turkey, USA) re-elected Duckhwan Lee(Korea) as the
Chair. The Co-opted member will be decided before the 4th Jury with the recommendation of the
Turkey host.
86
5. 4th International Jury Meeting (20:00-21:00 Monday 26th July at the Auditorium of OVTA)
The business session was chaired by Duckhwan Lee.
1) Steering Committee (August 2010 – July 2011).
- elected members:
Wolfang Hampe(Germany, 2008, [email protected])
Peter Wothers (UK, 2010, [email protected])
Sasha Gladilin(Russia, 2010, [email protected])
John Kiappes(USA, 2008, [email protected])
Mark Ellison(Australia, 2007, [email protected])
Duckhwan Lee(Korea, 2007, [email protected], Chair)
- Organizers:
Tadashi Watanabe (Japan, 2010, [email protected])
Jale Hacaloglu(Turkey, 2011, [email protected])
G. Bryan Balazs (USA, 2012, [email protected])
Representative of IChO-2014(Vietnam)
- Coopted members:
Manfred Kerschbaumer(Austria, [email protected])
Carlos Castro-Acuna(Mexico, [email protected])
Olivier Plaidy(France, [email protected])
2) IUPAC Travel Fund
Country
2010
Venezuela
Croatia
Tajikistan
Argentina
2009
Uruguay
Peru
Cuba
2008
Cuba
Kyrgystan
Peru
Tajikistan
Uruguay
Venezuela
Participation Fee
Travel Expenses
Total
$ 2000
$ 2400
$ 2853
$ 1800
$ 2000
$ 3100
$ 2853
$ 1800
$ 700
$ 1100
$ 600
$ 1700
$ 1800
$ 1600
$ 900
$ 500
$ 1200
$ 900
$ 1300
$ 1150
$ 1000
$ 1000
$ 900
$ 2900
$ 600
$ 3000
$ 2800
$ 1740
$ 500
$ 1150
$ 2900
$ 900
9 It was explained that the support for travel expenses should not be more than the actual fares for
travel for which clear receipts such as air fare tickets should be submitted to the Organizer.
87
9 The support should be used for paying the participation fee and/or the actual travel expenses for
one extra student or mentor, never for observer or guest.
9 The request for support must be submitted to the Ankara Organizer no later than the end of
November.
9 The contact information for the IUPAC will be delivered to Jale from Ito.
3) Miscellaneous
9 A list of participants with e-mail addresses would be helpful. (Sasha)
9 We need a clear guideline for the number of significant figures in theoretical tasks. (Wolfgang)
9 There was an appeal by the Nigerian observer that they had experienced difficulties in
observing last two year even though they have had National Olympiad in Nigeria, and that they
really want to participate with their students. (Sunday Adedeji, the observer of Nigeria)
4) Report of Arbitration and Medal Allocation
9 Postponed to Jul 27 due to the mix-ups in the marking of the final results.
9 The Science Committee was informed that they have to stick to the markings that the HM
signed.
9 Plan for July 27
7:00 Breakfast
7:45 Distribution of the Markings
8:00 5th Jury Meeting for the Report and Medal Allocation
6. 5th International Jury Meeting (8:00-9:00 Tuesday 27th July at the Auditorium of OVTA)
1) Report on Arbitration (Professor Onaka)
9 The double penalty cases have been taken care of after the arbitration for 9.4% of the students.
2) Medal Allocation (Professor Sugahara)
9 The total number of students was 267.
9 32 (11.6%) for Gold, 58(21.0%) for Silver, 86(31.2%) for Bronze, 9 for Honorable Mention
3) On request by Peter, a letter, signed by all members of SC, explaining the need for the second year
of observation by the Regulation, was delivered to the Nigeria Observer before they can send
students to participate in the Olympiad..
88
Regulations of the International Chemistry Olympiad (IChO)
General Statement
§ 1 Aims of the competition
The International Chemistry Olympiad (IChO) is a chemistry competition for students at
secondary school level with the aim of promoting international contacts in chemistry. It is
intended to stimulate the activities of students interested in chemistry by way of the
independent and creative solution of chemical problems. The IChO competitions help to
facilitate cordial relations between young adults of different nationalities; they encourage
cooperation and international understanding.
Organization of the IChO
§ 2 Organization and invitation
(1) The IChO is organized every year, as a rule at the beginning of July, in one of the
participating countries by the Education Ministry or an appropriate institution of the
organizing country (hereafter referred to as the organizer).
(2) Unless directed otherwise by the International Jury, the organizer is obliged to invite all
countries that participated in the preceding IChO competition. The official invitation to
participate in the forthcoming IChO should be sent to countries by the November
preceding the competition. The countries invited must confirm their participation in the
IChO according to requirements of the organizer.
(3) Countries that wish to take part in the IChO must apply to the organizer by the end of
November preceding the Olympiad. The organizer has the right to invite the countries
only with the agreement of the organizers of the two subsequent IChO competitions.
Invited countries must send an observer to two consecutive Olympiads before its
pupils can participate in the IChO.
§ 3 Delegations
(1) Each participating country's delegation consists of competitors and accompanying
persons (also known as mentors). It is expected that there are four competitors and
two mentors in the delegation. Furthermore, the countries may include two scientific
observers as part of their delegation.
(2) The competitors must not be university students. They can only be students of
secondary schools that are not specialized in chemistry and, if they have already
graduated before the 1st of May of the year of the competition, the organizer must be
informed as to the month and year of their graduation. Moreover, they must be under
the 20 years of age on the 1st of July of the year of the competition.
The competitors must be passport holders of the country they represent or have taken
part in the secondary school educational system of this country for more than one
academic year.
All members of a delegation must provide themselves with medical insurance for the
journey to and from the organizing country and for the period of their stay in the
organizing country.
(3) The mentors act as members of the International Jury (see § 6). One of the mentors is
designated as the head of delegation (head mentor).
89
(4) The mentors:
a) must guarantee the fulfillment of those conditions specified in section 2 of this
paragraph,
b) must be capable of translating the text of the competition tasks from English into
the language used by their students and be able to judge the set of tasks and
correct the work of the students.
c) have the right to enter a protest which should be addressed to the Chair of the
International Jury or the Steering Committee and, when necessary, ask for a
resolution of the problem at the next meeting of the International Jury.
§ 4 Obligations of the Organizer
(1) The organizer provides:
a) the itinerary of the IChO,
b) transportation from/to an airport/station (which is designated by the host country)
on the day of arrival and departure,
c) that the organization of the competition will adhere to the regulations,
d) accident insurance for all participants in connection with the itinerary,
e) the opportunity for the mentors to inspect the working room and practical apparatus
to be used for the practical tasks before the competition takes place,
f) all necessary arrangements for the observance of safety regulations,
g) the medals, certificates and prizes, which are presented at the official closing
ceremony,
h) a report on the competition to be distributed not later than six months after the
competition.
(2) A meeting of the Steering Committee must be hosted in the organizing country in the
December prior to the IChO. The organizing country will provide some travel
assistance.
§ 5 Financing
(1) The participating country covers the return travel costs of the students and the
accompanying persons to the designated airport/station or to the location where the
competition is held.
(2) Participating countries must pay a participation fee, the amount of which must be
approved by the International Jury.
(3) All other costs incurred in connection with the organized program, including the costs
of accommodation for all competitors and members of the International Jury, are
covered by the organizer.
(4) The organizers of the next two consecutive Olympiads may send two observers to the
current IChO with their expenses covered by the host as mentioned in § 5, section 3.
Institutions of the IChO
§ 6 International Jury
(1) The International Jury consists of its chair and members. The chair of the International
Jury is nominated by the organizer. The members of the International Jury are the two
mentors from the individual delegations and the chair of the Steering Committee (see §
8).
90
(2) The chair of the International Jury or his/her delegate calls and chairs the meetings of
the International Jury concerning the current competition, while the business sessions
concerning general problems of the IChO are chaired by the SC chair.
(3) Resolutions of common International Jury sessions or its split sessions are passed by
the International Jury when they are agreed by a simple majority of votes in the
presence of at least 75% of the delegations. Each participating country has one vote.
Changes in the regulations can only be made at the common sessions of the
International Jury and require a qualified majority of two thirds of the votes. The chair
has a casting vote in the event of a tie. The decisions of the International Jury are
binding for both organizer and participants.
(4) The working language of the International Jury is English.
§ 7 Responsibilities of the International Jury
(1) The International Jury:
a) is in charge of the actual competition and its supervision according to the
regulations,
b) discusses in advance the competition tasks presented by the organizer, their
solutions and the marking guidelines, gives comments and takes decisions in case
of changes,
c) supervises the marking of the examination papers and guarantees that all
participants are judged by equal criteria,
d) determines the winners and decides on prizes for the competitors,
e) monitors the competition and suggests changes to the regulations, organization
and contents for future IChOs,
f) makes decisions on the exclusion of a participant or an entire team from the
competition (see also § 11, section 7),
g) elects members of the Steering Committee of the IChO,
h) may form working groups to solve specific chemistry related problems of the IChO.
(2) The members of the International Jury:
a) are obliged to maintain a professional discretion about any relevant information
they receive during the IChO and must not assist any participants,
b) keep the marking and results secret until announced by the International Jury.
§ 8 Steering Committee
(1) The long term work involved in organizing the International Chemistry Olympiads is
coordinated by the Steering Committee.
(2) Members of the Committee are elected by the International Jury by a secret ballot to
serve a two year term. There must be at least one person from each of the following
regions: the Americas, Asia and Europe. Other three members can come from any
region. The term of the elected committee begins on the 1st day after the IChO.
Members are elected for no more than two consecutive terms.
(3) There are the following ex-officio members of the Steering Committee:
a) a representative of the current IChO,
b) a representative of the immediately preceding IChO,
c) representatives of the subsequent two IChOs,
d) the immediate past chair of the SC (for one year only)
(4) The incoming Steering Committee elects its own Chair from among its elected
members at a meeting held before the committee’s term begins.
91
The Chair:
a) calls and chairs the meetings of the Steering Committee,
b) calls and chairs the business meetings of the International Jury dealing with
general problems of future International Chemistry Olympiads,
c) may invite non-voting guests to the meetings of the Steering Committee after
consultation with the host of the meeting,
d) has the right to call extraordinary meetings of the International Jury when
necessary.
(5) The Steering Committee:
a) provides organizational oversight for the International Chemistry Olympiad,
b) proposes items for consideration at the International Jury sessions.
c) may co-opt 1–3 non-voting members for their particular expertise for periods of one
year.
d) may invite representatives of confirmed future IChOs.
(6) The Steering Committee is not empowered to make any decisions affecting the
International Chemistry Olympiad that would interfere with the duties and
responsibilities of the International Jury (see § 6 and 7).
§ 9 International Information Center
There is an International Information Center of the International Chemistry Olympiads
gathering and providing (when necessary) all the documentation of the IChOs from the
beginning of the Olympiad to the present. The seat of the Office is in Bratislava, Slovakia.
Competition
§ 10 Preparation for the IChO competition
(1) The organizer distributes a set of preparatory tasks written in English to all participating
countries in January of the competition year. The preparatory tasks are intended to
give students a good idea of the type and difficulty of the competition tasks, including
safety aspects (see §12 and Appendix “B”). SI units should be used throughout the
preparatory tasks.
(2) The total number of theoretical and experimental tasks in the set of preparatory
problems cannot be lower than 25 and 5, respectively.
(3) Appendix C of the regulations contains a list of concepts and skills expected to be
mastered by the participants. Organizers may freely include questions and tasks in the
theoretical or experimental competition based on the knowledge listed there.
The organizer can include problems in the exams based on the use of concepts and
skills from not more than 6 theoretical and 2 practical fields outside this list, if a
minimum of 2 tasks from each field is included and the necessary skills demonstrated
in the set of preparatory problems. Examples of such external fields are listed in
Appendix C. Fields not already listed should have a breadth similar to the examples.
These 6 theoretical and 2 practical fields must be stated explicitly at the beginning of
the Preparatory problems. If an equation not covered by the listed fields is required for
the solution of the exam questions, then this should be defined in the exam text.
(4) Appendix D contains an outline of the factual knowledge supposedly familiar to the
competitors. If specific facts, not included in Appendix D, are required for the solution
of the exam questions, then these should be included in the exam text or in the
preparatory problems and their solutions.
92
(5) Training or any other special instruction, that is carried out for a selected group of 50 or
fewer students, containing the IChO team, must be no longer than two weeks.
§ 11 Organization of the IChO Competition
(1) The competition consists of two parts:
a) part one, the practical (experimental) competition,
b) part two, the theoretical competition.
(2) A working time of four to five hours is allotted for each part. There is at least one day of
rest between the two parts.
(3) Competitors receive all relevant information in the language of their choice.
(4) There must be no contact between mentors and competitors once the mentors have
received the competition tasks for consideration. Information regarding the competition
tasks must not be passed to the competitors directly or indirectly prior or during the
competition.
(5) When pocket calculators are not provided by the organizer, only non-programmable
pocket calculators may be used in the competition.
(6) The safety regulations announced by the organizer are binding for all participants.
(7) Breaking of any of the rules given in the preceding paragraphs (§ 3. section 2, § 10
section 5, § 11 sections 4, 5, and 6) has as its consequence exclusion from the whole
or a part of the competition.
§ 12 Safety
(1) During the experimental part, the competitors must wear laboratory coats and eye
protection. The competitors are expected to bring their own laboratory coats. Other
means of protection for laboratory work are provided by the organizer.
(2) When handling liquids, each student must be provided with a pipette ball or filler.
Pipetting by mouth is strictly forbidden.
(3) The use of very toxic substances (designation T+) is strictly forbidden. The use of toxic
substances (designation T) is not recommended, but may be allowed if special
precautions are taken. Substances belonging to the categories R 45, R 46, R 47 must
not be used under any circumstances (see Appendix B for definitions of these
categories).
(4) Detailed recommendations involving students´ safety and the handling and disposal of
chemicals can be found in Appendices A 1, A 2, and B. These appendices are based
on the directives of the European Communities and are updated automatically with
these directives.
a) Appendix A 1: Safety Rules for Students in the laboratory.
b) Appendix A 2: Safety Rules and Recommendations for the Host Country of the
IChO.
c) Appendix B contains:
B 1: Hazard Warning Symbols and Hazard Designations;
B 2: R-Ratings and S-Provisions: Nature of special risks (R) and safety advice (S);
B 3: Explanation of Danger Symbols (for use of chemicals in schools);
93
§ 13 Competition Tasks
(1) The organizer is responsible for the preparation of competition tasks by competent
experts/authors, who constitute the Scientific Board of the IChO. They propose the
methods of solution and the marking scheme.
(2) The tasks, their solutions and the marking schemes are submitted to the International
Jury for consideration and approval. The authors of the tasks should be present during
the discussion.
(3) The Chair of the International Jury may put the Chair of the Scientific Board in charge
of the proceedings when the tasks are considered.
(4) The total length of the theoretical or experimental tasks, including answer sheets,
should be kept to a minimum and not exceed 25,000 characters. The number of
characters must be stated at the end of each exam paper. SI units should be used
throughout the competition tasks.
(5) In the experimental part of the competition the following conditions must be fulfilled:
a) The experimental part must contain at least two independent tasks.
b) The marking cannot require subjective interpretation by the staff.
c) Competitors must receive the same substances when solving the tasks from
qualitative analytical chemistry.
d) When solving tasks from quantitative analytical chemistry competitors must receive
the same substances but with different concentrations.
e) In evaluating the quantitative tasks the master values must not be based on an
average of the results of the competitors.
f) The great majority of the grade in quantitative tasks must be given to the mean
value as reported by the competitors while some marks may also be given to the
corresponding equations, calculations, or explanations directly related to the work.
Points must not be awarded for reproducibility.
§ 14 Correcting and Marking
(1) A maximum of 60 points is allocated to the theoretical tasks and 40 points to the
practical tasks, making a total of 100 points.
(2) The competition tasks are corrected independently by the authors and by the mentors.
Consequential marking should be used so that students are not punished twice for the
same error. Both corrections are then compared; however, the authors present their
evaluation first. After a discussion the final score for each participant is reached and
agreed by both sides. The organizer retains the original marked manuscripts.
(3) The International Jury discusses the results and decides on the final scores.
(4) In order to eliminate any doubts about possible mistakes in the processing of the
results the organizer must provide the mentors with a list of their students’ total results
before the closing award ceremony.
§ 15 Results and Prizes
(1) Official results of the competition and the number of medals awarded are decided by
the International Jury.
(2) The number of gold medals awarded is in the range of 8% to 12%, silver 18% to 22%,
and bronze medals 28% to 32% of the total number of competitors. The exact number
of medals is decided on the basis of an blind review of the results.
94
(3) Each medalist must receive the medal and a corresponding certificate from the
organizer.
(4) In addition to the medals other prizes may be awarded.
(5) An honorable mention is awarded to competitors who are among the best 10% of non
medalists.
(6) Each competitor receives a certificate of participation.
(7) In the awarding ceremony, the non-medalists are called alphabetically.
(8) Team classification is not made.
(9) The organizer must provide a complete list of results as a part of the final report.
§ 16 Final Regulations
(1) Those who take part in the competition acknowledge these regulations through their
participation.
(2) This version of regulations has been approved by the International Jury in Tokyo
(Japan) in July 2010, and is issued to replace the former regulations approved in
Budapest (Hungary) in July 2008.
(3) The regulations are valid from the 1st of September, 2010. Changes to the regulations
can be made only by the International Jury and require a qualified majority (two third of
the votes with regard to total number of participating countries).
95
APPENDIX A
A 1: SAFETY RULES FOR STUDENTS IN THE LABORATORY
All students of chemistry must recognize that hazardous materials cannot be completely
avoided. Chemists must learn to handle all materials in an appropriate fashion. While it is
not expected that all students participating in the International Chemistry Olympiad know
the hazards of every chemical, the organizers of the competition will assume that all
participating students know the basic safety procedures. For example, the organizers will
assume that students know that eating, drinking or smoking in the laboratory or tasting a
chemical is strictly forbidden.
In addition to the common-sense safety considerations to which students should have
been previously exposed, some specific rules, listed below, must also be followed during
the Olympiad. If any question arises concerning safety procedures during the practical
exam, the student should not hesitate to ask the nearest supervisor for direction.
Rules regarding personal protection
1. Eye protection must be worn in the laboratories at all times. If the student wears
contact lenses, full protection goggles must also be worn. Eye protection will be
provided by the host country.
2. A laboratory coat is required. Each student will supply this item for himself/herself.
3. Long pants and closed-toed shoes are recommended for individual safety. Long hair
and loose clothing should be confined.
4. Pipetting by mouth is strictly forbidden. Each student must be provided with a pipette
bulb or pipette filler.
Rules for Handling Materials
1. Specific instructions for handling hazardous materials will be included by the host
country in the procedures of the practical exam. All potentially dangerous materials will
be labeled using the international symbols below. Each student is responsible for
recognizing these symbols and knowing their meaning (see Appendix B 1, B 2 and B
3).
2. Do not indiscriminately dispose chemicals in the sink. Follow all disposal rules
provided by the host country.
A 2: SAFETY RULES AND RECOMMENDATIONS FOR THE HOST
COUNTRY OF THE INTERNATIONAL CHEMISTRY OLYMPIAD
Certainly it can be assumed that all students participating in the IChO have at least modest
experience with safety laboratory procedures. However, it is the responsibility of the
International Jury and the organizing country to be sure that the welfare of the students is
carefully considered. Reference to the Safety Rules for Students in the Laboratory will
show that the students carry some of the burden for their own safety. Other safety matters
will vary from year to year, depending on practical tasks. The organizers of these tasks for
the host country are therefore assigned responsibility in the areas listed below. The
organizers are advised to carefully test the practical tasks in advance to ensure the safety
of the experiments. This can best be accomplished by having students of ability similar to
that of IChO participants carry out the testing.
96
Rules for the Host Country (see also A 1):
1. Emergency first-aid treatment should be available during the practical examination.
2. Students must be informed about the proper methods of handling hazardous materials.
a) Specific techniques for handling each hazardous substance should be included in
the written instructions of the practical examination.
b) All bottles (containers) containing hazardous substances must be appropriately
labeled using internationally recognized symbols (see Appendix B 1).
3. Chemical disposal instructions should be provided to the students within the written
instructions of the practical examination. Waste collection containers should be used
for the chemicals considered hazardous to the environment.
4. The practical tasks should be designed for appropriate (in other words, minimum)
quantities of materials.
5. The laboratory facilities should be chosen with the following in mind:
a) Each student should not only have adequate space in which to work, but should be
in safe distance from other students.
b) There should be adequate ventilation in the rooms and a sufficient number of
hoods when needed.
c) There should be more than one emergency exit for each room.
d) Fire extinguishers should be near by.
e) Electrical equipment should be situated in an appropriate spot and be of a safe
nature.
f) There should be appropriate equipment available for clean-up of spills.
6. It is recommended that one supervisor be available for every four students in the
laboratory to adequately ensure safe conditions.
7. The organizers should follow international guidelines for the use of toxic, hazardous or
carcinogenic substances in the IChO.
97
APPENDIX B
B 1: HAZARD WARNING SYMBOLS AND HAZARD DESIGNATIONS AND
THEIR EXPLANATION (Applied for Chemicals in Schools)
1. Explosive substances (E)
These are substances which can be caused to explode by exposure to a flame or which
are more sensitive to impact of friction than 1,3-dinitrobenzene (e.g. picrates, organic
peroxides). In particular they include substances with R ratings R1 - R3 (see B 2),
designation E.
When using and storing these substances, the S provisions (S15 - S17) must be observed
(see B 2).
2. Fire inducing substances, Oxidizing (O)
These are substances which can have a strong exothermic reaction on coming into
contact with other, particularly flammable substances or organic peroxides. They include in
particular substances R 7 to R 9, designation O.
3. Highly flammable, easily flammable and flammable substances (F+, F)
In liquid form, highly flammable substances have an ignition point below 0 °C and a boiling
point of 35 °C maximum. They are to be designated by the danger symbol F+ and the
rating R 12.
Substances are easily flammable if they:
a) can heat up and ignite at normal air temperature without energy supply,
b) are easily ignited in solid state by short exposure to a source of flammation and
continue to burn or glow after removal of the latter,
c) ignite below 21 °C in liquid state,
d) ignite in gaseous state if mixed with air at 101.3 kPa and 20 °C,
e) develop easily flammable gases in dangerous quantities when in contact with water
or damp air,
f) ignite if brought into contact with air when in dustlike state.
These substances are to be designated with the danger symbol F and the rating R 11.
Flammable substances have in liquid form an ignition point of 21 °C to 55 °C and are to
designated with the rating R 10, no danger symbol.
When dealing with highly flammable, easily flammable and flammable liquids may only be
heated using sealed electrical heating equipment which is not in itself a source of
flammation. All substances must be heated in such a way that the dangerous vapours
liberated by heating cannot escape into the atmosphere. This does not apply to fire
hazardous substances in small quantities for fire demonstrations.
The regulations laid down by the state fire authorities must be observed.
4. Toxic substances (T +, T, Xn )
Legislation applying to chemicals distinguishes three categories of toxicants:
highly toxic substances (R 26 R 28), danger symbol T+,
toxic substances (R 23 R 25), danger symbol T,
less toxic substances (R 20 R 22), danger symbol Xn.
98
Highly toxic substances are those which can cause grave acute or chronic health damage
or death almost immediately if inhaled, swallowed or absorbed through the skin in small
amounts.
Toxic substances are those which can cause considerable acute or chronic health damage
or death if inhaled, swallowed or absorbed through the skin in small amounts.
Less toxic substances (noxious substances) are those which can cause restricted health
damage if inhaled, swallowed or absorbed through the skin.
If highly toxic or toxic substances are produced in the course of an experiment (e.g.
chlorine, hydrogen sulfide), these may only be produced in the quantities necessary for the
experiment. in the case of volatile substances, the experiment must be conducted under a
hood where the gas can be drawn off. Residue must be appropriately disposed of after the
experiment and may on no account be stored. If the facilities for disposal are not available,
the experiment may not be conducted.
Less toxic substances and preparations may be obtained without a permit. Less toxic
substances are also those which contain a highly toxic or toxic substance at a level of
concentration below that determined by law as the maximum for classification as noxious.
Chlorine water, bromine water and hydrogen sulfide solution in a concentration of up to 1%
may therefore be used in instruction.
5. Corrosives and irritants (C, X i )
Caustic or corrosive substances (R 34, R 35), designation C, are those which can destroy
living materials by their action upon it. Substances are classed as irritants (R 36 R 38),
designation Xi, if they cause inflammation without being corrosive on direct, prolonged
or repeated contact with the skin or mucous membranes. The relevant safety
recommendations (S 22 S 28) should be observed.
6. Carcinogenic, genotype or embryo damaging, chronically harmful
substances
Substances may not be used for instruction if they have a proven carcinogenic effect (R
45), if they cause hereditary damage (R 46) or embryo damage (R 47), or if they are
chronically damaging (R 48), particularly those substances classed as unmistakably
carcinogenic. Such substances must be removed from all school stocks. Storage is not
permitted under any circumstances.
Further, substances for which there is a well founded suspicion of carcinogenic potential
(R 40) may only be used if corresponding safety precautions are taken and only in such
cases where they cannot be replaced by less dangerous chemicals.
B 2: R RATINGS AND S PROVISIONS
Nature of special risks (R)
R1
R2
R3
R4
R5
R6
R7
R8
R9
Explosive when dry.
Risk of explosion by shock, friction, fire or other sources of ignition.
Extreme risk of explosion by shock, friction, fire or other sources of ignition.
Forms very sensitive explosive metallic compounds.
Heating may cause an explosion.
Explosive with or without contact with air.
May cause fire.
Contact with combustible material may cause fire.
Explosive when mixed with combustible material.
99
R 10
R 11
R 12
R 13
R 14
R 15
R 16
R 17
R 18
R 19
R 20
R 21
R 22
R 23
R 24
R 25
R 26
R 27
R 28
R 29
R 30
R 31
R 32
R 33
R 34
R 35
R 36
R 37
R 38
R 39
R 40
R 41
R 42
R 43
R 44
R 45
R 46
R 47
R 48
Flammable.
Highly flammable.
Extremely flammable.
Extremely flammable liquefied gas.
Reacts violently with water.
Contact with water liberates highly flammable gases.
Explosive when mixed with oxidizing substances.
Spontaneously flammable in air.
In use, may form flammable/explosive vapour air mixture.
May form explosive peroxides.
Harmful by inhalation.
Harmful in contact with skin.
Harmful if swallowed.
Toxic by inhalation.
Toxic in contact with skin.
Toxic if swallowed.
Very toxic by inhalation.
Very toxic in contact with skin.
Very toxic if swallowed.
Contact with water liberates toxic gas.
Can become highly flammable in use.
Contact with acids liberates toxic gas.
Contact with acids liberates very toxic gas.
Danger of cumulative effects.
Causes burns.
Causes severe burns.
Irritating to eyes.
Irritating to respiratory system.
Irritating to skin.
Danger of very serious irreversible effects.
Possible risks of irreversible effects.
Danger of serious eye damage.
May cause sensitization by inhalation.
May cause sensitization by skin contact.
Risk of explosion if heated by occlusion.
May cause cancer.
May cause hereditary damage.
May cause embryo damage.
Danger of chronic damage.
Safety advice (S)
S1
S2
S3
S4
S5
S6
S7
S8
Keep locked up.
Keep out of reach of children.
Keep in a cool place.
Keep away from living quarters.
Keep contents under .... (appropriate liquid to be specified by the manufacturer).
Keep under .... (inert gas to be specified by the manufacturer).
Keep container tightly closed.
Keep container dry.
100
S9
S 10
S 11
S 12
S 13
S 14
S 15
S 16
S 17
S 18
S 20
S 21
S 22
S 23
S 24
S 25
S 26
Keep container in a well ventilated place.
Keep contents wet.
Avoid contact with air.
Do not keep the container sealed.
Keep away from food, drink and animal feeding stuffs.
Keep away from .... (incompatible materials to be indicated by the manufacturer).
Keep away from heat.
Keep away from sources of ignition No smoking.
Keep away from combustible materials.
Handle and open container with care.
When using do not eat or drink.
When using do not smoke.
Do not inhale dust.
Do not inhale gas/fumes/vapour/spray.
Avoid contact with skin.
Avoid contact with eyes.
In case of contact with eyes, rinse immediately with plenty of water and seek
medical advice.
S 27 Take off immediately all contaminated clothing.
S 28 After contact with skin, wash immediately with plenty of .... (to be specified by the
manufacturer).
S 29 Do not empty into drains.
S 30 Never add water to this product.
S 31 Keep away from explosive materials.
S 33 Take precautionary measures against static discharges.
S 34 Avoid shock and friction.
S 35 This material and its container must be disposed of in a safe way.
S 36 Wear suitable protective clothing.
S 37 Wear suitable gloves.
S 38 In case of insufficient ventilation, wear suitable respiratory equipment.
S 39 Wear eye/face protection.
S 40 To clean the floor and all objects contaminated by this material, use .... (to be
specified by the manufacturer).
S 41 In case of fire and/or explosion do not breathe fumes.
S 42 During fumigation/spraying wear suitable respiratory equipment.
S 43 In case of fire, use .... (indicate in space the precise type of fire fighting equipment.
If water increases the risk, add Never use water).
S 44 If you feel unwell, seek medical advice (show the label where possible).
S 45 In case of accident or if you feel unwell, seek medical advice (show the label a
where
101
B 3: EXPLANATION OF DANGER SYMBOLS
toxic (T) substances
flammable (F) substances
irritating (Xi) substances
and
and
and
very toxic (T+) substances extremely flammable (F+) substances harmful (Xn) substances
explosive (E) substances
oxidizing (O) substances
corrosive (C) substances
environmentally
dangerous (N) substances
102
Appendix C
Concepts and skills expected to be known by all participants:
(predominantly equivalent to former number 1 and 2 topics)
Concepts
Awareness of experimental errors, use of significant figures;
Maths skills commonly encountered at secondary school level, including solving quadratic
equations, use of logarithms and exponentials, solving simultaneous equations with 2
unknowns, the meaning of sine and cosine, elementary geometry such as Pythagoras’
theorem, plotting graphs
(more advanced maths skills such as differentiation and integration, if required must be
included as one of the advanced topics)
Nucleons, isotopes, radioactive decay and nuclear reactions (alpha, beta, gamma);
Quantum numbers (n,l,m) and orbitals (s,p,d) in hydrogen-like atoms;
Hund’s rule, Pauli exclusion principle;
Electronic configuration of main group and the first row transition metal atoms and their
ions;
Periodic table and trends (electronegativity, electron affinity, ionization energy, atomic and
ionic size, melting points, metallic character, reactivity);
Bond types (covalent, ionic, metallic), intermolecular forces and relation to properties;
Molecular structures and simple VSEPR theory (up to 4 electron pairs);
Balancing equations, empirical formulae, mole concept and Avogadro constant,
stoichiometric calculations, density, calculations with different concentration units;
Chemical equilibrium, Le Chatelier’s principle, equilibrium constants in terms of
concentrations, pressures and mole fractions;
Arrhenius and Bronsted acid-base theory, pH, self ionization of water, equilibrium
constants of acid-base reactions, pH of weak acid solutions, pH of very dilute solutions
and simple buffer solutions, hydrolysis of salts;
Solubility constants and solubility;
Complexation reactions, definition of coordination number, complex formation constants;
Basics of electrochemistry: Electromotive force, Nernst equation; Electrolysis, Faraday’s
laws;
Rate of chemical reactions, elementary reactions, factors affecting the reaction rate, rate
law for homogeneous and heterogeneous reactions, rate constant, reaction order, reaction
energy profile, activation energy, catalysis, influence of a catalyst on thermodynamic and
kinetic characteristics of a reaction;
Energy, heat and work, enthalpy and energy, heat capacity, Hess’ law, standard formation
enthalpies, solution, solvation and bond enthalpies;
Definition and concept of entropy and Gibbs’ energy, second law of thermodynamics,
direction of spontaneous change;
103
Ideal gas law, partial pressures;
Principles of direct and indirect titration (back titration);
Acidi- and alkalimetry, acidimetric titration curves, choice and color of indicators for
acidimetry;
Redox titrations (permanganometric and iodometric);
Simple complexometric and precipitation titrations;
Basic principles of inorganic qualitative analysis for ions specified in factual knowledge,
flame tests;
Lambert-Beer law;
Organic structure-reactivity relations (polarity, electrophilicity, nucleophilicity, inductive
effects, relative stability)
Structure-property relations (boiling point, acidity, basicity);
Simple organic nomenclature;
Hybridization and geometry at carbon centers;
Sigma and pi bonds, delocalization, aromaticity, mesomeric structures;
Isomerism (constitutional, configuration, conformation, tautomerism)
Stereochemistry (E-Z, cis-trans isomers, chirality, optical activity, Cahn-Ingold-Prelog
system, Fisher projections);
Hydrophilic and hydrophobic groups, micelle formation;
Polymers and monomers, chain polymerizations, polyaddition and polycondensation;
Laboratory skills
Heating in the laboratory, heating under reflux;
Mass and volume measurement (with electronic balance, measuring cylinder, pipette and
burette, volumetric flask);
Preparation and dilution of solutions and standard solutions;
Operation of a magnetic stirrer;
Carrying out of test tube reactions;
Qualitative testing for organic functional groups (using a given procedure);
Volumetric determination, titrations, use of a pipette bulb;
Measurement of pH (by pH paper or calibrated pH meter);
Examples of concepts and skills allowed in the exam only if included
and demonstrated in the preparatory problems
6 theoretical and 2 practical topics from these or other topics of similar breadth are allowed
in a preparatory problem set. It is intended that a topic can be introduced and discussed in
a lecture of 2-3 hours before a prepared audience.
•
VSEPR theory in detail (with more than 4 ligands);
•
Inorganic stereochemistry, isomerism in complexes;
•
Solid state structures (metals, NaCl, CsCl) and Bragg’s law;
•
Relation of equilibrium constants, electromotive force and standard Gibbs energy;
104
•
Integrated rate law for first order reactions, half-life, Arrhenius equation, determination
of activation energy;
•
Analysis of complex reactions using steady-state and quasi-equilibrium approximations,
mechanisms of catalytic reactions, determination of reaction order and activation
energy for complex reactions;
•
Collision theory
•
Simple phase diagrams and the Clausius-Clapeyron equation, triple and critical points;
•
Stereoselective transformations (diastereoselective, enantioselective), optical purity
•
Conformational analysis, use of Newman projections, anomeric effect
•
Aromatic nucleophilic substitution, electrophilic substitution on polycyclic aromatic
compounds and heterocycles
•
Supramolecular chemistry
•
Advanced polymers, rubbers, copolymers, thermosetting polymers. Polymerization
types, stages and kinetics of polymerization;
•
Amino acid side groups, reactions and separation of amino acids, protein sequencing;
•
Secondary, tertiary and quaternary structures of proteins, non-covalent interactions,
stability and denaturation, protein purification by precipitation, chromatography and
electrophoresis;
•
Enzymes and classification according to reaction types, active sites, coenzymes and
cofactors, mechanism of catalysis;
•
Monosaccharides, equilibrium between linear and cyclic forms, pyranoses and
furanoses, Haworth projection and conformational formulae;
•
Chemistry of carbohydrates, oligo- and polysaccharides, glycosides, determination of
structure;
•
Bases, nucleotides and nucleosides with formulae, Functional nucleotides, DNA and
RNA, hydrogen bonding between bases, replication, transcription and translation, DNA
based applications;
•
Complex solubility calculations (with hydrolyzing anions, complex formation);
•
Simple Schrödinger equations and spectroscopic calculations;
•
Simple MO theory;
•
Basics of mass spectrometry (molecular ions, isotope distributions);
•
Interpretation of simple NMR spectra (chemical shift, multiplicity, integrals);
•
Synthesis techniques: filtrations, drying of precipitates, thin layer chromatography.
•
Synthesis in microscale equipment;
•
Advanced inorganic qualitative analysis;
•
Gravimetric analysis;
•
Use of a spectrophotometer;
•
Theory and practice of extraction with immiscible solvents;
•
Column chromatography;
105
Appendix D
Outline of the factual knowledge supposed to be known by the
competitors:
Reactions of s-block elements with water, oxygen and halogens, their color in flame tests;
Stoichiometry, reactions and properties of binary non-metal hydrides;
Common reactions of carbon, nitrogen and sulfur oxides (CO, CO2, NO, NO2, N2O4, SO2,
SO3);
Common oxidation states of p-block elements, stoichiometry of common halides and
oxoacids (HNO2, HNO3, H2CO3, H3PO4, H3PO3, H2SO3, H2SO4, HOCl, HClO3, HClO4);
Reaction of halogens with water;
Common oxidation states of first row transition metals (Cr(III), Cr(VI), Mn(II), Mn(IV),
Mn(VII), Fe(II), Fe(III), Co(II), Ni(II), Cu(I), Cu(II), Ag(I), Zn(II), Hg(I), and Hg(II) )and the
color of these ions;
Dissolution of these metals and Al, amphoteric hydroxides (Al(OH)3, Cr(OH)3, Zn(OH)2);
Permanganate, chromate, dichromate ions and their redox reactions;
Iodometry (reaction of thiosulfate and iodine);
Identification of Ag+, Ba2+, Fe3+, Cu2+, Cl–, CO32–, SO42– ;
Organic:
Common electrophiles and nucleophiles
Electrophilic addition: addition to double and triple bonds, regioselectivity (Markovnikoff’s
rule), stereochemistry
Electrophilic substitution: substitution on aromatic rings, influence of substituents on the
reactivity and regioselectivity, electrophilic species;
Elimination: E1 and E2 reactions at sp3 carbon centers, stereochemistry, acid-base
catalysis, common leaving groups;
Nucleophilic substitution: SN1 and SN2 reactions at sp3 carbon centers, stereochemistry;
Nucleophilic addition: addition to carbon-carbon and carbon-hetero atom double and triple
bonds, addition-elimination reactions, acid-base catalysis;
Radical substitution: reaction of halogens and alkanes;
Oxidations and reductions: switching between the different oxidation levels of common
functional groups (alkyne – alkene – alkane – alkyl halide, alcohol – aldehyde, ketone –
carboxylic acid derivatives, nitriles – carbonates)
Cyclohexane conformations;
Grignard reaction, Fehling and Tollens reaction;
Simple polymers and their preparation (polystyrene, polyethylene, polyamides,
polyesters);
Amino acids and their classification in groups, isoelectric point, peptide bond, peptides and
proteins;
Carbohydrates: open chain and cyclic form of glucose and fructose;
Lipids: general formulae of triacyl glycerides, saturated and unsaturated fatty acids;
106
List of the participated Head Mentors, Mentors,
Observers, and Guests
Country
Argentina
Armenia
Australia
Austria
Azerbaijan
Belarus
Belgium
Brazil
Bulgaria
Canada
China
Chinese
Taipei
Costa Rica
Role
HM
M
HM
M
HM
M
O
O
HM
M
HM
M
HM
M
HM
M
O
HM
M
O
HM
M
HM
M
HM
M
O
O
HM
M
O
O
G
G
G
HM
M
G
G
G
Name
Maria Laura Uhrig
Vicente Gregorio Povse
Lida Sahakyan
Vitush Vano Sargsyan
Tristan Andrew Reekie
Anne Trinh
Mark John Ellison
William Cedar Jackson
Manfred Kerschbaumer
Liesbeth Berner
Igrar Nazarov
Khammad Asadov
Viktar Khvalyuk
Yauheni Paulechka
Hans Vanhoe
Cédric Malherbe
Geoffroy Kaisin
Sergio Maia Melo
Dantas Jose Arimateia Lopes
Rubens Conilho Junior
Donka Nikolova Tasheva
Penka Vasileva Tsanova
Stanislaw Skonieczny
Jeffrey David Mo
Lianyun Duan
Ping Lu
Yingxia Wang
Qiaohong He
I-Jy Chang
Wann-Yin Lin
Affiliation
Universidad de Buenos Aires
Universidad de Buenos Aires
Yerevan State Medical University
Yerevan Institute Plastpolymer
The Australian National University
University of Sydney
The Australian National University
Australian National University
Albertus Magnus Gymnasium
retired
Institute of Petrochemical Processes
Baku State University
The Belarusian State University
Belarusian State University
University of Ghent
University of Liège
University of Liège
FUNCAP
Universidade Federal do Piaui
Colegio Etapa
Sofia University “St. Kl. Ohridsky”
Sofia University “St. Kl. Ohridsky”
University of Toronto
Massachusetts Institute of Technology
Peking University
Zhejiang University
Peking University
Zhejiang University
National Taiwan Normal University
National Taiwan University
Chingfa Yao
National Taiwan Normal University
Ya-Ling Chen
Tai-Shan Fang
Szu-Chi Hsieh
Tsu-Rong George Shiau
Jose Vega
Randall Syedd - León
Ana - Rocio Madrigal - Gutierrez
Maria De Los Angeles ArguedasMadrigal
Rafael Angel Rodriguez - Campos
Taipei Municipal Jianguo High School
National Taiwan Normal University
Ministry of Education
College Entrance Examination Center
Universidad Nacional
Universidad Nacional
Universidad Nacional
107
Pensionada
Pensionado
Country
Croatia
Cuba
Cyprus
Czech
Republic
Denmark
Estonia
Finland
France
Germany
Greece
Hungary
Iceland
India
Indonesia
Role
HM
M
HM
HM
M
HM
M
Branka Zorc
Tomislav Cvitaš
Rolando Alfonso Valdes
Paraskevas Panteli
Erineos Koromias
Petr Holzhauser
Eva Muchova
HM
M
O
HM
M
HM
M
O
HM
M
O
O
HM
M
O
HM
M
HM
M
O
Kurt Bjønager Nielsen
Brian Schou Rasmussen
Nina Lock
Uno Mäeorg
Jaak Nerut
Jorma Kullervo Koskimies
Kjell Knapas
Teemu Samuel Arppe
Olivier Plaidy
Laurence Petit
Christine Hirschler
Alban A. Letailleur
Wolfgang Hampe
Alexander Rodenberg
Timo Gehring
Nikolas Psaroudakis
Eystratios Asimellis
Gyorgy Zoltan Tarczay
Szilard Varga
Attila Villanyi
HM
Finnbogi Óskarsson
M
Ísak Sigurjón Bragason
HM
Lakshmy Ravishankar
M
O
HM
M
O
O
Radha Vijay Jayaram
Pradeep Tryambakrao Deota
Riwandi Sihombing
Djulia Onggo
Deana Wahyuningrum
Ismunaryo Moenandar
G
I. R. of Iran
Name
HM
M
O
O
Hastuti Musikaningsih
Mansour Abedini
Ebrahim Kianmehr
Mahin Jabalameli
Alireza Shayesteh
108
Affiliation
University of Zagreb
University of Zagreb
IPVCE Ernesto Guevara
Ministry of Education
Ministry of Education
Institute of Chemical Technology, Prague
Faculty of Science, Charles University
Prague
Ordrup Gymnasium
University of Copenhagen
Aarhus University
University of Tartu
University of Tartu
University of Helsinki
University of Helsinki
University of Helsinki
Ministère Education Nationale
Sciences à l'Ecole
Lycée Jean Mermoz
Université Pierre et Marie Curie
University of Zuerich
Karlsruher Institute for Technology (KIT)
University of Athens
3rd EPAL of Athens, Greece
Eotvos University
Chemical Researh Center HAS
Eotvos Lorand University
Menntaskólinn í Reykjavík
(Reykjavík Junior College)
University of Iceland
V.G. Vaze College of Arts,
Science and Commerce
Institute of Chemical Technology
M.S. University of Baroda
University of Indonesia
Bandung Institute of Technology
Bandung Institute of Technology
University of Indonesia
Ministry of Education, Republic of
Indonesia
University of Tehran
University of Tehran
The Young Scholars Club
University of Tehran
Country
Ireland
Israel
Italy
Japan
Role
HM
M
O
HM
M
O
HM
M
HM
M
O
O
HM
Kazakhstan
M
G
Korea
HM
M
O
O
G
Paraic James
Matthew John Cook
Michael Anthony Cotter
Iris Barzilai
Miriam Iris Barak
Khalil Abo Nofal
Mario Anastasia
Raffaele Colombo
Nobuhiro Kihara
Asao Nakamura
Yasuhiro Yamada
Takeshi Kanazawa
Kurmangali Batyrbekovich
Bekishev
Gulnar Mukhangaliyevna
Akimzhanova
Anna Ivanovna Bekisheva
Hackjin Kim
Yunkyoung Ha
Bokyoung Lee
Do Hyun Ryu
Duckhwan Lee
G
Eun Soo Kim
G
Won Sun Jung
HM
Kuwait
Name
Barak Mehdi Hadi
M
Fotouh Abdullah Alshamali
O
Muna Ibrahim Alansari
O
G
G
Essa Menhal Alqallaf
Jasem Osamah Abul
Meshari Osamah Abul
G
Abdulaziz Ibrahim Alquraishi
G
Khalida Zaid Alzamel
HM
Minira Batkibekova
Kyrgyzstan
M
Latvia
HM
M
Fanargul Abdyldaeva
Juris Fotins
Skaidrīte Pakule
109
Affiliation
Dublin City University
Queen's University Belfast
Dublin City University
Technion-Israel Institute of Technology
Technion-Israel Institute of Technology
ST. Joseph High School and Seminary
University of Milan
University of Milan
Kanagawa University
Shibaura Institute of Technology
Tokyo University of Science
Hokkaido Sapporo Nishi High School
Kazakh National University
RSPC Daryn
Almaty Institute of Energetic and
Connection
Chungnam National University
Hongik University
Yonsei University
Sungkyunkwan University
Sogang University
Ministry of Education, Science and
Technology
Korea Foundation for the Advancement of
Science & Creativity
The Kuwait Foundation for the
Advancement of Sciences
The Kuwait Foundation for the
Advancement of Sciences
The Kuwait Foundation for the
Advancement of Sciences
Ministry of Education
Kuwait English School
Kuwait English School
The Kuwait Foundation for the
Advancement of Sciences
The Kuwait Foundation for the
Advancement of Sciences
Scientific Research Institute on Chemistry
and Technology at Kyrgyz State Technical
University Named after I. Razzakov
Karabalty Technological Institute at
Kyrgyz State Technical University named
after I. Razzakov
Latvian Institute of Organic Synthesis
University of Latvia
Country
Liechtenstein
Lithuania
Malaysia
Mexico
Moldova
Mongolia
Netherlands
New Zealand
Nigeria
Norway
Pakistan
Peru
Poland
Role
O
HM
M
HM
M
O
O
Name
Karin Andrea Birbaum
Rimantas Raudonis
Marius Jurgelenas
Noorsaadah A. Rahman
Sharifuddin Mohd. Zain
Ilani Ibrahim
Shamsir Jemain
HM
Ramiro Eugenio Dominguez
Danache
ETH Zurich
Vilnius University
Vilnius University
University of Malaya
University of Malaya
Malaysian Institute of Chemistry
Education Department
M
Eugenio Octavio Reyes
O
Carlos Mauricio Castro Acuña
Universidad Nacional Autónoma De
Mexico
Universidad Nacional Autónoma De
Mexico
Universidad Nacional Autónoma De
Mexico
State University of Moldova
State University of Moldova
National University of Mongolia
National University of Mongolia
National University of Mongolia
Gemeentelijk Gymnasium Hilversum/SLO
Enschede
SLO
SLO
University of Canterbury
The University of Auckland
National Mathematical Centre, Abuja
Ministry Of Education, Akure
National Mathematical Centre, Abuja
University of Oslo
National University Hospital
University of Karachi
University of Karachi
Pontificia Universidad Católica del Perú
University of Warsaw
University of Warsaw
HM
M
HM
M
O
Ion Bulimestru
Victor Ţapcov
Dorj Daichaa
Nyamgerel Choijilsuren
Davaasuren Sandag
HM
Peter De Groot
M
O
HM
M
O
O
O
HM
M
HM
M
HM
HM
M
Emiel De Kleijn
Johan Broens
Owen John Curnow
Duncan James Mcgillivray
Sunday Asher Adedeji
Ayodele Ajayi Akanle
Clement Olajide Adeyemo
Tor Erik Kristensen
Bjørn Dalhus
Khalid Mohammed Khan
Muhammad Raza Shah
Bertha Beatriz Flores Alor
Marek Orlik
Karolina Agnieszka Pułka
Diana Cláudia Gouveia Alves
University of Aveiro
Pinto
Maria Do Amparo Ferreira
University of Aveiro
Faustino
HM
Portugal
M
Romania
Affiliation
HM
Marius Andruh
M
Daniela Bogdan
G
Ecaterina Florica Safarica
University of Bucharest
Ministry of Education, Research,
Youth and Sports
Ecaterina Florica Safarica
110
Country
Russian
Federation
Saudi Arabia
Serbia
Singapore
Slovakia
Slovenia
Spain
Sweden
Switzerland
Role
HM
M
O
O
G
G
G
G
O
O
G
G
O
HM
M
O
O
G
HM
M
O
O
HM
M
HM
M
O
O
HM
M
O
HM
M
O
HM
Syria
Name
Vadim Eremin
Alexander Gladilin
Ilya Glebov
Elena Eremina
Elena Korchagina
Liudmila Levina
Oxana Gladilina
Sergey Lyubimov
Dhaifallah Mohammed Aldhayan
Nada Abdulaziz Aljallal
Fahad Albakr
Faisal Albakr
Dušan Sladić
Yaw Kai Yan
Sheng Zhang
Untung Edy Rusbandi
Xuhao Alvin Pek
Kok Wei Foo
Anton Sirota
Martin Putala
Pavol Tarapčík
Ján Reguli
Andrej Godec
Darko Dolenc
Juan Antonio Rodriguez Renuncio
Maria Del Carmen Causape
Cartagena
Marta Enciso
Vicente Martí Centelles
Ulf Charles Jaglid
Anna Cecilia Stenberg
Per Henning Lindgren
Dustin Hofstetter
Peter Eladio Ludwig
Basile Isidore Martin Wicky
Emad Mouafak Alazeb
Affiliation
Moscow State University
Moscow State University
Moscow Institute of Open Education
Moscow State University
Shemyakin & Ovchinnikov Institute of
Bioorganic Chemistry, Russian Academy
ofSciences
Tsentrhimpress
ZKS, LLC
Neochem Ltd
King Saud University (KSU)
King Saud University (KSU)
Vinnell Arabia
Child
University of Belgrade
National Institute of Education
National University of Singapore
National University of Singapore
Victoria Junior College
Ministry of Education, Singapore
IUVENTA
Comenius University in Bratislava
Slovak Technical University, Bratislava
Trnava University
University of Ljubljana
University of Ljubljana
University Complutense at Madrid
Universidad Politecnica de Madrid
University Complutense at Madrid
Universitat Jaume I
Chalmers University of Technology
Kungsholmens Gymnasium
Erik Dahlbergs Gymnasiet
ETH Zurich
ETH Zürich
EPFL (Swiss Federal Institute of
Technology, Lausanne)
Syrian Youth Federation and SONA
(Syrian Science Olympiad National
Authority)
M
Mohamad Majed Mohamad
Alsabbagh
Damascus University
O
Salah Mahmod Asad
Syrian Youth Federation and SONA
(Syrian Science Olympiad National
Authority)
111
Country
Tajikistan
Thailand
Turkey
Turkmenistan
Ukraine
United
Kingdom
United States
Uruguay
Venezuela
Role
HM
M
HM
M
O
O
Erhan Güler
Parviz Khakimov
Ekasith Somsook
Thammarat Aree
Aroonsiri Shitangkoon
Yongsak Sritana-Anant
G
Supunnee Chanprasert
G
HM
M
O
O
O
Name
Ibrahim Ozdemiroglu
G
G
Ayse Gulay Ataman
Jale Balci
G
Nedret Ozkar
M
The Institute for the Promotion
Teaching Science and Technology.
Techin Chuladesa
Jale Hacaloglu
Osman Ataman
Saim Ozkar
Metin Balci
Ahmet Onal
O
HM
M
HM
Affiliation
Shelale Educational Corporation
Medical University of Tajikistan
Mahidol University
Chulalongkorn University
Chulalongkorn University
The University of Chulalongkorn
Guvanchmyrat Paytakov
Mametmurat Geldiniyazov
Yuriy Kholin
Harvard University
Middle East Technical University
Middle East Technical University
Middle East Technical University
Middle East Technical University
Middle East Technical University
The Scientific and Technological Research
Council of Turkey
Middle East Technical University
Middle East Technical University
Retired from Middle East Technical
University
Turgut Ozal High School
Magtymguly University
V.N. Karazin Kharkiv National University
Institute of Innovation Technologies and
Content of Education
Galyna Malchenko
O
HM
M
O
G
HM
M
O
O
O
O
O
G
Dmytro Kandaskalov
Andrew Francis Worrall
Ben Samuel Pilgrim
Peter David Wothers
Laura Dunn
Kimberly A. Gardner
Kristin Ayn Fletcher
Gabriel Bryan Balazs
John Leon Kiappes
Cecilia D. Hernandez
Natalia L. White
Andrea Elizabeth Morris
Javier E. Hernandez Fuentes
HM
Gustavo Seoane Muniz
HM
Amalia Torrealba Sanoja
M
of
José Rafael Camacho Gutiérrez
112
National Polytechnic Institute
Harrow School
University of Oxford
The University of Cambridge
Royal Society of Chemistry
US Air Force Academy
USAFA
Lawrence Livermore National Laboratory
The Scripps Research Institute
American Chemical Society
University of Maryland
University of Maryland
Interamerican Investment Corporation
Facultad de Quimica-Universidad de la
Republica
Venezuelan
Chemistry
Olympiad
Association
Universidad Simón Bolívar
Country
Viet Nam
Role
Name
HM Son Do Quy
M
Hien Pham Dinh
O
O
G
G
G
G
Dau Nguyen Van
Chinh Nguyen Quoc
Hung Pham Tuan
Phuong Mai Chau
Hung Nguyen Van
Nhi Nguyen Thi
G
Kha Nguyen Duy
G
Hoa Tran Thi Viet
G
Diep Dao Thi Phuong
Affiliation
Vietnam Atomic Energy Commision
Viet Nam Ministry of Education and
Training
National University of Ha Noi
National University of Ho Chi Minh
Hai Phong Educational Service
Lam Son School
Lam Son School
Nguyen Trai School
Viet Nam Ministry of Education and
Training
Viet Nam Ministry of Education and
Training
Hanoi National University of Education
113
List of the competed Students
Country
Argentina
Armenia
Australia
Austria
Azerbaijan
Belgium
Bulgaria
Belarus
Brazil
Canada
Name
Ezequiel Maidanik
Nicolas Villagran Dos Santos
Mario Rugiero
Matias Lanus Mendez Elizalde
Vahagn Tamazyan
Davit Arzumanyan
Artur Aslanyan
Edvard Sargsyan
Stuart Ferrie
Allan Chau
Cyril Tang
Kelvin Cheung
Sebastian Gogg
Johannes Hellwagner
Konstantin Krautgasser
Robert Pollice
Agil Azimzada
Rashad Yusifov
Agil Safaralizade
Vugar Mirzakhanov
Jasmine De Becker
Jeroen Van Cleemput
Kevin Renier
Florence Thiry
Vasil Vasilev
School
ORT
Instituto Ballester
Hipolito Yrigoyen
Escuela
Quantum
Fizmath School
Usum
Vanadzor Fizmat
Melbourne High School
James Ruse Agricultural High School
Sydney Grammar School
James Ruse Agricultural High School
pORg der Ursulinen Graz
HTL Villach
HTL Villach
GRg Sachsenbrunn
Lenkoran Private High School
Guba Private High School
Dede Gorgud Private High School
AZ1065, Pasha Nazarli School 60
Sint-Jozefinstituut Herentals
Europese School Brussel II
Athénée Royal de Huy
Athénée Royal Nivelles
MG
Ivan Bojidarov Dimov
National High School of Mathematics and
Science, Sofia
Fani Georgieva Madzharova
National High School of Mathematics and
Science, Sofia
Tsvetan Hristov Tarnev
National High School of Mathematics and
Science, Sofia
Lizaveta Durovich
Mikhail Kavalchuk
Dzianis Kuliomin
Natallia Yelavik
Levindo Jose Garcia Quarto
Jessica Kazumi Okuma
Raul Bruno Machado Da Silva
Andre Silva Franco
Connie Zhao
Philip Sohn
Richard Liu
Brian Bi
Gimnasium 7
School 71
State Lyceum of Minsk Region
School 22
Ari de Sa
Colégio Etapa
Colegio Farias Brito
Colegio Etapa
University of Toronto Schools
Northern Secondary School
University of Toronto Schools
Woburn Collegiate Institute
114
Country
Switzerland
Name
Michelle Frei
Michele Oliosi
Yannick Suter
Alain Vaucher
Xianghang Shangguan
Zhiyao Zhou
China
Qilei Zhu
Ruyi Wang
Rafael Angel Rodriguez Arguedas
Costa Rica
Tachmajal Corrales Sanchez
Oscar Garcia Montero
Wainer Camacho Araya
Cuba
Cyprus
Czech Republic
Germany
Denmark
Spain
Estonia
Finland
Ramón Lorenzo Panades Barrueto
Panayiota Katsamba
Christos Anastassiades
Stelios Chatzimichail
Andreas Sofokli
Ondrej Hak
Ondrej Henych
Frantisek Petrous
Pavel Svec
Florian Berger
Manuel Eberl
Leonard Hasenclever
Lukas Wagner
Mads Bøttger Hansen
Jakob Bank Kodal
Kristian Holten Møller
Niels Christian Holm Sanden
Andreu Tortajada Navarro
Jesús Alvaro Gómez Iregui
Marconi Nicolás Peñas De Frutos
Pablo Giomi
Gleb Široki
Maksim Mišin
Ivan Jakovlev
Kadi Liis Saar
Suvi Kaarina Klapuri
Oscar Salomon Kivinen
Jarkko Timo Olavi Järvelä
Jari Tapio Huisman
115
School
Kantonsschule Wettingen
Gymnase Auguste Piccard
Kantonsschule Wettingen
Collège Sainte-Croix
Changjun Middle School
No.1 High School Affiliated to Central
China Normal University
Hangzhou No.2 Middle School
High School Attached to Nanjing Normal
University
Colegio Cientifico Costarricense de
Occidente
Colegio Cientifico Costarricense de San
Pedro
Colegio Nueva Esperanza
Colegio Científico Costarricense de
Occidente
IPVCE Eusebio Olivera
Lyceum Agiou Georgiou Larnaka
Lyceum Apostolon Petrou & Pavlou
Pagyprio Lyceum Larnaka
Lyceum Paphos
Gymnasium a SOS Husova 1414
Gymnasium Jeronymova 27
Gymnasium Jirovcova 8
Gymnasium Jirovcova 8
Werner-Heisenberg-Gymnasium
Gymnasium Dingolfing
Wilhelm-Ostwald-Gymnasium
Max-Planck-Gymnasium
Aabenraa Statsskole
Esbjerg Statsskole
Slagelse Gymnasium
Ordrup Gymnasium
IES Benaguasil
Colegio N.S. del Buen Consejo (Logroño)
IES Parquesol (Valladolid)
Liceo Español Luis Buñuel
Tallinna Õismäe Russian Lyceum
Tallinn Mustamäe Real Gymnasium
Tallinn Õismäe Russian Lyceum
Tallinn Secondary Sceince School
Vaasan Lyseon Lukio
Helsingin Suomalainen Yhteiskoulu
Karkkilan Yhteislyseo
Jyväskylän Normaalikoulun Lukio
Country
France
United Kingdom
Greece
Croatia
Hungary
Name
Rémi Olivier Patin
Cédric Martin
Antton Curutchet
Baptiste Couet
David Edey
David Wade
Joshua Stedman
Ruth Franklin
Nikolaos Kaplaneris
Michael Matalliotakis
Georgios Papadimitriou
Stefanos Tyros
Vranješević Filip
Markovic Igor
Kucanda Kristina
Petricevic Fran
School
Lycée Montaigne
Lycée Thiers
Lycée René Cassin
Lycée Clemenceau
Alcester Grammar School
Northgate High School
Abingdon School
Manchester High School for Girls
1 Gel Glyfada
2 Gel Irakleio Crete
8 Gel Trikalon
Bougas School
V. Gimnazija, Zagreb
V. Gimnazija, Zagreb
I. Gimnazija Zagreb
III. Gimnazija Osijek
Eszter Najbauer
Ciszterci Rend Nagy Lajos Gimnáziuma
és Kollégiuma, Pécs
Attila Sveiczer
Eötvös József Gimnázium, Budapest
Áron Szigetvári
Máté Somlyay
Indonesia
Manoel Manuputty
Alimatun Nashira
Stephen Yuwono
Agung Hartoko
India
Amit Panghal
Diptarka Hait
Nikunj Saunshi
Surendra Kotra
Ireland
I. R. of Iran
Iceland
Anandagopal Srinivasan
Daniel Quill
Dermot Gillen
Jonathan Wilson
Hanieh Safari
Hossein Dadashazar
Seyed Amirhossein Nasseri
Mohammadreza Amirmoshiri
Árni Johnsen
Helgi Björnsson
Konráð Þór Þorsteinsson
Sigtryggur Kjartansson
116
Fazekas Mihály Fővárosi Gyakorló
Általános Iskola és Gimnázium, Budapest
ELTE Apáczai Csere János Gyakorló
Gimnáziuma
SMAK Penabur Gading Serpong Indonesia
SMAN 1 Yogyakarta, Indonesis
SMAN 1 Purwokerto, Indonesia
SMA
Taruna
Nusantara
Magelang
Indonesia
Bhartiya Public School
Salt Lake School
Sathaye College
Narayana
Junior
College,
Tarnaka,
Hyderabad
Metodist College, Belfast
St. Michael's College
Marist College, Athlone
Portora Royal School
Farzanegan
Allameh Tabatabaei
Shahid Soltani
Shahid Madani
Menntaskólinn við Hamrahlíð
Menntaskólinn við Hamrahlíð
Menntaskólinn í Reykjavík
Fjölbrautaskóli Suðurnesja
Country
Israel
Italy
Japan
Kazakhstan
Kyrgyzstan
Korea
Name
School
Hadera High School
Hagimnasia Harealit, Rishon Lezion
Hadera High School
Ort Horovitz, Carmiel
Eviatar Degani
Michael Michelachvili
Assaf Mauda
Anael Ben Asher
Luciano Barluzzi
Liceo Scientifico "G. Marconi"
Di Foligno
Luca Zucchini
Alberto Branchi
Giuseppe Recchia
Hiroki Uratani
Ken-Ichi Endo
ITIS "T. Buzzi" Di Prato
ITIS "E. Fermi" Di Mantova
ITIS
Shiga Prefactual Zeze High School
Eiko Gakuen
Kengo Kataoka
Junior and Senior High School at Komaba,
University of Tsukuba
Hayate Saitoh
Zhalgas Serimbetov
Ilya Skripin
Abylay Shakhizadayev
Miras Bekbergenov
Sagynbek Dadybaev
Kalysbek Abykeshov
NADA Junior and Senior High School
Kazakh-turkish High School
Kazakh-turkish High School
Kazakh-turkish High School
Kazakh-turkish High School
Tokmok Turkish High School
NARIN Turkish High School
JALAL-ABAD
Kyrgyz-Turkish
High
School
School-licen N28 Scryabina
Seoul Science High School
Gyeonggibuk Science High School
Sejong Science High School
Sejong Science High School
AHMAD AL-BISHER ROOMI High
School
ALNAHDA High School
MARYA ALQUTIA High School
ALYARMOUK High School
Vilniaus Gabijos Gimnazija
Klaipedos "Azuolyno" Gimnazija
KTU Gymnasium
KTU Gymnasium
Riga State Gymnasium No. 1
Riga 22nd Secondary School
Riga Secondary School No. 40
Sala Secondary School
Nicolae Iorga
Prometeu-Prim
N. Milescu Spataru
LC Magdacesti
Azizbek Usvaliev
Saltanat Mambetova
Jaehyun Lim
Hyeonjae Lee
Pilkeun Jang
Won Jae Kim
Mohammad Alabdulrazzaq
Kuwait
Lithuania
Latvia
Moldova
Hessah Alquraishi
Shahad Albaloul
Mariam Aldarweesh
Vidmantas Bieliunas
Emilis Bruzas
Vladimiras Oleinikovas
Dominykas Sedleckas
Roberts Bluķis
Viktors Pozņaks
Dmitrijs Jevdokimovs
Jānis Briška
Negrescu Dan
Dosca Anastasia
Buiucli Serafim
Pîrău Tudor
117
Country
Mexico
Mongolia
Malaysia
Netherlands
Norway
New Zealand
Pakistan
Peru
Name
Raymundo Esquer-Rodriguez
Alan Carrasco-Carballo
Oscar Palomino-Hernandez
Tania Lizeth Lopez-Silva
Amarsanaa Davaasuren
Selenge Enkhtuya
Enkhbat Myagmar
Gantulga Batbayar
Yeoh Keat Hor
Nicholas Thong Li Jie
Siti Fatma Hawaria Mokhtar
Rabi'Atul Adibah 'Allauddin
Alexander Blokhuis
Anatolij Babič
Istvan Kleijn
Manuel Van Rijn
Maartje Iris Romijn
Espen Auseth Nielsen
Lars Moen Strømsnes
Ingrid Eidsvaag Andersen
Stewart Alexander
David Bellamy
Jaimin Choi
Lujia Xu
Izhar Ali
Minahil Sana Qasim
Hafiz Hassan Ali
Muhammad Anus
Luis Fernando Merma Paucar
Anthony John Salcedo Meza
School
Instituto Salvatierra
Cobao 02 El Espinal
Escuela Hispano Mexicana
CBTIS117
School No. 11, Ulaanbaatar
School No. 28, Ulaanbaatar
"Shine Mongol" School, Mongolia
School No. 14, Orkhon Province
To be announced
To be announced
To be announced
To be announced
Pleincollege van Maerlant
Het Stedelijk Lyceum Zuid
Emelwerda College
RSG Tromp Meesters
Den Norske Skolen Gran Canaria
Adolf Øiens Skole
Fauske VGS, avd. Vestmyra
Bergen Katedralskole
Christchurch Boys' High School
Christ's College
Massey High School
Massey High School
Agha Khan Higher Secondary School
Beacon House Defence Campus
Fazaia Degree College PAF Base Faisal
St. Patricks High School
Pascual Saco Oliveros
Pascual Saco Oliveros
Kornel Ocytko
I Liceum Ogolnoksztalcace, ul. Kilinskiego
7, 65-508 Zielona Gora, Poland
Marcin Malinowski
V Liceum Ogolnoksztalcace im. Ks. J.
Poniatowskiego, Warsaw
Witold Hoffmann
VIII Liceum Ogolnoksztalcace im. Adama
Mickiewicza, Poznan
Maciej Gryszel
I Liceum Ogolnoksztalcace im. M.
Kopernika, Kolobrzeg
Poland
Portugal
Gonçalo Vitorino Bonifácio
Alexandre Faia Carvalho
Marta Cristina Neves Aguiar
Jorge Pedro Martins Nogueiro
118
Secundária José Saramago
Escola secundária de Peniche
Escola Secundária Homem Cristo
Escola Secundária com 3º ciclo Emídio
Garcia
Country
Name
School
Colegiul "COSTACHE NEGRUZZI" IAŞI
Tudor Balan
Constantin Giurgiu
Liceul Teoretic ’’N.BĂLCESCU”
CLUJ-NAPOCA
Alexandru Sava
Colegiul National "VLAICU VODĂ"
CURTEA DE ARGEŞ
Romania
Russian
Federation
Singapore
Ioana Moga
Daniil Khokhlov
Kirill Sukhoverkov
Maxim Kozlov
Alexander Kochnev
Khu Boon Hou Derek
Tng Jia Hao Barry
Fong Jie Ming Nigel
Lum Jian Yang
Marek Buchman
Slovakia
Slovenia
Sweden
Ladislav Hovan
Dominik Štefanko
Marek Vician
Valter Bergant
Božidar Aničić
Žiga Perko
Nejc Petek
David Ahlstrand
Viktor Mattias Johansson
Emil Marklund
Oscar Hans Emil Mickelin
Ali Mourtada
Syria
Thailand
Tajikistan
Turkmenistan
Ali Issa
Mohammad Shubat
Rouaa Al Nan
Pinnaree Tea-Mangkornpan
Khetpakorn Chakarawet
Alif Noikham
Jiraborrirak Charoenpattarapreeda
Alisher Rakhimov
Saidali Kholzoda
Shakhboz Zulfaliev
Ulugbek Barotov
Wepa Roziyev
Myrat Annamuhammedov
Rahym Ashirov
Begmyrat Cholukov
119
Colegiul National ”E.GOJDU” ORADEA
School 167, St-Petersburg
Gimnazium 22, Barnaul
Gimnazium of Dimitrovgrad
Lyceum 230, Zarechnyi
Hwa Chong Institution
Raffles Institution
Raffles Institution
Raffles Institution
Gymnazium for Extraordinary Gifted
Children
Gymnazium, Exnárova 10
Gymnázium Andreja Vrábla, Levice
Gymnázium V.B. Nedožerského
Šolski Center Rudolfa Maistra Kamnik
II. Gimnazija Maribor
II. Gimnazija Maribor
Gimnazija Litija
Erik Dahlbergsgymnasiet
Ostrabo 1
Forsmarks Skola
Sodra Latins Gymnasium
Distinguished Student School Damascus
Syria
Distinguished Student
Distinguished Student
Distinguished Student
Triam Udom Suksa School
Trium Udom Suksa School
Mahidolwittayanusorn School
Suankularb Wittayalai
Haji Kemal Tajik-Turkish High School
Haji Kemal Tajik Turkish High School
Mavlono High School
Mavlono High School
Bashkent Turkmen-turkish High School
Bashkent Turkmen-turkish High School
Bashkent Turkmen-turkish High School
Beyik S. Turkmenbasy ad. AZCM
Country
Turkey
Chinese Taipei
Name
School
Ozel Yamanlar Fen Lisesi
Yamanlar Koleji
Istanbul Ozel Kasimoglu Coskun Fen Lisesi
Ozel Samanyolu Fen Lisesi
Taipei Municipal Jianguo High School
National Taichung First Senior High School
National Taichung First Senior High School
Taipei Municipal Jianguo High School
Fatih Alcicek
Deniz Caglin
Makbule Esen
Mehmet Cem Sahiner
Yu-Chi Kuo
Ming-Ko Cho
Wei-Che Tsai
Bo-Yun Gu
Sergiy Shyshkanov
Specialized Sanatorium Boarding School
for Gifted Children "Erudit"
Vladyslav Panarin
Specialized Sanatorium Boarding School
for Gifted Children "Erudit"
Anton Topchiy
200 Anniversary Lugansk Communal
Institution Lugansk Secondary Specialized
I-III Level Gymnasium No. 60
Dmytro Frolov
Lviv Physics and Mathematics Lyceum of
Ivan Franko Lviv National University
Melissa Bariani
Norberto Andres Canepa
Alejandro Rodriguez
Sebastian Andres Martinez
Colin Lu
Alexander Siegenfeld
Utsarga Sikder
Richard Li
Carlos Javier Berrio Barrera
Arnaldo Enmanuel Marin Suárez
Erwin Wilfredo Mora Flores
María Victoria Moreno Hernández
Binh Nguyen Duc
Cuc Mai Thu
Quang Luu Nguyen Hong
Tuan Le Anh
Liceo Nº1 Q.F. Heinzen
The British Schools
PREU
Colegio Sagrada Familia
Vestal High School
Hopkins School
South Brunswick High School
River Hill High School
Liceo Bolivariano Julio Bustamante
Colegio la Salle-Lara
Liceo Bolivariano Pedro Fontes
Institutos Educacionales Asociados
Lam Son
Nguyen Trai
PTNK Ho Chi Minh
Tran Phu
Ukraine
Uruguay
United States
Venezuela
Viet Nam
120
Country Participation Fees
Country
Years
in
2010
Fee
in USD
Fee
in JPN
Yen
Argentina
16
$1,600
¥144,000
Armenia
5
$500
¥45,000
Australia
12
$1,200
Austria
30
Azerbaijan
Country
Years
in
2010
Fee
in USD
Fee
in JPN
Yen
Kuwait
18
$1,800
¥162,000
Kyrgyzstan
11
$1,100
¥99,000
¥108,000
Latvia
20
$2,000
¥180,000
$2,000
¥180,000
Lithuania
20
$2,000
¥180,000
11
$1,100
¥99,000
Malaysia
5
$500
¥45,000
Belarus
15
$1,500
¥135,000
Mexico
19
$1,900
¥171,000
Belgium
27
$2,000
¥180,000
Moldova
4
$400
¥36,000
Brazil
12
$1,200
¥108,000
Mongolia
5
$500
¥45,000
Bulgaria
29
$2,000
¥180,000
Netherlands
8
$800
¥72,000
Canada
13
$1,300
¥117,000
New Zealand
19
$1,900
¥171,000
China
15
$1,500
¥135,000
Norway
16
$1,600
¥144,000
Chinese Taipei
5
$500
¥45,000
Pakistan
5
$500
¥45,000
Costa Rica
1
$100
¥9,000
Peru
7
$700
¥63,000
Croatia
11
$1,100
¥99,000
19
$1,900
¥171,000
Cuba
18
$1,800
¥162,000
Portugal
8
$800
¥72,000
Cyprus
Czech
Republic
Denmark
21
$2,000
¥180,000
27
$2,000
¥180,000
18
$1,800
¥162,000
3
$300
¥27,000
10
$1,000
¥90,000
Romania
Russian
Federation
Saudi Arabia
5
$500
- observe
9
$900
- absent
Singapore
21
$2,000
¥180,000
Estonia
17
$1,700
¥153,000
Slovakia
18
$1,800
¥162,000
Finland
22
$2,000
¥180,000
Slovenia
20
$2,000
¥180,000
France
20
$2,000
¥180,000
Spain
15
$1,500
¥135,000
Germany
6
$600
¥54,000
Sweden
28
$2,000
¥180,000
Greece
7
$700
¥63,000
Switzerland
24
$2,000
¥180,000
Hungary
2
$200
¥18,000
Syria
1
$100
¥9,000
Iceland
9
$900
¥81,000
Tajikistan
7
$700
- IUPAC
India
9
$900
¥81,000
Thailand
11
$1,100
¥99,000
Indonesia
11
$1,100
¥99,000
Turkey
17
$1,700
¥153,000
I. R. of Iran
18
$1,800
¥162,000
Turkmenistan
9
$900
¥81,000
Ireland
13
$1,300
¥117,000
Ukraine
17
$1,700
¥153,000
5
$500
¥45,000
1
$100
¥9,000
17
$1,700
¥153,000
United States
18
$1,800
¥162,000
0
$0
¥0
Uruguay
12
$1,200
¥108,000
13
$1,300
¥117,000
Venezuela
18
$1,800
- IUPAC
4
$400
¥36,000
15
$1,500
¥135,000
Egypt
Israel
Italy
Japan
Kazakhstan
Korea
Poland
United Kingdom
Viet Nam
121
Budget of the 42nd IChO (Estimation as of September 15.)
103 Yen
Total budget
1. Government Sources
2. Sponsors
3. Country Participation Fees
4. Observer and Guest Fees
Expenditures
1. Examination Preparation
1.1 Equipment and Reagents
1.2 Preparatory and Examination Tasks
2. Accommodation and Food
2.1 Students and Guides
2.2 Mentors etc.
3. Transportation
3.1 Students and Guides
3.2 Mentors etc.
4. Ceremonies and Banquet
4.1 Opening and Closing Ceremonies
4.2 Banquet etc.
5. Cultural Program
5.1 Students and Guides
5.2 Mentors etc.
6. Secretariat
6.1 Staff Costs
6.2 Facilities
6.3 Equipment and Services
7. Guides
8. Public Relations
8.1 Catalyzers
8.2 Souvenirs
8.3 Presentation, Mass Media
9. IT Support
10. Final Report
11. Operational Expenses
11.1 Personnel
11.2 Facilities and Services
11.3 Consumables
11.4 Communication
12. Others
13. Preliminary Events
122
103 US $
($1 = ¥90)
¥411,992
¥235,105
¥153,176
¥7,506
¥16,205
¥411,992
¥52,222
¥42,017
¥10,205
$4,578
$2,612
$1,702
$83
$180
$4,578
$580
$467
$113
¥55,307
¥24,283
¥31,024
$615
$270
$345
¥20,108
¥15,474
¥4,634
$223
$172
$51
¥31,648
¥13,246
¥18,402
$352
$147
$204
¥7,442
¥6,958
¥484
$83
$77
$5
¥81,686
¥56,122
¥16,647
¥8,917
$908
$624
$185
$99
¥19,365
¥48,890
¥5,944
¥7,478
¥35,468
$215
$543
$66
$83
$394
¥5,354
¥1,200
¥47,824
¥13,298
¥28,968
¥2,632
¥2,926
$59
$13
$531
$148
$322
$29
$33
¥14,269
¥26,677
$159
$296
List of Organizers
President
Ryoji Noyori (RIKEN)
Organizing Committee
Chair:
Deputy Chair:
Member:
Observers:
Ryoji Noyori (RIKEN)
Hiroyuki Nakanishi (Mitsui Chemicals, Inc.)
Kuniaki Tatsuta (Waseda University)
Tadashi Watanabe (The University of Tokyo)
Tetsuji Yanami (Daicel Chemical Industries Limited.)
President of Waseda University (Katsuhiko Shirai)
President of The University of Tokyo (Hiroshi Komiyama, Junichi Hamada)
President of the Association for the Progress of New Chemistry (Mitsuo Ohashi, Ryuichi
Tomizawa)
President of Catalysis Society of Japan (Makoto Imanari, Takashi Tatsumi, Miki Niwa)
President of the Ceramic Society of Japan (Teruyoshi Hiraoka, Eiichi Yasuda, Katsuji
Fujimoto, Koichi Niihara)
President of the Chemical Society of Japan (Akira Fujishima, Hiroyuki Nakanishi,
Yasuhiro Iwasawa)
President of the Electrochemical Society of Japan (Zempachi Ogumi, Makoto Yoda,
Ken-ichiro Ota, Higashi Ito)
President of Japan Chemical Industry Association (Ryuichi Tomizawa, Hiromasa
Yonekura)
President of The Japan Institute of Energy (Takao Kashiwagi, Koji Ukekawa)
President of Japan Oil Chemists' Society (Toshihiro Ito, Hiroyuki Shimasaki)
President of the Japan Petroleum Institute (Eiichi Kikuchi, Toshikazu Kobayashi)
President of The Japan Society for Analytical Chemistry (Hideaki Koizumi,
Hiroki Haraguchi, Hitoshi Watarai, Hiroshi Nakamura)
President of Japan Society for Bioscience, Biotechnology, and Agrochemistry (Shuichi
Kaminogawa, Akira Isogai, Sakayu Shimizu,)
President of The Pharmaceutical Society of Japan (Masakatsu Shibasaki, Hideo Utsumi,
Tetsuo Nagano, Norio Matsuki)
President of The Society of Chemical Engineers, Japan (Kouichi Miura, Kanji Shono,
Takashi Tsuchiya)
President of The Society of Polymer Science, Japan (Hiroyuki Nishide, Mitsuo Sawamoto)
President of The Society of Synthetic Organic Chemistry, Japan (Takeshi Nakai, Ryozo
Sakoda, Fukuyama Tohru)
Kenji Tsuboi (Japan Science Foundation)
Ken Takahashi (Mitsui Chemicals, Inc.)
Masato Ito (Soka University)
Takayuki Homma (Waseda University)
Kazuaki Kudo (The University of Tokyo)
Hiroshi Tachibana (Tokyo Metropolitan University)
Makoto Onaka (The University of Tokyo)
Yoshiyuki Sugahara (Waseda University)
Ministry of Education, Culture, Sports, Science and Technology (Kimihiko Oda,
Yasutaka Moriguchi, Shinichiro Izumi)
Ministry of Economy, Trade and Industry (Tetsuhiro Hosono, Tomofumi Hiraku)
Science Council of Japan (Yasuhiro Iwasawa, Akira Fujishima)
Japan Science and Technology Agency (Koichi Kitazawa, Toru Amano, Yutaka Hishiki)
Japan Chemical Innovation Institute (Akiyoshi Somemiya, Kazuhiko Hiyoshi)
Japan Society of Physics and Chemistry Education (Yasuo Tomioka)
Zenkoku Tyugakkou Rikakyouiku Kenkyukai (Kunio Ryuzaki, Eiji Seta, Akira Miyashita)
Japan Broadcasting Corporation (Nobuo Hayakawa)
123
The Asahi Shimbun (Atsuko Tsuji )
The Chemical Daily (Hiroshi Seta)
The Chunichi Shimbun (Hajime Hikino, Tadashi Himeno)
The Mainichi Newspapers (Yukiko Motomura, Hidetoshi Togasawa, Kazuhisa Nakai)
Nikkei Inc. (Junichi Taki)
Sankei Shimbun Co. (Shohei Nagatsuji)
The Science News Ltd (Fujita Ikeda)
The Yomiuri Shimbun (Shigeyuki Koide, Fumitaka Shibata)
Operating Managers (*: Chair)
Tadashi Watanabe (The University of Tokyo)*
Hiroyuki Nakanishi (Mitsui Chemicals, Inc.)
Kuniaki Tatsuta (Waseda University)
Tetsuji Yanami (Daicel Chemical Industries Limited.)
Masahiro Kobayashi (IChO Japan Committee Secretary General)
Finance Committee (*: Chair)
Tetsuji Yanami (Daicel Chemical Industries Limited.)*
Kayo Sakata (Daicel Chemical Industries Limited.) Fumiyuki Asano (Daicel Chemical Industries Limited.)
Michio Tanaka (Mitsui Chemicals, Inc.)
Keizo Tajima (Mitsui Chemicals, Inc.)
Nobuyuki Kawashima (The Chemical Society of Japan)
Teruto Ohta (The Chemical Society of Japan)
Hiroshi Moriya (Japan Chemical Industry Association)
Shigeo Okumura (Japan Chemical Industry Association)
Kenzo Tamura (Japan Chemical Industry Association) Ryoichi Nakano (Japan Science Foundation)
Masahiro Niwano (The Society of Polymer Science, Japan)
Masahiko Iyoda (Tokyo Metropolitan University)
Kazunori Kataoka (The University of Tokyo)
Funding Committee (*: Chair)
Hiroyuki Nakanishi (Mitsui Chemicals, Inc.)*
Satoshi Kamata (Asahi Glass Co., Ltd.)
Yuji Mizuno (Asahi Kasei Corporation)
Tetsuji Yanami (Daicel Chemical Industries Limited.)
Shunji Ehara (DIC Corporation)
Shinpei Ikenoue (Fujifilm Corporation)
Katsuki Miyauchi (Hitachi Chemical Company, Ltd.) Shuichi Ohmiya (Idemitsu Kosan Co., Ltd.)
Hozumi Sato (JSR Corporation)
Satomi Takahashi (Kaneka Corporation)
Toshiharu Numata (Kao Corporation)
Atsushi Baba (Mitsubishi Chemical Corporation)
Kuniaki Kawakami (Mitsubishi Gas Chemical Company, Inc.)
Hiroshi Tokumaru (Mitsui Chemicals, Inc.)
Kan Ueno (Nippon Oil Corporation)
Yuichi Kita (Nippon Shokubai Co., Ltd.)
Takeshi Yoshida (Sekisui Chemical Co., Ltd.)
Koji Kudo (Showa Denko K.K.)
Giichi Morita (Teijin Limited)
Tamotsu Yahata (Sumitomo Bakelite Co., Ltd.)
Hideaki Ezaki (Sumitomo Bakelite Co., Ltd.)
Yoshimasa Takao (Sumitomo Chemical Company, Limited)
Norio Tsuzumi (The Tokyo Electric Power Company, Inc.)
Nobuyuki Kuramoto (Tokuyama Corporation)
Norihiko Saito (Toray Industries, Inc.)
Hiroshige Wagatsuma (Tosoh Corporation)
Makoto Umezu (Ube Industries, Ltd.)
Kenji Tsuboi (Japan Science Foundation)
Sub-committee on Fund-raising (*: Chief)
Ken Takahashi (Mitsui Chemicals, Inc.)*
Tadashi Takamizawa (Asahi Kasei Corporation)
Hiroshi Gouda (Asahi Kasei Corporation)
Takashizu Minato (Asahi Glass Co., Ltd.
Ryutaro Yamaki (Asahi Glass Co., Ltd.)
Akiko Matsumoto (Daicel Chemical Industries Limited.)
Akira Konishi (DIC Corporation)
Takatoshi Ishikawa (Fujifilm Corporation)
Shunichi Aida (Fujifilm Corporation)
Masahiko Okamura (Hitachi Chemical Company, Ltd.)
Keiichiro Yoshi (Idemitsu Kosan Co., Ltd.)
Masaru Ohta (JSR Corporation)
Hideyuki Matsui (Kaneka Corporation)
Toru Tejima (Kao Corporation)
Kimihiko Hori (Kao Corporation)
Satoshi Kusunoki (Mitsubishi Chemical Corporation)
Hiroshi Katayama (Mitsubishi Chemical Corporation) Daiji Tsuchiyama (Mitsubishi Chemical Corporation)
Yoshinao Kashima (Mitsubishi Gas Chemical Company, Inc.)
Naruyuki Nagaoka (Mitsubishi Gas Chemical Company, Inc.)
Yasushi Kanda (Nippon Oil Corporation)
Miaki Asakawa (Nippon Shokubai Co., Ltd.)
Kiyokazu Kato (Sekisui Chemical Co., Ltd.)
Tetsuzo Ishikawa (Showa Denko K.K.)
124
Fumio Kondo (Teijin Limited)
Kaoru Sato (Sumitomo Chemical Company, Limited)
Yutaka Shiokawa (Tosoh Corporation)
Kunihiko Fujii (Tokuyama Corporation)
Tokio Obata (Ube Industries, Ltd.)
Nobuyuki Tamura (Sumitomo Bakelite Co., Ltd.)
Yuji Masuda (The Tokyo Electric Power Company, Inc.)
Syuichi Nakai (Tosoh Corporation)
Kimikazu Nagase (Toray Industries, Inc.)
Execution Committee (*: Chair)
Tadashi Watanabe (The University of Tokyo)*
Takayuki Homma (Waseda University)
Makoto Onaka (The University of Tokyo)
Kayo Sakata (Daicel Chemical Industries Limited.)
Ryoichi Nakano (Japan Science Foundation)
Masato Ito (Soka University)
Kazuaki Kudo (The University of Tokyo)
Yoshiyuki Sugahara (Waseda University)
Ken Takahashi (Mitsui Chemicals, Inc.)
Subcommittee on General Affairs (*: Chief)
Masato Ito (Soka University)*
Kensuke Arai (Nihon Pharmaceutical University)
Satoshi Arai (Waseda University)
Takayuki Homma (Waseda University)
Eiji Iwato (Tokyo Gakugei University Senior High School)
Hajime Hosoi (Waseda University)
Kensei Kobayashi (Yokohama National University) Kenya Kubo (International Christian University)
Kazuaki Kudo (The University of Tokyo)
Fumitaka Mafune (The University of Tokyo)
Makoto Minato (Yokohama National University)
Minoru Seki (Chiba University)
Akio Shimizu (Soka University)
Yoshiyuki Sugahara (Waseda University)
Makoto Onaka (The University of Tokyo)
Hiroshi Tachibana (Tokyo Metropolitan University)
Subcommittee on Public Relations (*: Chief)
Takayuki Homma (Waseda University)*
Yoshinobu Aoyama (Japan Chemical Industry Association)
Hideki Hayashi (Nagoya Municipal Industrial Research Institute)
Ayumu Inoue (Japan Chemical Industry Association)
Takaki Kanbara (University of Tsukuba)
Masaki Karayama (Toyo University)
Shigeru Machida (Tokyo National College of Technology)
Atsunori Mori (Kobe University)
Chigusa Rao (Japan Science and Technology Agency
Shigeo Satokawa (Seikei University)
Hiroshi Seta (The Chemical Daily)
Yuki Yamasaki (Hosei University)
Catalyzer Editors (*: Chief)
Haruo Hosoya*
Atsunori Mori (Kobe University)
Shigeru Machida (Tokyo National College of Technology)
Hiroshi Seta (The Chemical Daily)
Daisuke Takeushi (Tokyo Institute of Technology)
Akiko Utagawa (Tama University Hijirigaoka High School and Junior High School)
Yuki Yamasaki (Hosei University)
Preliminary Task Group (*: Chief)
Masaki Karayama (Toyo University)*
Yasunao Kuriyama (Yamagata University)
Kazuo Fujioka (Suginami Gakuin Junior and Senior High School)
Eiji Iwato (Tokyo Gakugei University Senior High School)
Shiho Kamiya (Senzoku Gakuen Junior and Senior High School)
Kazuhiro Miyamoto (The Kaisei Junior and Senior High School)
Tomohiro Watanabe (Rikkyo Niiza Junior and Senior High School)
Masakatsu Takamatsu
Yuki Yamasaki (Hosei University)
Subcommittee on Events and Transportation (*: Chief)
Kazuaki Kudo (The University of Tokyo)*
Kazuo Fujioka (Suginami Gakuin Junior and Senior High School)
Eiji Iwato (Tokyo Gakugei University Senior High School)
Masaki Karayama (Toyo University)
Yoshitaka Minai (Musashi University)
Masatada Matsuoka (Komaba Toho Junior and Senior High School )
125
Tomomi Samejima (Tokyo Gakugei University International Secondary School)
Yukihiko Ueno (Waseda University Honjo Senior High School)
Akiko Utagawa (Tama University Hijirigaoka High School and Junior High School)
Tomohiro Watanabe (Rikkyo Niiza Junior & Senior High School)
Subcommittee on Information Technology (*: Chief)
Hiroshi Tachibana (Tokyo Metropolitan University)* Shoji Matsumoto (Chiba University)
Scientific Committee (*: Chair)
Kuniaki Tatsuta (Waseda University)
Sub-committee on Practical Exam (*: Chief)
Yoshiyuki Sugahara (Waseda University)*
Nobuhiro Kanomata (Waseda University)
Jun Matsuoka (The University of Shiga Prefecture) Masahiko Murakami (Nihon University)
Kazuo Nagasawa (Tokyo University of Agriculture and Technology)
Kazuki Nakanishi (Kyoto University)
Teruyuki Nakato (Tokyo University of Agriculture and Technology)
Kenji Ogino (Tokyo University of Agriculture and Technology)
Kenichi Oyaizu (Waseda University)
Satoshi Tsukahara (Hiroshima University)
Hiroyuki Yamamoto (Waseda University)
Subcommittee on Theoretical Exam (*: Chief)
Makoto Onaka (The University of Tokyo)*
Yukihiko Hashimoto (The University of Tokyo)
Hiroyuki Kagi (The University of Tokyo)
Hitoshi Kawaji (Tokyo Institute of Technology)
Hiroshi Kondoh (Keio University)
Fumitaka Mafune (The University of Tokyo)
Masaru Miyayama (The University of Tokyo)
Akira Miyoshi (The University of Tokyo)
Kazuki Morita (The University of Tokyo)
Kiyotaka Shigehara (Tokyo University of Agriculture and Technology)
Kiyotake Suenaga (Keio University)
Toshiki Sugai (Toho University)
Koichi Tsukiyama (Tokyo University of Science)
Takeshi Wada (The University of Tokyo)
Atsuo Yasumori (Tokyo University of Science)
Naoko Yoshie (The University of Tokyo)
Secretary Office (*: Secretary General)
Masahiro Kobayashi*
Makiko Akaho
Supporting Staffs at Secretary Office
Shigeru Endo (Chemical Society of Japan)
Jun Miyasaka (Japan Science Foundation)
Yusuke Kawase (Chemical Society of Japan)
Hiroyuki Okura (Chemical Society of Japan)
Hirokazu Shimizu (VIC Computer Support)
Chigusa Rao (Japan Science and Technology Agency)
Atsuyo Yoshimi (Japan Science and Technology Agency)
Ako Imaoka
Megumi Tanabe
Guides
Students
Ayasa Aizawa (Waseda University)
Shinichi Akizuki (Soka University)
Dashdemberel Batchunag (Tokyo Institute of Technology)
Chew Sok Chen (Soka University)
Mayumi Chiba (International Christian University)
Imon Cho (International Christian University)
Andrei Dinu-lonita (Columbia University)
Takahisa Fujimori (The University of Tokyo)
Miki Fukuda (International Christian University)
Shinya Fukuzawa (The University of Tokyo)
Mitsuha Furuie (International Christian University)
Jambaldorj Ganchimeg (Tokyo Institute of Technology)
Norito Hagino (Soka University of America)
Nozomi Hayashi (University of Birmingham)
Yukei Hirasawa (International Christian University) Leung Wai Hong (Soka University)
Yuki Hori (International Christian University)
Sayuri Ikeda (International Christian University)
Kazuhiko Imai (Soka University of America)
Rie Inoue (Osaka University)
Sho Ishiwata (International Christian University)
Miho Isobe (Soka University of America)
126
Amane Iwai (International Christian University)
Taeko Iwamoto (Soka University of America)
Seiichi Izumi (Soka University of America)
Satsuki Kamihagi (University of Nottingham)
Kyohei Kanomata (Tohoku University)
Tomoko Kawabe (International Christian University)
Kazuki Kimura (Waseda University)
Shiori Kitajima (International Christian University)
Yasuhito Koda (Tokyo University of Science)
Hirotomo Kou (The University of Tokyo)
Zahariev Ivan Krasimirov (The University of Tokyo) Eri Kubota (International Christian University)
Qu Jun Lan (Soka University)
Marina Masuda (International Christian University)
Fumiko Matsushima (Soka University)
Qu Mengxuan (International Christian University)
Sa Migeum (Waseda University)
Soetrisno Misawa (DIC Co.)
Maiko Miura (Soka University of America)
Gorgoll Ricardo Mizoguchi (The University of Tokyo)
Mariko Monoi (International Christian University)
Moe Murai (International Christian University)
Kenichi Nagasawa (Soka University of America)
Misato Nakano (Takushoku University)
Mari Nakamura (Tokyo University of Agriculture and Technology)
Mika Nakaoka (Waseda University)
Nobuyuki Nakatomi (Soka University)
Minako Nishiyama (Soka University)
Mariko Nitta (Waseda University)
Remi Nozaki (Aoyama Gakuin University)
Mari Ogasa (Ochanomizu University)
Haruka Ohtake (International Christian University) Ayako Osada (International Christian University)
Naoya Otsuka (University of Tsukuba)
Izmailov Ramazan (Takushoku University)
Fatemeh Rezaeifar (The University of Tokyo)
Naomi Sakai (University of Cambridge)
Keiko Sato (Soka University of America)
Yuriko Sato (Soka University of America)
Shiori Sawasaka (International Christian University) Goh Lee See (Soka University)
Aiki Segawa (Soka University of America)
Asaka Seki (International Christian University)
Shuto Seki (International Christian University)
Mia Suda (Takushoku University)
Jun Sumida (International Christian University)
Miho Suzuki
Shunsuke Takagi (Tokyo University of Science)
Misaki Takano (International Christian University)
Masayuki Takeuchi (Soka University of America)
Haruka Tashiro (International Christian University)
Terumi Terashima (International Christian University)Minori Tomidokoro (Takushoku University)
Riho Ueno (International Christian University)
Rina Watanabe (International Christian University)
Kanae Yama (International Christian University)
Asako Yamada (Tokyo University of Foreign Studies)
Chiharu Yamamura (International Christian University)
Noyuri Yamamura (International Christian University)
Rodrigo Kendy Yamashita (The University of Tokyo) Yukiko Yano (International Christian University)
Mitsuhiro Yoshimura (The University of Tokyo)
Tomoe Yuasa (Tokyo University of Foreign Studies)
Shang Yuying (Keio University)
Mentors, Observers and Guests
Hiroko Aiso (Keio University)
Takuya Ishida (The University of Tokyo)
Yoshie Ishikawa (Soka University)
Yoshiko Sugita (Soka University of America)
Junko Taira (Soka University of America)
Miki Takata (Soka University)
Atsushi Yoshitake (Tokyo Institute of Technology)
Xu Chen (International Christian University)
Shingo Ishikawa (Hosei University)
Riku Sato (International Christian University)
Nozomu Suzuki (International Christian University)
Saki Takanashi (Yokohama National University)
Ang Foong Yee (Soka University)
Supporting Staffs
NYC
Maho Fujita (Tokyo University of Science)
Chisa Koda (International Christian University)
Sumire Kurosawa (Tokyo University of Pharmacy and Life Sciences)
Sumire Ono (The University of Tokyo)
Yoshiyuki Takasu (Soka University of America)
Yoshiki Tanaka (The University of Tokyo)
Nozomi Yoshikawa (Nihon Pharmaceutical University)
OVTA
Kazuya Aoki (The University of Tokyo)
Yohei Hattori (The University of Tokyo)
Yuki Ito (The University of Tokyo)
Kazuki Ootaka (The University of Tokyo)
Miyuki Hashimoto (International Christian University)
Satomi Inagaki (International Christian University)
Katsuyoshi Kimigafukuro (Kobe University)
Toru Takazawa (Overspec Production Studio)
127
Catalyzer
Yohei Mark Odagiri (Hosei University)
Ryu Kojima (Hosei University)
Tomohito Ide (Tokyo Institute of Technology)
Masahiro Suzuki (Hosei University )
Yoshitaka Tsuchido (Tokyo Institute of Technology)
Information Technology
Batmunkh Erdenebolor (Chiba University)
Hirokazu Kageyama (Chiba University)
Tomoko Tsuji (Chiba University)
Dai Hirahara (Chiba University)
Koji Takagi (Chiba University)
Practical Exam
Kenichi Aizawa (Waseda University)
Tomoko Akama (Waseda University)
Yuta Asai (Waseda University)
Yusuke Ariake (Waseda University)
Wonsung Choi (Waseda University)
Tomohiro Fukuda (Waseda University)
Shan Gen (Waseda University)
Ryotarou Hara (Waseda University)
Shinpei Hatano (Waseda University)
Masatoshi Hattori (Waseda University)
Syou Hideshima (Waseda University)
Yuuki Honda (Waseda University)
Keiske Iida (Tokyo University of Agriculture and Technology)
Seijiro Hosokawa (Waseda University)
Shuhei Hotta (Waseda University)
Takuya Imaoka (Tokyo University of Agriculture and Technology)
Machi Ito (Waseda University)
Masanori Ito (Waseda University)
Yuki Iwamoto (Waseda University)
Yoshie Kaifu (Waseda University)
Hisashi Kambe (Waseda University)
Taichi Kaneko (Waseda University)
Mizuki Kamishiro (Waseda University)
Yosuke Kanno (Waseda University)
Miki Kanao (Tokyo University of Agriculture and Technology)
Yasuhiro Kataoka (Waseda University)
Ryo Kawahara (Waseda University)
Kazuo Kimura (Waseda University)
Nanako Kimura (Waseda University)
Chiaki Kobayashi (Waseda University)
Manabu Kobayashi (Waseda University)
Miyuki Kobayashi (Waseda University)
Yuta Kobayashi (Waseda University)
Fumitaka Kondoh (Waseda University)
Akiko Kubota (Waseda University)
Masahiro Kunimoto (Waseda University)
Yoshiyuki Kuroda (Waseda University)
Masato Matsuda (Waseda University)
Kaoru Matsushita (Waseda University)
Ayumi Matsuo (Waseda University)
Kanao Miki (Waseda University)
Yoshihiro Minamino (Tokyo University of Agriculture and Technology)
Noriko Mitsui (Waseda University)
Katsuyoshi Miura (Waseda University)
Takahiro Mochizuki (Waseda University)
Toshiyuki Monma (Waseda University)
Yuki Mukaeda (Waseda University)
Shintaro Nagahama (Waseda University)
Mai Nagashima (Waseda University)
Makoto Nakabayashi (Waseda University)
Sho Nakagawa (Waseda University)
Satoshi Nakajima (Waseda University)
Chu Nakamura (Waseda University)
Atsushi Nakata (Waseda University)
Kuniko Nitta (Waseda University)
Takashi Niwa (Waseda University)
Toshiya Ohba (Waseda University)
Mayumi Okamoto (Waseda University)
Teruyuki Okayasu (Waseda University)
Hiroyoshi Ootsu (Waseda University)
Takanari Oouchi (Waseda University)
Shimon Osada (Waseda University)
Hiroshi Oshio (Waseda University)
Hitomi Saito (Waseda University)
Heisuke Sakai (Waseda University)
Junji Seino (Waseda University)
Keisuke Seto (Waseda University)
Toshimichi Shibue (Waseda University)
Tsuyoshi Shimada (Waseda University)
Syunsuke Sueki (Waseda University)
Teiichi Someya (Waseda University)
Natsuhiko Sugimura (Waseda University)
Kazuhiro Sugiyama (Waseda University)
Hiromi Sunaga (Waseda University)
Katsumi Suzuki (Waseda University)
Nagisa Toihara (Waseda University)
Chihiro Urata (Waseda University)
Chisato Yamazaki (Waseda University)
Yuji Takagi (Waseda University)
Katsuyuki Takahashi (Waseda University)
Nobuyuki Takahashi (Waseda University)
Yoshie Takakura (Waseda University)
Takeko Takano (Waseda University)
Shigeo Tanabe (Waseda University)
Jun Tanaka (Waseda University)
Shiro Tanie (Waseda University)
Masayuki Tera (Waseda University)
Sonoko Tokishita (Waseda University)
Hitomi Tsukagoshi (Waseda University)
Ryo Watabe (Waseda University)
128
Naoko Watanabe (Waseda University)
Masamichi Yamada (Waseda University)
Yusuke Yamamoto (Waseda University)
Naoki Yokoyama (Waseda University)
Koji Yasui (Tokyo University of Agriculture and Technology)
Shogo Yonemura (Waseda University)
Shinpei Yoshida (Waseda University)
Yuri Kodachi
Theoretical Exam
Kiyohiro Adachi (The University of Tokyo)
Mitsunori Araki (Tokyo University of Science)
Junta Fuchiwaki (The University of Tokyo)
Junichi Furukawa (The University of Tokyo)
Daijiro Hayashi (The University of Tokyo)
Hidenori Himeno (The University of Tokyo)
Yojiro Hiranuma (The University of Tokyo)
Ryota Horikawa (The University of Tokyo)
Hiroshi Hyodo (Tokyo University of Science)
Junichi Ishida (The University of Tokyo)
Tomoyuki Iwamoto (The University of Tokyo)
Takuya Kaji (The University of Tokyo)
Yusuke Kanakubo (The University of Tokyo)
Yuji Katagiri (The University of Tokyo)
Ryoichi Kira (The University of Tokyo)
Yuta Kirihara (The University of Tokyo)
Shunsuke Kodama (The University of Tokyo)
Yoichi Masui (The University of Tokyo)
Akira Matsugi (The University of Tokyo)
Hiroaki Matsumoto (The University of Tokyo)
Ken Miyajima (The University of Tokyo)
Haruko Miyake (The University of Tokyo)
Maho Morita (Keio University)
Toshiaki Nagata (The University of Tokyo)
Naoya Nakajima (Keio University)
Tomoki Nishiguchi (The University of Tokyo)
Nobuhiro Ooya (The University of Tokyo)
Yuki Saito (The University of Tokyo)
Motohiro Sakamoto (The University of Tokyo)
Kei-ichi Sato (The University of Tokyo)
Kazuhiro Shikinaka (Tokyo University of Agriculture and Technology)
Ichiro Tanabe (The University of Tokyo)
Naru Tanaka (The University of Tokyo)
Mituru Tomita (The University of Tokyo)
Kosuke Tsuchiya (Tokyo University of Agriculture and Technology)
Akira Yamada (The University of Tokyo)
Kazuya Yamada (The University of Tokyo)
Hirotaka Yamamoto (The University of Tokyo)
Sayaka Yanagida (Tokyo University of Science)
Hirotaka Yonezawa (The University of Tokyo)
Yuki Nakamura (The University of Tokyo)
Chigako Yoshida (The University of Tokyo)
Eunjin Bae (Tokyo Metropolitan Kokusai H. S.)
Winnie Chan (Tokyo Metropolitan Kokusai H. S.)
Marina Dan (Tokyo Metropolitan Kokusai H. S.)
Sara David (Tokyo Metropolitan Kokusai H. S.)
Minami Fujisawa (Tokyo Metropolitan Kokusai H. S.) Jyunka Funaki (Tokyo Metropolitan Kokusai H. S.)
Eri Hanzawa (Tokyo Metropolitan Kokusai H. S.)
Kei Kaneshiro (Tokyo Metropolitan Kokusai H. S.)
Shin Kim (Tokyo Metropolitan Kokusai H. S.)
Minori Kunii (Tokyo Metropolitan Kokusai H. S.)
Haein Lee (Tokyo Metropolitan Kokusai H. S.)
Hiromu Narita (Tokyo Metropolitan Kokusai H. S.)
Chokkaku Otobe (Tokyo Metropolitan Kokusai H. S.) Seinan Saku (Tokyo Metropolitan Kokusai H. S.)
Gabriel Sato (Tokyo Metropolitan Kokusai H. S.)
Safia Sexton (Tokyo Metropolitan Kokusai H. S.)
Mutsumi Takase (Tokyo Metropolitan Kokusai H. S.) Kaho Takeda (Tokyo Metropolitan Kokusai H. S.)
Nan Tang (Tokyo Metropolitan Kokusai H. S.)
Aki Tempaku (Tokyo Metropolitan Kokusai H. S.)
Hidehiro Ushijima (Tokyo Metropolitan Kokusai H. S.)
129
IChO
Japan Committee